You are on page 1of 179

Rs.

90

UPSC
PRELIMINARY EXAMINATION
(PAPER-I)
GENERAL STUDIES

PREVIOUS YEARS QUESTIONS

(YEAR 2010 TO 2019)


https://t.me/UPSC_PDF

UPSC
PRELIMINARY EXAMINATION
(PAPER-I)
GENERAL STUDIES

PREVIOUS YEARS QUESTIONS

(YEAR 2010 TO 2019)

THEME  INDIAN POLITY AND


GOVERNANCE

https://telegram.me/CivilServices_UPSC
https://t.me/UPSC_PDF

INDIAN POLITY AND GOVERNANCE

2019
3. The Ninth Schedule was introduced in the
constitution of India during the prime
1. Consider the following statements: ministership of
th
1. The 44 Amendment to the (a) Jawaharlal Nehru
Constitution of India introduced an (b) Lai Bahadur Shastri
Article placing the election of the
(c) Indira Gandhi
Prime Minister beyond judicial
review. (d) Morarji Desai
2. The Supreme Court of India struck 4. Consider the following statements:
down the 99th Amendment to the 1. The Parliament (Prevention of
Constitution of India as being Disqualification) Act, 1959 exempts
violative of the independence of several posts from disqualification on
judiciary. the grounds of 'Office of Profit'.
Which of the statements given above is/are 2. The above-mentioned Act was
correct? amended five times.
(a) 1 only 3. The term 'Office of Profit' is well-
(b) 2 only defined in the Constitution of India.
(c) Both 1 and 2 Which of the statements given above is/are
correct?
(d) Neither 1 nor 2
(a) 1 and 2 only
(b) 3 only
2. Consider the following statements:
(c) 2 and 3 only
1. The motion to impeach a Judge of
the Supreme Court of India cannot (d) 1, 2 and 3
be rejected by the Speaker of the Lok
Sabha as per the Judges (Inquiry) 5. Under which Schedule of the
Act, 1968. Constitution of India can the transfer of
2. The Constitution of India defines and tribal land to private parties for mining be
gives details of what constitutes declared null and void?
incapacity and proved misbehaviour' (a) Third Schedule
of the Judges of the Supreme Court (b) Fifth Schedule
of India. (c) Ninth Schedule
3. The details of the process of (d) Twelfth Schedule
impeachment of the Judges of the
Supreme Court of India are given in
6. With reference to the Constitution of India,
the Judges (Inquiry) Act, 1968.
consider the following statements :
4. If the motion for the impeachment of
1. No High Court shall have the
a Judge is taken up for voting, the
jurisdiction to declare any central law
law requires the motion to be backed
to be constitutionally invalid.
by each House of the Parliament and
supported by a majority of total 2. An amendment to the Constitution of
membership of that House and by India cannot be called into question
not less than two-thirds of total by the Supreme Court of India.
members of that House present and Which of the statements given above is/are
voting. correct?
Which of the statements given above is/are (a) 1 only
correct? (b) 2 only
(a) 1 and 2 (c) Both 1 and 2
(b) 3 only (d) Neither 1 nor 2
(c) 3 and 4 only
(d) 1, 3 and 4

https://telegram.me/CivilServices_UPSC
https://t.me/UPSC_PDF

INDIAN POLITY AND GOVERNANCE

7. In the context of polity, which one of the (a) The decisions taken by the Election
following would you accept as the most Commission of India while
appropriate definition of liberty? discharging its duties cannot be
(a) Protection against the tyranny of challenged in any court of law.
political rulers (b) The Supreme Court of India is not
(b) Absence of restraint constrained in the exercise of its
(c) Opportunity to do whatever one likes powers by laws made by the
Parliament.
(d) Opportunity to develop oneself fully
(c) In the event of grave financial crisis
in the country, the President of India
8. With reference to the management of minor
can declare Financial Emergency
minerals in India, consider the following
without the counsel from the Cabinet.
statements :
(d) State Legislatures cannot make laws
1. Sand is a 'minor mineral' according to
on certain matters without the
the prevailing law in the country.
concurrence of Union Legislature.
2. State Governments have the power
to grant mining leases of minor
11. With reference to the Legislative Assembly
minerals, but the powers regarding
of a State in India, consider the following
the formation of rules related to the
statements :
grant of minor minerals lie with the
Central Government. 1. The Governor makes a customary
address to Members of the House at
3. State Governments have the power
the commencement of the first
to frame rules to prevent illegal
session of the year.
mining of minor minerals.
2. When a State Legislature does not
Which of the statements given above is/are
have a rule on a particular matter, it
correct?
follows the Lok Sabha rule on that
(a) 1 and 3 only
matter.
(b) 2 and 3 only
Which of the statements given above is/are
(c) 3 only correct?
(d) 1, 2 and 3 (a) 1 only
(b) 2 only
9. Which one of the following suggested that (c) Both 1 and 2
the Governor should be an eminent person
(d) Neither 1 nor 2
from outside the State and should be a
detached figure without intense political
links or should not have taken part in 12. Consider the following statements :
politics in the recent past? 1. As per recent amendment to the
(a) First Administrative Reforms Indian Forest Act, 1927, forest
Commission (1966) dwellers have the right to fell the
bamboos grown on forest areas.
(b) Rajamannar Committee (1969)
2. As per the Scheduled Tribes and
(c) Sarkaria Commission (1983)
Other Traditional Forest Dwellers
(d) National Commission to Review the
(Recognition of Forest Rights) Act,
Working of the Constitution (2000)
2006, bamboo is a minor forest
produce.
10. With reference to the Constitution of India, 3. The Scheduled Tribes and Other
prohibitions or limitations or provisions Traditional Forest Dwellers
contained in ordinary laws cannot act as (Recognition of Forest Rights) Act,
prohibitions or limitations on the 2006 allows ownership of minor
constitutional powers under Article 142. It forest produce to forest dwellers.
could mean which one of the following?
Which of the statements given above is/are
correct?

https://telegram.me/CivilServices_UPSC
https://t.me/UPSC_PDF

INDIAN POLITY AND GOVERNANCE

(a) 1 and 2 only 1. Pregnant women are entitled for


(b) 2 and 3 only three months pre-delivery and three
(c) 3 only months post-delivery paid leave.
(d) 1, 2 and 3 2. Enterprises with creches must
allow the mother minimum six creche
visits daily.
13. Which Article of the Constitution of India
3. Women with two children get
safeguards one's right to marry the person
reduced entitlements.
of one's choice?
Select the correct answer using the code
(a) Article 19
given below.
(b) Article 21
(a) 1 and 2 only
(c) Article 25
(b) 2 only
(d) Article 29
(c) 3 only
(d) 1, 2 and 3
14. Consider the following statements:
1. According to the Indian Patents Act,
a biological process to create a seed 2018
can be patented in India.
2. In India, there is no Intellectual 1. If the President of India exercises his power
Property Appellate Board. as provided under Article 356 of the
3. Plant varieties are not eligible to be Constitution in respect of a particular State,
patented in India. then
Which of the statements given above is/are (a) the Assembly of the State is
correct? automatically dissolved.
(a) 1 and 3 only (b) the powers of the Legislature of that
(b) 2 and 3 only State shall be exercisable by or under the
(c) 3 only authority of the Parliament.
(d) 1, 2 and 3 (c) Article 19 is suspended in that State.
(d) the President can make laws relating to
15. Consider the following statements: that State.
1. Petroleum and Natural Gas
Regulatory Board (PNGRB) is the 2. Right to Privacy is protected as an intrinsic
first regulatory body set up by the part of Right to Life and Personal Liberty.
Government of India. Which of the following in the Constitution of
2. One of the tasks of PNGRB is to India correctly and appropriately imply the
ensure competitive markets for gas. above statement ?
3. Appeals against the decisions of (a) Article 14 and the provisions under the
PNGRB go before the Appellate 42nd Amendment to the Constitution
Tribunals for Electricity. (b) Article 17 and the Directive Principles of
Which of the statements given above are State Policy in Part IV
correct? (d) Article 21 and the freedoms guaranteed
(a) 1 and 2 only in Part III
(b) 2 and 3 only (d) Article 24 and the provisions under the
(c) 1 and 3 only 44th Amendment to the Constitution
(d) 1, 2 and 3
3. Consider the following statements :
16. Which of the following statements is/arc 1. In the first Lok Sabha, the single largest
correct regarding the Maternity Benefit party in the opposition was the Swatantra
(Amendment) Act, 2017? Party.

https://telegram.me/CivilServices_UPSC
https://t.me/UPSC_PDF

INDIAN POLITY AND GOVERNANCE

2. In the Lok Sabha, a "Leader of the 7. Regarding Money Bill, which of the following
Opposition" was recognised for the first statements is not correct ?
time in 1969. (a) A bill shall be deemed to be a Money Bill
3. In the Lok Sabha, if a party does not if it contains only provisions relating to
have a minimum of 75 members, its imposition, abolition, remission, alteration
leader cannot be recognised as the or regulation of any tax.
Leader of the Opposition. (b) A Money Bill has provisions for the
Which of the statements given above is/ are custody of the Consolidated Fund of India
correct? or the Contingency Fund of India.
(a) 1 and 3 only (c) A Money Bill is concerned with the
(b) 2 only appropriation of moneys out of the
(c) 2 and 3 only Contingency Fund of India.
(d) 1, 2 and 3 (d) A Money Bill deals with the regulation of
borrowing of money or giving of any
guarantee by the Government of India.
4. Consider the following statements
1. The Parliament of India can place a
8. With reference to the election of the
particular law in the Ninth Schedule of the
President of India, consider the following
Constitution of India.
statements :
2. The validity of a law placed in the Ninth
1. The value of the vote of each MLA varies
Schedule cannot be examined by any
from State to State.
court and no judgement can be made on
it. 2. The value of the vote of MPs of the Lok
Sabha is more than the value of the vote
Which of the statements given above is/
of MPs of the Rajya Sabha.
are correct ?
Which of the statements given above is/ are
(a) 1 only
correct ?
(b) 2 only
(a) 1 only
(c) Both 1 and 2
(b) 2 only
(d) Neither 1 nor 2
(c) Both 1 and 2
(d) Neither 1 nor 2
5. Consider the following statements :
1. Aadhaar card can be used as a proof of
9. Consider the following statements :
citizenship or domicile.
1. The Speaker of the Legislative Assembly
2. Once issued, Aadhaar number cannot be
shall vacate his/her office if he/she
deactivated or omitted by the Issuing
ceases to be a member of the Assembly.
Authority.
2. Whenever the Legislative Assembly is
Which of the statements given above is/ are
dissolved, the Speaker shall vacate
correct ?
his/her office immediately.
(a) 1 only
Which of the statements given above is/ are
(b) 2 only
correct ?
(c) Both 1 and 2
(a) 1 only
(d) Neither 1 nor 2
(b) 2 only
(c) Both 1 and 2
6. "Rule of Law Index" is released by which of the
(d) Neither 1 nor 2
following ?
(a) Amnesty International
10. Which one of the following reflects the most
(b) International Court of Justice
appropriate relationship between law and
(c) The Office of UN Commissioner for liberty ?
Human Rights
(d) World Justice Project

https://telegram.me/CivilServices_UPSC
https://t.me/UPSC_PDF

INDIAN POLITY AND GOVERNANCE

(a) If there are more laws, there is less


liberty. 14. With reference to the Parliament of India,
(b) If there are no laws, there is no liberty. which of the following Parliamentary
(c) If there is liberty, laws have to be made Committees scrutinizes and reports to the
by the people. House whether the powers to make
(d) If laws are changed too often, liberty is in regulations, rules, sub-rules, by-laws, etc.
danger. conferred by the Constitution or delegated by
the Parliament are being properly exercised
by the Executive within the scope of such
11. Consider the following statements :
delegation ?
1. No criminal proceedings shall be
(a) Committee on Government
instituted against the Governor of a State
Assurances
in any court during his term of office.
(b) Committee on Subordinate Legislation
2. The emoluments and allowances of the
(c) Rules Committee
Governor of a State shall not be
diminished during his term of office. (d) Business Advisory Committee
Which of the statements given above is/ are
correct ? 15. Consider the following statements :
(a) 1 only 1. As per the Right to Education (RTE) Act,
(b) 2 only to be eligible for appointment as a
teacher in a State, a person would be
(c) Both 1 and 2
required to possess the minimum
(d) Neither 1 nor 2
qualification laid down by the concerned
State Council of Teacher Education.
12. Which of the following are regarded as the 2. As per the RTE Act, for teaching primary
main features of the "Rule of Law" ? classes, a candidate is required to pass a
1. Limitation of powers Teacher Eligibility Test conducted in
2. Equality before law accordance with the National Council of
3. People's responsibility to the Government Teacher Education guidelines.
4. Liberty and civil rights 3. In India, more than 90% of teacher
Select the correct answer using the code education institutions are directly under
given below : the State Governments.

(a) 1 and 3 only Which of the statements given above is/ are
correct ?
(b) 2 and 4 only
(a) 1 and 2
(c) 1, 2 and 4 only
(b) 2 only
(d) 1, 2, 3 and 4
(c) 1 and 3
(d) 3 only
13. The identity platform 'Aadhaar' provides open
"Application Programming Interfaces (APIs)".
What does it imply ? 16. Consider the following statements :
1. It can be integrated into any electronic 1. The Food Safety and Standards Act,
device. 2006 replaced the Prevention of Food
2. Online authentication using iris is Adulteration Act, 1954.
possible. 2. The Food Safety and Standards Authority
Which of the statements given above is/ are of India (FSSAI) is under the charge of
correct ? Director General of Health Services in the
Union Ministry of Health and Family
(a) 1 only
Welfare.
(b) 2 only
Which of the statements given above is/ are
(c) Both 1 and 2 correct ?
(d) Neither 1 nor 2 (a) 1 only

https://telegram.me/CivilServices_UPSC
https://t.me/UPSC_PDF

INDIAN POLITY AND GOVERNANCE

(b) 2 only (c) Both 1 and 2


(c) Both 1 and 2 (d) Neither 1 nor 2
(d) Neither 1 nor 2
2. One of the implications of equality in society
17. With reference to the provisions made under is the absence of
the National Food Security Act, 2013, (a) Privileges
consider the following statements: (b) Restraints
1. The families coming under the category (c) Competition
of 'below poverty line (BPL)' only are (d) Ideology
eligible to receive subsidised food grains.
2. The eldest woman in a household, of age
3. Which principle among the following was
18 years or above, shall be the head of
added to the Directive Principles of State
the household for the purpose of
Policy by the 42nd Amendment to the
issuance of a ration card.
Constitution ?
3. Pregnant women and lactating mothers
(a) Equal pay for equal work for both men
are entitled to a 'take-home ration' of
and women
1600 calories per day during pregnancy
(b) Participation of workers in the
and for six months thereafter.
management of industries
Which of the statements given above is/ are
(c) Right to work, education and public
correct ?
assistance
(a) 1 and 2
(d) Securing living wage and human
(b) 2 only
conditions of work to workers
(c) 1 and 3
(d) 3 only
4. Which one of the following statements is
correct ?
18. International Labour Organization's (a) Rights are claims of the State against the
Conventions 138 and 182 are related to citizens.
(a) Child labour (b) Rights are privileges which, are
(b) Adaptation of agricultural practices to incorporated in the Constitution of a
global climate change State.
(c) Regulation of food prices and food (c) Rights are claims of the citizens against
security the State.
(d) Gender parity at the workplace (d) Rights are privileges of a few citizens
against the many.

2017
5. Local self-government can be best explained
as an exercise in
1. With reference to the Parliament of India, (a) Federalism
consider the following statements :
(b) Democratic decentralisation
1. A private member's bill is a bill presented
(c) Administrative delegation
by a Member of Parliament who is not
elected but only nominated by the (d) Direct democracy
President of India.
2. Recently, a private member's bill has 6. Consider the following statements :
been passed in the Parliament of India With reference to the Constitution of India,
for the first time in its history. the Directive Principles of State Policy
Which of the statements given above is/ are constitute limitations upon
correct ? 1. legislative function.
(a) 1 only 2. executive function.
(b) 2 only

https://telegram.me/CivilServices_UPSC
https://t.me/UPSC_PDF

INDIAN POLITY AND GOVERNANCE

Which of the above statements is/are correct


? 11. In the context of India, which one of the
(a) 1 only following is the correct relationship between
(b) 2 only Rights and Duties ?
(c) Both 1 and 2 (a) Rights are correlative with Duties.
(d) Neither 1 nor 2 (b) Rights are personal and hence
independent of society and Duties.
7. Which of the following statements is/are true (c) Rights, not Duties, are important, for the
of the Fundamental Duties of an Indian advancement of the personality of the
citizen ? citizen.
1. A legislative process has been provided (d) Duties, not Rights, are important for the
to enforce these duties. stability of the State.
2. They are correlative to legal duties.
Select the correct answer using the code 12. The mind of the makers of the Constitution of
given below: India is reflected in which of the following?
(a) 1 only (a) The Preamble
(b) 2 only (b) The Fundamental Rights
(c) Both 1 and 2 (c) The Directive Principles of State Policy
(d) Neither 1 nor 2 (d) The Fundamental Duties

8. Which one of the following objectives is not 13. The Parliament of India exercises control
embodied in the Preamble to the Constitution over the functions of the Council of Ministers
of India ? through
(a) Liberty of thought 1. Adjournment motion
(b) Economic liberty 2. Question hour
(c) Liberty of expression 3. Supplementary questions
(d) Liberty of belief Select the correct answer using the code
given below :
(a) 1 only
9. Democracy's superior virtue lies in the fact
that it calls into activity (b) 2 and 3 only
(a) the intelligence and character of ordinary (c) 1 and 3 only
men and women. (d) 1, 2 and 3
(b) the methods for strengthening executive
leadership. 14. For election to the Lok Sabha, a nomination
(c) a superior individual with dynamism paper can be filed by
and vision. (a) anyone residing in India.
(d) a band of dedicated party workers (b) a resident of the constituency from which
the election is to be contested.
10. The main advantage of the parliamentary (c) any citizen of India whose name appears
form of government is that in the electoral roll of a constituency.
(a) the executive and legislature work (d) any citizen of India.
independently.
(b) it provides continuity of policy and is 15. Consider the following statements :
more efficient. 1. In the election for Lok Sabha or State
(c) the executive remains responsible to the Assembly, the winning candidate must
legislature. get at least 50 percent of the votes
(d) the head of the government cannot be polled, to be declared elected.
changed without election. 2. According to the provisions laid down in
the Constitution of India, in Lok Sabha,

https://telegram.me/CivilServices_UPSC
https://t.me/UPSC_PDF

https://telegram.me/CivilServices_UPSC
https://t.me/UPSC_PDF

https://telegram.me/CivilServices_UPSC
https://t.me/UPSC_PDF

INDIAN POLITY AND GOVERNANCE

Which of the statements given above is/ are (a) Preamble


correct? (b) Directive Principles of State Policy
(a) 1 only (c) Fundamental Rights
(b) 2 only (d) Seventh Schedule
(c) Both 1 and 2
(d) Neither 1 nor 2 5. There is a Parliamentary System of
Government in India because the
(a) Lok Sabha is elected directly by the
2015
people
(b) Parliament can amend the Constitution
1. Consider the following statements: (c) Rajya Sabha cannot be dissolve
1. The Legislative Council of a State in India (d) Council of Ministers is responsible to the
can be larger in size than half of the Lok Sabha
Legislative Assembly of that particular
State.
6. Consider the following statements regarding
2. The Governor of a State nominates the
the Directive Principles of State Policy
Chairman of Legislative Council of that
particular State. 1. The Principles spell out the socio
economic democracy in the country.
Which of the statements given above is/ are
correct? 2. The provisions contained in these
Principles are not enforceable by any
(a) 1 only
court.
(b) 2 only
Which of the statements given above is/ are
(c) Both 1 and 2 correct?
(d) Neither 1 nor 2
(a) 1 only
(b) 2 only
2. "To uphold and protect the Sovereignty, Unity (c) Both 1 and 2
and Integrity of India" is a provision made in
(d) Neither 1 nor 2
the
(a) Preamble of the Constitution
7. Consider the following statements :
(b) Directive Principles of State Policy
1. The Rajya Sabha has no power either to
(c) Fundamental Rights
reject or to amend a Money Bill.
(d) Fundamental Duties
2. The Rajya Sabha cannot vote on the
Demands for Grants.
3. The Raj the fundamental object of Panchayati 3. The Rajya Sabha cannot discuss the
system is to ensure which among following? Annual Financial Statement.
(1) People's participation in development.
Which of the statements given above is/ are
(2) Political accountability correct?
(3) Democratic decentralization (a) 1 only
(4) Financial mobilization (b) 1 and 2 only
Select the correct answer using the code (c) 2 and 3 only
given below. (d) 1, 2 and 3
(a) 1, 2 and 3 only
(b) 2 and 4 only
8. When a bill is referred to a joint sitting of both
(c) 1 and 3 only the Houses of the Parliament, it has to be
(d) 1, 2, 3 and 4 passed by -
(a) a simple majority of members present
4. The ideal of Welfare State' in the Indian and voting
Constitution is enshrined in its

11

https://telegram.me/CivilServices_UPSC
https://t.me/UPSC_PDF

INDIAN POLITY AND GOVERNANCE

(b) three-fourths majority of members 12. Which of the following is the custodian of the
present and voting Constitution of India?
(c) two-thirds majority of the Houses (a) The President of India
(d) absolute majority of the Houses (b) The Prime Minister of India
(c) The Lok Sabha Secretariat
9. Consider the following statements: (d) The Supreme Court of India
(1) The Executive Power of the Union of
India is vested in the Prime Minister
2014
(2) The Prime Minister is the Ex officio
Chairman of the Civil Services Board.
Which of the statements given above is/ are 1. Consider the following statements :
correct? 1. The President shall make rules for the
(a) 1 only more convenient transaction of the
(b) 2 only business of the Government of India, and
for the allocation among Ministers of the
(c) Both 1 and 2
said business.
(d) Neither 1 nor 2
2. All executive actions of the Government
of India shall be expressed to be taken in
10. The provisions in Fifth Schedule and Siath the name of the Prime Minister.
Schedule in the Constitution of India are Which of the statements given above is/ are
made in order to correct?
(a) protect the interests of Scheduled Tribes (a) 1 only
(b) determine the boundaries between (b) 2 only
States
(c) Both 1 and 2
(c) determine the powers, authority and
(d) Neither 1 nor 2
responsibilities of Panchayats
(d) protect the interests of all the border
States 2. Consider the following statements regarding
a No-Confidence Motion in India:
1. There is no mention of a No-Confidence
11. With reference to the Union Government,
Motion in the Constitution of India.
consider the following statements:
2. A Motion of No-Confidence can be
(1) The Department of Revenue is
introduced in the Lok Sabha only.
responsible for the preparation of Union
Budget that is presented to the Which of the statements given above is/ are
Parliament. correct?
(a) 1 only
(2) No amount can be withdrawn from the
Consolidated Fund of India without the (b) 2 only
authorization from the Parliament of (c) Both 1 and 2
India. (d) Neither 1 nor 2
(3) All the disbursements made from Public
Account also need the authorization from 3. The power of the Supreme Court of India to
the Parliament of India. decide disputes between the Centre and the
Which of the statements given above is/ are States falls under its
correct? (a) advisory jurisdiction
(a) 1 and 2 only (b) appellate jurisdiction
(b) 2 and 3 only (c) original jurisdiction
(c) 2 only (d) writ jurisdiction
(d) 1, 2 and 3
4. The power to increase the number of judges
in the Supreme Court of India is vested in

12

https://telegram.me/CivilServices_UPSC
https://t.me/UPSC_PDF

INDIAN POLITY AND GOVERNANCE

(a) the President of India (c) 2 and 5 only


(b) the Parliament (d) 1, 2, 3, 4 and 5
(c) the Chief Justice of India
(d) the Law Commission 10. Which of the following is/are the
function/functions of the Cabinet Secretariat?
5. Which one of the following is the largest 1. Preparation of agenda for Cabinet
Committee of the Parliament? Meetings
(a) The Committee on Public Accounts 2. Secretarial assistance to Cabinet
(b) The Committee on Estimates Committees
(c) The Committee on Public Undertakings 3. Allocation of financial resources to the
Ministries
(d) The Committee on Petitions
Select the correct answer using the code
given below.
6. Which one of the following Schedules of the
(a) 1 only
Constitution of India contains provisions
regarding anti-defection? (b) 2 and 3 only
(a) Second Schedule (c) 1 and 2 only
(b) Fifth Schedule (d) 1, 2 and 3
(c) Eighth Schedule
(d) Tenth Schedule 11. Consider the following statements:
A Constitutional Government is one which
7. The national motto of India, 'Satyameva 1. places effective restrictions on individual
Jayate' inscribed below the Emblem of India liberty in the interest of State Authority
is taken from 2. places effective restrictions on the
(a) Katha Upanishad Authority of the State in the interest of
(b) Chandogya Upanishad individual liberty
(c) Aitareya Upanishad Which of the statements given above is/ are
(d) Mundaka Upanishad correct?
(a) 1 only
8. In the Constitution of India, promotion of (b) 2 only
international peace and security is included in
(c) Both 1 and 2
the
(d) Neither 1 nor 2
(a) Preamble to the Constitution
(b) Directive Principles of State Policy
(c) Fundamental Duties 12. Which of the following are the discretionary
(d) Ninth Schedule powers given to the Governor of a State?
1. Sending a report to the President of India
9. Which of the following are associated with for imposing the President's rule
'Planning' in India? 2. Appointing the Ministers
1. The Finance Commission 3. Reserving certain bills passed by the
2. The National Development Council State Legislature for consideration of the
3. The Union Ministry of Rural Development President of India
4. The Union Ministry of Urban 4. Making the rules to conduct the business
Development of the State Government
5. The Parliament Select the correct answer using the code
Select the correct answer using the code given below.
given below. (a) 1 and 2 only
(a) 1, 2 and 5 only
(b) 1 and 3 only
(b) 1, 3 and 4 only

13

https://telegram.me/CivilServices_UPSC
https://t.me/UPSC_PDF

INDIAN POLITY AND GOVERNANCE

(c) 2, 3 and 4 only (c) No procedure has been laid down in the
(d) 1, 2, 3 and 4 Constitution of India for the removal of a
Governor from his/her post
(d) In the case of a Union Territory having a
2013 legislative setup, the Chief Minister is
appointed by the Lt. Governor on the
basis of majority support
1. Under the Scheduled Tribes and Other
Traditional Forest Dwellers (Recognition of
Forest Rights) Act, 2006, who shall be the 5. Consider the following statements
authority to initiate, the process for 1. An amendment to the Constitution of
determining the nature and extent of India can be initiated by an introduction of
individual or community forest rights or both? a bill in the Lok Sabha only.
(a) State Forest Department 2. If such an amendment seeks to make
(b) District Collector/ Deputy changes in the federal character of the
Commissioner Constitution, the amendment also
(c) Tahsildar/Block Development Officer/ requires to be ratified by the legislature of
Mandal Revenue Officer all the States of India.
(d) Gram Sabha Which of the statements given above is/ are
correct?
(a) 1 only
2. According to the Constitution of India, which
of the following are fundamental for the (b) 2 only
governance of the country? (c) Both 1 and 2
(a) Fundamental Rights (d) Neither 1 nor 2
(b) Fundamental Duties
(c) Directive Principles of State Policy 6. Consider the following statements Attorney
(d) Fundamental Rights and General of India can
Fundamental Duties 1. take part in the proceedings of the Ink
Sabha
3. What will follow if a Money Bill is substantially 2. be a member of a committee of the Lok
amended by the Rajya Sabha? Snbha
(a) The Lok Sabha may still proceed with the 3. speak in the Lok Sabha
Bill, accepting or not accepting the 4. vote in the Lok Sabha
recommendations of the Rajya Sabba Which of the statements given above is/ are
(b) The Lok Sabha cannct consider the Bill correct?
further (a) 1 only
(c) The Lok Sabha may send the 'Bill to the (b) 2 and 4
Rajya Sabha for reconsideration (c) 2 and 3
(d) The President may call a joint sitting for (d) 1 and 3 only
passing the Bill
7. Which of the following bodies Does not/ do
4. Which one of the following statements is not find mention in the Constitution?
correct? 1. National Development Council
(a) In India, the same 'peraon cannot be 2. Planning Commission
appointed as Governor for two or more
3. Zonal Councils
States at the same time
Select the correct answer using the codes
(b) The Judges of the High Court of the
given below.
States in India are appointed by the
Governor of the State just as the Judges (a) 1 and 2 only
of the Supreme Court are appointed by (b) 2 only
the President (c) 1 and 3 only

14

https://telegram.me/CivilServices_UPSC
https://t.me/UPSC_PDF

INDIAN POLITY AND GOVERNANCE

(d) 1, 2 and 3 3. The Prime Minister shall communicate to


the President about the proposals for
8. The Parliament can make any law for whole legislation.
or arty part of India for implementing Which of the statements given above is/ are
international treaties correct?
(a) with the consent of all the States (a) 1 only
(b) with the consent of the majority of States (b) 2 and 3 only
(c) with the consent of the States concerned (c) 1 and 3only
(d) without the consent of any State (d) 1, 2 and 3

9. Consider the following statements The 12. Consider the following statements
Parliamentary Committee on Public Accounts 1. National Development Council is an
1. consists of not more than 25 Members of organ of the Planning Commission.
the Ink Sabha 2. The Economic and Social Planning is
2. scrutinizes appropriation and finance kept in the Concurrent List in the
accounts of the Government Constitution of India.
3. examines the report of the Comptroller 3. The Constitution of India prescribes that
and Auditor General of India Panchayats should be assigned the task
Which of the statements given above is/ are of preparation of plans for economic
correct? development and social justice.
(a) 1 only Which of the statements given above is/ are
correct?
(b) 2 and 3 only
(a) 1 only
(c) 3 only
(b) 2 and 3 only
(d) 1, 2 and 3
(c) 1 and 3 only
(d) 1, 2 and 3
10. In the context of India, which of the following
principles is/are implied institutionally in the
parliamentary government? 13. Consider the following statements
1. Members of the cabinet are Members of 1. The Chairmen and the Deputy Chairman
the Parliament of the Rajya Sabha are not the members
2. Ministers hold the office till they enjoy of that House.
confidence in the parliament. 2. While the nominated members of the two
3. Cabinet is headed by the Head of the Houses of the Parliament have no voting
state. right in the presidential election, they
have the right to vote in the election of
Select the correct answer using the codes
the Vice President.
given below.
Which of the statements given above is/ are
(a) 1 and 2 only
correct?
(b) 3 only
(a) 1 only
(c) 2 and 3 only
(b) 2 only
(d) 1, 2 and 3
(c) Both 1 and 2
(d) Neither 1 nor 2
11. Consider the following statements
1. The Council of Ministers in the Centre
14. With reference to National Legal Services
shall be collectively responsible to the
Authority, consider the following statements
Parliament.
1. Its objective is to provide free and
2. The Union Ministers shall hold the office
competent legal services to the weaker
during the pleasure of the President of
sections of the society on the basis of
India.
equal opportunity.

15

https://telegram.me/CivilServices_UPSC
https://t.me/UPSC_PDF

INDIAN POLITY AND GOVERNANCE

2. It issues guidelines for the State Legal become a member of the Lok Sabha
Services Authorities to implement the within six months
legal programmes and schemes (c) must be a member of one of the Houses
throughout the country. of the Parliament
Which of the statements given above is/ are (d) must be a member of the Lok Sabha
correct?
(a) 1 only 3. With reference to the Delimitation
(b) 2 only Commission, consider the following
(c) Both 1 and 2 statements :
(d) Neither 1 nor 2 1. The orders of the Delimitation
Commission cannot be challenged in a
Court of Law.
2012 2. When the orders of the Delimitation
Commission are laid before the Lok
1. In India, other than ensuring that public funds Sabha or State Legislative Assembly,
are used efficiently and for intended purpose, they cannot effect any modifications in
what is the importance of the office of the the orders.
Comptroller and Auditor General (CAG) ? Which of the statements given above is/ are
1. CAG exercises exchequer control on correct?
behalf of the Parliament when the (a) 1 only
President of India declares national (b) 2 only
emergency financial emergency. (c) Both 1 and 2
2. CAG reports on the execution of projects (d) Neither 1 nor 2
or programmes by the ministries are
discussed by the Public Accounts 4. According to the Constitution of India, it is the
Committee. duty of the President of India to cause to be
3. Information from CAG reports can be laid before the Parliament which of the
used by investigating agencies to press following?
charges against those who have violated 1. The Recommendations of the Union
the law while managing public finances. Finance Commission
4. While dealing with the audit and 2. The Report of the Public Accounts
accounting of government companies, Committee
CAG has certain judicial powers for
3. The Report of the Comptroller and
prosecuting those who violate the law.
Auditor General
Which of the statements above is/are
4. The Report of the National Commission
correct?
for Scheduled Castes
(a) 1, 3 and 4 only
Select the correct answer using the codes
(b) 2 only given below:
(c) 2 and 3 only (a) 1 only
(d) 1, 2, 3 and 4 given (b) 2 and 4 only
(c) 1, 3 and 4 only
2. The Prime Minister of India, at the time of (d) 1 , 2, 3 and 4
his/her appointment
(a) need not necessarily be a member of one
5. A deadlock between the Lok Sabha and the
of the Houses of the Parliament but must
Rajya Sabha calls for a joint sitting of the
become a member of one of the Houses
Parliament during the passage of
within six months
1. Ordinary Legislation
(b) need not necessarily be a member of one
2. Money Bill
of the Houses of the Parliament but must
3. Constitution Amendment Bill

16

https://telegram.me/CivilServices_UPSC
https://t.me/UPSC_PDF

INDIAN POLITY AND GOVERNANCE

Select the correct answer using the codes 8. In the areas covered under the Panchayat
given below : (Extension to the Scheduled Areas) Act,
(a) 1 only 1996, what the role/power of Gram Sabha?
(b) 2 and 3 only 1. Gram Sabha has the power to prevent
(c) 1 and 3 only alienation of land in the Scheduled Areas.
(d) 1, 2 and 3 2. Gram Sabha has the ownership of minor
forest produce.
3. Recommendation of Gram Sabha is
6. Which of the following is/are among the
required for granting prospecting licence
Fundamental Duties of citizens laid down in
or mining lease for any mineral in the
the Indian Constitution?
Scheduled Areas.
1. To preserve the rich heritage of our
Which of the statements given above is/ are
composite culture
correct ?
2. To protect the weaker sections from
(a) 1 only
social injustice
(b) 1 and 2 only
3. To develop the scientific temper and spirit
of inquiry (c) 2 and 3 only
4. To strive towards excellence in all (d) 1, 2 and 3
spheres of individual and collective
activity. 9. In the Parliament of India, the purpose of an
Select the correct answer using the codes adjournment motion is
given below : (a) to allow a discussion on a definite matter
(a) 1 and 2 only of urgent public importance
(b) 2 only (b) to let opposition members collect
(c) 1, 3 and 4 only information from the ministers
(d) 1, 2, 3 and 4 (c) to allow a reduction of specific amount in
demand for grant
(d) to postpone the proceedings to check the
7. What is the provision to safeguard the
inappropriate or violent behaviour on the
autonomy of the Supreme Court of India?
part of some members
1. While appointing the Supreme Court
Judges, the President of India has to
consult the Chief Justice of India. 10. The National Green Tribunal Act, 2010 was
enacted in consonance with which of the
2. The Supreme Court Judges can be
following provisions of the Constitution of
removed by the Chief Justice of India
India?
only.
1. Right to healthy environment, construed
3. The salaries of the Judges are charged
as a part of Right to life under Article 21
on the Consolidated Fund of India to
which the legislature does not have to 2. Provision of grants for raising the level of
vote. administration in the Scheduled Areas for
the welfare of Scheduled Tribes under
4. All appointments of officers and staffs of
Article 275(1)
the Supreme Court of India are made by
the Government only after consulting the 3. Powers and functions of Gram Sabha as
Chief Justice of India. mentioned under Article 243(A)
Which of the statements given above is/ are Select the correct answer using the codes
correct? given below :
(a) 1 and 3 only (a) 1 only
(b) 3 and 4 only (b) 2 and 3 only
(c) 4 only (c) 1 and 3 only
(d) 1, 2, 3 and 4 (d) 1, 2 and 3

17

https://telegram.me/CivilServices_UPSC
https://t.me/UPSC_PDF

INDIAN POLITY AND GOVERNANCE

11. Consider the following provisions under the (b) 2 and 3 only
Directive Principles of State Policy as (c) 1 and 3 only
enshrined in the Constitution of India : (d) 1, 2 and 3
1. Securing for citizens of India a uniform
civil code
14. Regarding the office of the Lok Sabha
2. Organizing village Panchayats Speaker, consider the following statements:
3. Promoting cottage industries in rural 1. He / She holds the office during the
areas pleasure of the President.
4. Securing for all the workers reasonable 2. He / She need not be a member of the
leisure and cultural opportunities House at the time of his/her election but
Which of the above Gandhian Principles that has to become a member of the House
are reflected in the Directive Principles of within six months from the date of his/her
State Policy? election.
(a) 1, 2 and 4 only 3. If he/she intends to resign, the letter of
(b) 2 and 3 only his/her resignation has to be addressed
(c) 1, 3 and 4 only to the Deputy Speaker.
(d) 1, 2, 3 and 4 Which of the statements given above is/ are
correct?
12. Consider the following statements (a) 1 and 2 only
1. Union Territories are not represented in (b) 3 only
the Rajya Sabha. (c) 1, 2 and 3
2. It is within the purview of the Chief (d) None
Election Commissioner to adjudicate the
election disputes. 15. Which of the following are included in the
3. According to the Constitution of India, the original jurisdiction of the Supreme Court?
Parliament consists of the Lok Sabha and 1. A dispute between the Government of
the Rajya Sabha only. India and one or more States
Which of the statements given above is/ are 2. A dispute regarding elections to either
correct? House of the Parliament or that of
(a) 1 only Legislature of a State
(b) 2 and 3 3. A dispute between the Government of
(c) 1 and 3 India and a Union Territory
(d) None 4. A dispute between two or more States
Select the correct answer using the codes
13. With reference to consumers' rights/ given below :
privileges under the provisions of law in India, (a) 1 and 2
which of the following statements is/are (b) 2 and 3
correct? (c) 1 and 4
1. Consumers are empowered to take (d) 3 and 4
samples for food testing.
2. When a Consumer files a complaint in 16. Which of the following special powers have
any consumer forum, no fee is required been conferred on the Rajya Sabha by the
to be paid. Constitution of India?
3. In case of death of a Consumer, his/ her (a) To change the existing territory of a State
legal heir can file a complaint in the and to change the name of a State
Consumer forum on his/her behalf.
(b) To pass a resolution empowering the
Select the correct answer using the codes Parliament to make laws in the State List
given below : and to create one or more All India
(a) 1 only Services

18

https://telegram.me/CivilServices_UPSC
https://t.me/UPSC_PDF

INDIAN POLITY AND GOVERNANCE

(c) To amend the election procedure of the


2011
President and to determine the pension
of the President after his/her retirement
(d) To determine the functions of the Election 1. The Constitution (Seventy-Third Amendment)
Commission and to determine the Act, 1992, which aims at promoting the
number of Election Commissioners Panchayati Raj Institutions in the country,
provides for which of the following?

17. Which of the following are the methods of 1. Constitution of District Planning
Committees.
Parliamentary control over public finance in
India? 2. State Election Commissions to conduct
all panchayat elections.
1. Placing Annual Financial Statement
before the Parliament 3. Establishment of State Finance
Commissions.
2. Withdrawal of moneys from Consolidated
Fund of India only after passing the Select the correct answer using the codes
Appropriation Bill given below:
3. Provisions of supplementary grants and (a) 1 only
vote-on-account (b) 1 and 2 only
4. A periodic or at least a mid-year review of (c) 2 and 3 only
programme of the Government against (d) 1, 2 and 3
macroeconomic forecasts and
expenditure by a Parliamentary Budget
2. Karl Marx explained the process of class
Office
struggle with the help of which one of the
5. Introducing Finance Bill in the Parliament following theories?
Select the correct answer using the codes (a) Empirical liberalism
given below:
(b) Existentialism
(a) 1, 2, 3 and 5 only
(c) Darwin's theory of evolution
(b) 1, 2 and 4 only
(d) Dialectical materialism
(c) 3, 4 and 5 only
(d) 1, 2, 3, 4 and 5
3. In India, if a religious sect/ community is
given the status of a national minority, what
18. Which of the following provisions of the special advantages it is entitled to?
Constitution of India have a bearing on 1. It can establish and administer exclusive
Education? educational institutions.
1. Directive Principles of State Policy 2. The President of India automatically
2. Rural and Urban Local Bodies nominates a representative of the
3. Fifth Schedule community to Lok Sabha.
4. Sixth Schedule 3. It can derive from the Prime Minister's
5. Seventh Schedule 15- Point Programme.
Select the correct answer using the codes Which of the statements given above is/ are
given below : correct?
(a) 1 and 2 only (a) 1 only
(b) 3, 4 and 5 only (b) 2 and 3 only
(c) 1, 2 and 5 only (c) 1 and 3 only
(d) 1, 2, 3, 4 and 5 (d) 1, 2 and 3

4. When the annual Union Budget is not passed


by the Lok Sabha,
(a) the Budget is modified and presented
again

19

https://telegram.me/CivilServices_UPSC
https://t.me/UPSC_PDF

INDIAN POLITY AND GOVERNANCE

(b) the Budget is referred to the Rajya Sabha The Supreme Court of India tenders advice to
for suggestions. the President of India on matters of law or
(c) the Union Finance Minister is asked to fact
resign 1. on its own initiative (on any matter of
(d) the Prime Minister submits the larger public interest).
resignation of Council of Ministers. 2. if he seeks such an advice.
3. only if the matters relate to the
5. Under the Constitution of India, which one of Fundamental Rights of the citizens.
the following is not a fundamental duty? Which of the statements given above is/ are
(a) To vote in public elections. correct?
(b) To develop the scientific temper (a) 1 only
(c) To safeguard public property (b) 2 only
(d) To abide by the Constitution and respect (c) 3 only
its ideals. (d) 1 and 2

6. Consider the following statements : 3. With reference to Lok Adalats, which of the
In India, a Metropolitan Planning Committee following statements is correct ?
1. is constituted under the provisions of the (a) Lok Adalats have the jurisdiction to settle
Constitution of India the matters at pre-litigative stage and not
2. prepares the draft development plans for those matters pending before any court
metropolitan area. (b) Lok Adalats can deal with matters which
3. has the sole responsibility for are civil and not criminal in nature
implementing Government sponsored (c) Every Lok Adalat consists of either
schemes in the metropolitan area. serving or retired judicial officers only and
Which of the statements given above is/ are not any other person
correct? (d) None of the statements given above is
(a) 1 and 2 only correct
(b) 2 only
(c) 1 and 3 only 4. The "Instrument of Instructions" contained in
the Government of India Act 1935 have been
(d) 1, 2 and 3
incorporated in the Constitution of India in the
year 1950 as
2010 (a) Fundamental Rights
(b) Directive Principles of State Policy
1. With reference to the Constitution of India, (c) Extent of executive power of State
consider the following: (d) Conduct of business of the Government
1. Fundamental Rights of India
2. Fundamental Duties
3. Directive Principles of State Policy 5. Who of the following shall cause every
recommendation made by the Finance
Which of the above provisions of the
Commission to be laid before each House of
Constitution of India is/are fulfilled by the
Parliament ?
National Social Assistance Programme
launched by the Government of India? (a) The President of India
(a) 1 only (b) The Speaker of Lok Sabha
(b) 3 only (c) The Prime Minister of India
(c) 1 and 3 only (d) The Union Finance Minister
(d) 1, 2 and 3

2. Consider the following statements:

20

https://telegram.me/CivilServices_UPSC
https://t.me/UPSC_PDF

INDIAN POLITY AND GOVERNANCE

6. Which one of the following is responsible for as to the principles for determining the taxes
the preparation and presentation of Union and duties which may be appropriated by the
Budget to the Parliament? Panchayats in that particular State?
(a) Department of Revenue (a) District Planning Committees
(b) Department of Economic Affairs (b) State Finance Commission
(c) Department of Financial Services (c) Finance Ministry of that State
(d) Department of Expenditure (d) Panchayati Raj Ministry of that State

7. Which one of the following authorities makes


recommendation to the Governor of a State

21

https://telegram.me/CivilServices_UPSC
https://t.me/UPSC_PDF

ANSWERS - INDIAN POLITY AND GOVERNANCE

ANSWERS OF INDIAN POLITY AND GOVERNANCE

2019 12. (c) 1, 2 and 4 only


13. (c) Both 1 and 2
1. (b) 2 only 14. (b) Committee on Subordinate
Legislation
2. (c) 3 and 4 only
15. (a) 1 and 2
3. (a) Jawaharlal Nehru
16. (a) 1 only
4. (a) 1 and 2 only
17. (b) 2 only
5. (b) Fifth Schedule
18. (a) Child labour
6. (d) Neither 1 nor 2
7. (d) Opportunity to develop oneself fully
8. (a) 1 and 3 only 2017
9. (c) Sarkaria Commission (1983)
10. (b) The Supreme Court of India is not 1. (d) Neither 1 nor 2
constrained in the exercise of its 2. (a) Privileges
powers by laws made by the 3. (b) Participation of workers in the
Parliament. management of industries
11. (a) 1 only 4. (c) Rights are claims of the citizens
12. (b) 2 and 3 only against the State.
13. (b) Article 21 5. (b) Democratic decentralisation
14. (c) 3 only 6. (d) Neither 1 nor 2
15. (b) 2 and 3 only 7. (d) Neither 1 nor 2
16. (c) 3 only 8. (b) Economic liberty
9. (a) the intelligence and character of
ordinary men and women.
2018 10. (c) the executive remains responsible to
the legislature.
1. (b) the powers of the Legislature of that 11. (a) Rights are correlative with Duties.
State shall be exercisable by or 12. (a) The Preamble
under the authority of the Parliament. 13. (d) 1, 2 and 3
2. (c) Article 21 and the freedoms 14. (c) any citizen of India whose name
guaranteed in Part III appears in the electoral roll of a
3. (b) 2 only constituency.
4. (a) 1 only 15. (d) Neither 1 nor 2
5. (b) 2 only 16. (c) Constitutional Right
6. (d) World Justice Project 17. (d) 3 only
7. (c) A Money Bill is concerned with the 18. (a) the power of the Judiciary to
appropriation of moneys out of the pronounce upon the constitutionality
Contingency Fund of India. of laws and executive orders.
8. (a) 1 only 19. (b) 1 and 3 only
9. (a) 1 only 20. (c) 1 and 4 only
10. (b) If there are no laws, there is no 21. (b) A mechanism for speeding up the
liberty. activities of the Government whose
11. (c) Both 1 and 2
22

https://telegram.me/CivilServices_UPSC
https://t.me/UPSC_PDF

ANSWERS - INDIAN POLITY AND GOVERNANCE

responsibilities are increasing day by 8. (b) Directive Principles of State Policy


day. 9. (c) 2 and 5 only
22. (d) A device for strengthening the hands 10. (c) 1 and 2 only
of the head of the Government 11. (b) 2 only
whose hold over the people is in a
12. (b) 1 and 3 only
state of decline.

2013
2016
1 (d) Gram Sabha
1. (c) 1, 2 and 4 2 (c) Directive Principles of State Policy
2. (b) 2 only 3 (a) The Lok Sabha may still proceed
3. (d) Rajya Sabha by a majority of not less with the Bill, accepting or not
than two-thirds of its members accepting the recommendations of
present and voting the Rajya Sabba
4. (b) 2 only 4 (c) No procedure has been laid down in
5. (b) 2 only the Constitution of India for the
6. (d) Neither 1 nor 2 removal of a Governor from his/her
post
2015 5 (d) Neither 1 nor 2
6 (c) 2 and 3
7 (d) 1, 2 and 3
1 (d) Neither 1 nor 2
8 (d) without the consent of any State
2. (d) Fundamental Duties
9 (b) 2 and 3 only
3. (c) 1 and 3 only
10 (a) 1 and 2 only
4. (b) Directive Principles of State Policy 11 (b) 2 and 3 only
5. (d) Council of Ministers is responsible to 12 (b) 2 and 3 only
the Lok Sabha
13 (b) 2 only
6. (c) Both 1 and 2 14 (c) Both 1 and 2
7. (b) 1 and 2 only
8. (a) a simple majority of members 2012
present and voting
9. (d) Neither 1 nor 2
1 (b) 2 only
10. (a) protect the interests of Scheduled
In India, CAG has no power in the
Tribes
capacity of comptroller as he does not
11. (c) 2 only exercise any type of control on
12. (d) The Supreme Court of India exchequer CAG reports are discussed
in the PAC meetings and it can call the
2014 CAG to attend its meetings. CAG does
not have any judicial powers for
prosecuting anyone.
1. (a) 1 only
2 (a) need not necessarily be a member of
2. (c) Both 1 and 2 one of the Houses of the Parliament
3. (c) original jurisdiction but must become a member of one of
4. (b) the Parliament the Houses within six months
5. (b) The Committee on Estimates It is nowhere written in the constitution
6. (d) Tenth Schedule that the P.M. at the time of
7. (d) Mundaka Upanishad appointment must be a member of any

23

https://telegram.me/CivilServices_UPSC
https://t.me/UPSC_PDF

ANSWERS - INDIAN POLITY AND GOVERNANCE

house of the Parliament. But he The Primary object of an adjournment


should be elected within six months motion is to draw the attention of the
either in Rajya Sabha or Lok Sabha. House to a recent matter of urgent
3 (c) Both 1 and 2 public importance.
The Delimitation Commission in India 10 (a) 1 only
is a high power body whose orders 11 (b) 2 and 3 only
have the force of law and cannot be Art. 40 : Establishment of village
called in question before any court of Panchayat.
law. These orders come into force on Art. 43 : Promotion of Cottage
a date to be specified by the President industry.
of India. The copies of its orders are
12 (d) None
laid before the House of people and
the State legislative assembly The Rajya Sabha is composed of the
concerned but no modifications are representatives of States and the two
permissible therein by them. union territories of Delhi and
Puducherry and 12 members are
4 (c) 1, 3 and 4 only
nominated by the President of India.
The following articles of the
constitution provide for the formation The election disputes are adjudicated
of the institution as well as laying by the special election benches
down of its reports by the President. designated for election petitions that
should only be formed in the High
Article 280 Finance Constitution.
Court. The Indian Parliament consists
Article 338 National Commission for
of the President of India, the Lok
Scheduled Castes.
Sabha and the Rajya Sabha.
Article 148 to 151 Comptroller and
Auditor General. 13 (c) 1 and 3 only
Public Accounts Committee is not The Consumer Protection Act, 1986
even mentioned in the Constitution. sought to provide better protection of
the intersects of consumers and for
5 (a) 1 only
that purpose, made provisions for the
Deadlock between the Lok Sabha and establishment of consumer councils
the Rajya Sabha calls for a joint sitting and other authorities for the settlement
of the Parliament during the passage of consumer disputes and for matters
of ordinary legislation only. No joint connected therewith.
sitting is possible in case of money bill
14 (b) 3 only
and constitutional amendment bill.
The Speaker of the Lok Sabha is
6 (c) 1, 3 and 4 only
elected from the members of the Lok
Fundamental Duties are contained in Sabha and holds the office till the
Article 51A of the Constitution. constitution of new Lok Sabha or till he
7 (a) 1 and 3 only is removed. The Speaker can vacate
Article 124 of the Constitution provides the office by rendering his resignation
that President has to consult the Chief addressed to the Deputy Speaker.
Justice of India in the appointment to 15 (c) 1 and 4
Supreme Court Judges. Article 146 Article 131 gives information about the
provides that salaries of judges are
original Jurisdiction of the Supreme
charged on the Consolidated Fund of Court.
India.
8 (d) 1, 2 and 3 16 (b) To pass a resolution empowering the
Parliament to make laws in the State
9 (a) to allow a discussion on a definite
matter of urgent public importance

24

https://telegram.me/CivilServices_UPSC
https://t.me/UPSC_PDF

ANSWERS - INDIAN POLITY AND GOVERNANCE

List and to create one or more All India To vote in public elections is not a
Services. fundamental duties under the
Rajya Sabha is empowered under Art. Constitution of India.
249(i) and Art. 312(i) to pass 6. (a) 1 and 2 only
resolution empowering the parliament Statement (3) is incorrect. The
to make laws on the subject Metropolitan Planning Committee
enumerated in the State List and to plays the role of an advisor rather
create one or more all India services than an executive role.
respectively.
17 (a) 1, 2, 3 and 5 only 2010
Parliament exercise following types
control over the expenditure : Placing 1. (b) 3 only
Annual Financial statement before the
National Social Assistance
Parliament under Art. 112. Under Art.
Programme (NSAP) is a social
114, it is given that no money can be
assistance programme for poor
withdrawn from Consolidated Fund of
households. It was launched on 15th
India without passing appropriation
August, 1995 and represents a
Act. Provisions of Supplementary
significant step towards the fulfillment
grants and vote-on-account is given in
of the Directive Principles of State
Art. 115 and Article 116. Article 107
Policy in Articles 41 and 42 the
stipulates provisions for finance bill.
Constitution, recognizing the
18 (d) 1, 2, 3, 4 and 5 concurrent responsibilities of the
Central and the State Governments
2011 in the matter.
Various schemes under NSAP also
1. (c) 2 and 3 only fulfill the criteria Right to Life, given
Statement (1) is incorrect. The under Article 21 of the Constitution
Constitution of District Planning (Fundamental Right).
Committee is as per the 74th
2. (b) 2 only
Constitutional Amendment and not
by the 73rd Constitutional Article – 143 of the Constitution deals
Amendment. with the – powers of the President to
Consult Supreme Court. If at any
2. (d) Dialectical materialism
time it appears to the President that
Karl Marx explained the process of
a question of law or fact has arisen,
clan struggle into the help of the
or is likely to arise, which is of such a
“Theory of Dialectical Materialism”. nature and of such public importance
3. (c) 1 and 3 only that it is expendient to obtain the
Statement (2) is incorrect. There is opinion of the Supreme Court upon
no special mention of minorities in it, he (the President) can share such
the Constitution. an advice
4. (d) the Prime Minister submits the 3. (d) None of the statements given above
resignation of Council of Ministers. is correct
When the annual Union Budget is not The jurisdiction of the Lok Adalats is
passed by the Lok Sabha, the Prime- wide, any matter falling within the
Minister submits the resignation of jurisdiction of civil, criminal or
the Council of Ministers. revenue courts or tribunals can be
5. (a) To vote in public elections. taken up. What about in – Direction
of Civil Statements (a) & (c).

25

https://telegram.me/CivilServices_UPSC
https://t.me/UPSC_PDF

ANSWERS - INDIAN POLITY AND GOVERNANCE

4. (b) Directive Principles of State Policy


The Directive Principles of State
Policy are like instruments of
instructions, which were issued to the
Governor-General and Governors of
the colonies by the British
Government under the Governor of
India Act of 1935 Act the same were
incorporated Colomn in 1950 in the
Constitution of India.
5. (a) The President of India
The President of India shall cause
every recommendation made by the
Finance Commision to be laid before
each House of the Parliament.
6. (b) Department of Economic Affairs
The Department of Economic Affairs
(DEA) of the Ministry of Finance is
the nodal agency of the Union
Government to formulate and
monitor the country’s economic
politics and programmes having a
bearing on domestic and
international aspects of the economic
management. A principal
responsibility of this Department is
the preparation of the Union Budget
annually (excluding the Railway
Budget).
7. (b) State Finance Commission
According to Article 243-I of the
Indian Constitution, the Governor of
a state shall constitute a Finance
Commission to review the financial
position of the Panchayats.

26

https://telegram.me/CivilServices_UPSC
https://t.me/UPSC_PDF

UPSC
PRELIMINARY EXAMINATION
(PAPER-I)
GENERAL STUDIES

PREVIOUS YEARS QUESTIONS

(YEAR 2010 TO 2019)

THEME  HISTORY OF INDIA AND


INDIAN NATIONAL MOVEMENT

https://telegram.me/CivilServices_UPSC
https://t.me/UPSC_PDF

HISTORY OF INDIA AND INDIAN NATIONAL MOVEMENT

1. In the revenue administration of Delhi


2019
Sultanate, the in-charge of revenue
collection was known as 'Amil'.
1. Which one of the following is not a 2. The Iqta system of Sultans of Delhi
Harappan site? was an ancient indigenous institution.
(a) Chanhudarp 3. The office of 'Mir Bakshi' came into
(b) Kot Djii existence during the reign of Khalji
(c) Sohgaura Sultans of Delhi.
(d) Desalpur Which of the statements given above is/are
correct?
2. In which of the following relief sculpture (a) 1 only
inscriptions is 'Ranyo Ashoka' (King (b) 1 and 2 only
Ashoka) mentioned along with the stone (c) 3 only
portrait of Ashoka? (d) 1, 2 and 3
(a) Kanganahalli
(b) Sanchi 7. Consider the following statements:
(c) Shahbazgarhi 1. Saint Nimbarka was a contemporary
(d) Sohgaura of Akbar.
2. Saint Kabir was greatly influenced by
3. Consider the following: Shaikh Ahmad Sirhindi.
1. Deification of the Buddha. Which of the statements given above is/are
2. Treading the path of Bodhisattvas correct?
3. Image worship and rituals. (a) 1 only
Which of the above is/are the feature/ (b) 2 only
features of Mahayana Buddhism? (c) Both 1 and 2
(a) 1 only (d) Neither 1 nor 2
(b) 1 and 2 only
(c) 2 and 3 only 8. With reference to Mian Tansen, which one
(d) 1, 2 and 3 of the following statements is not correct?
(a) Tansen was the title given to him by
4. With reference to forced labour (Vishti) in Emperor Akbar.
India during the Gupta period, which one of (b) Tansen composed Dhrupads on
the following statements is correct? Hindu gods and goddesses.
(a) It was considered a source of income (c) Tansen composed songs on his
for the State, a sort of tax paid by the patrons.
people. (d) Tansen invented many Ragas.
(b) It was totally absent in the Madhya
Pradesh and Kathiawar regions of 9. Who among the following Mughal
the Gupta Empire. Emperors shifted emphasis from illustrated
(c) The forced labourer was entitled to manuscripts to album and individual
weekly wages. portrait?
(d) The eldest son of the labourer was (a) Humayun
sent as the forced labourer. (b) Akbar
(c) Jahangir
5. Building 'Kalyaana Mandapas' was a (d) Shah Jahan
notable feature in the temple
construction in the kingdom of 10. With reference to Mughal India, what is/are
(a) Chalukya the difference/differences between Jagirdar
(b) Chandela and Zamindar?
(c) Rashtrakuta 1. Jagirdars were holders of land
(d) Vijayanagara assignments in lieu of judicial and
police duties, whereas Zamindars
6. Consider the following statements: were holders of revenue rights

https://telegram.me/CivilServices_UPSC
https://t.me/UPSC_PDF

HISTORY OF INDIA AND INDIAN NATIONAL MOVEMENT

without obligation to perform any duty Neogy Constituent


other than revenue collection. Assembly
2. Land assignments to Jagirdars were 3. P. C. Joshi : General Secretary,
hereditary and revenue rights of Communist Party of
Zamindars were not hereditary. India
Select the correct answer using the code
Which of the pairs given above is/are
given below.
correctly matched?
(a) 1 only
(a) 1 only
(b) 2 only
(b) 1 and 2 only
(c) Both 1 and 2
(c) 3 only
(d) Neither 1 nor 2
(d) 1, 2 and 3

11. Consider the following pairs:


14. With reference to land reforms in
Famous place : River independent India, which one of the
1. Pandharpur : Chandrabhaga following statements is correct?
2. Tiruchirappalli : Cauvery (a) The ceiling laws were aimed at family
3. Hampi : Malaprabha holdings and not individual holdings.
(b) The major aim of land reforms was
Which of the pairs given above are providing agricultural land to all the
correctly matched? landless.
(a) 1 and 2 only (c) It resulted in cultivation of cash crops
(b) 2 and 3 only as a predominant form of cultivation.
(c) 1 and 3 only (d) Land reforms permitted no
(d) 1, 2 and 3 exemptions to the ceiling limits.

12. With reference to the British colonial rule in 15. Consider the following statements about
India, consider the following statements: 'the Charter Act of 1813':
1. Mahatma Gandhi was instrumental in 1. It ended the trade monopoly of the
the abolition of the system of East India Company in India except
Indentured labour'. for trade in tea and trade with China.
2. In Lord Chelmsford's War 2. It asserted the sovereignty of the
Conference', Mahatma Gandhi did British Crown over the Indian
not support the resolution on territories held by the Company
recruiting Indians for World War. 3. The revenues of India were now
3. Consequent upon the breaking of controlled by the British Parliament.
Salt Law by Indian people, the Indian Which of the statements given above are
National Congress was declared correct?
illegal by the colonial rulers. (a) 1 and 2 only
Which of the statements given above are (b) 2 and 3 only
correct? (c) 1 and 3 only
(a) 1 and 2 only (d) 1, 2 and 3
(b) 1 and 3 only
(c) 2 and 3 only 16. With reference to Swadesh Movement
(d) 1, 2 and 3 consider the following statements:
1. It contributed to the revival of the
13. With reference to Indian National indigenous artisan crafts and
Movement, consider the following pairs: industries.
Person : Position held 2. The National Council of Education
1. Sir Tej : President, All India was established as a part of
Bahadur Liberal Federation Swadeshi Movement.
Sapru Which of the statements given above is/are
2. K. C. : Member, The correct?
(a) 1 only

https://telegram.me/CivilServices_UPSC
https://t.me/UPSC_PDF

HISTORY OF INDIA AND INDIAN NATIONAL MOVEMENT

(b) 2 only (c) Bankim Chandra Chattopadhyay wrote


(c) Both 1 and 2 Anandmath.
(d) Neither 1 nor 2 (d) Satyendranath Tagore became the first
Indian to succeed in the Indian Civil
17. Consider the following pairs: Services Examination.
Movement/Organiza : Leader
tion
4. With reference to educational institutions
1. All India Anti- : Mahatma Gandhi during colonial rule in India, consider the
Untouchability following pairs :
League
2. All India Kisan : Swami Institution Founder
Sabha Sahajanand 1. Sanskrit College at - William Jones
Saraswati Benaras
3. Self-Respect : E. V. Ramaswami
Movement Naicker 2. Calcutta Madarsa - Warren
Hastings
Which of the pairs given above is/are
correctly matched? 3. Fort William - Arthur
(a) 1 only College Wellesley
(b) 1 and 2 only Which of the pairs given above is/are correct ?
(c) 2 and 3 only (a) 1 and 2
(d) 2 and 3
(b) 2 only
(c) 1 and 3
2018 (d) 3 only

1. Which of the following led to the introduction 5. After the Santhal Uprising subsided, what
of English Education in India? was/were the measure/measures taken by
1. Charter Act of 1813 the colonial government ?
2. General Committee of Public Instruction, 1. The territories called 'Santhal Paraganas'
1823 were created.
3. Orientalist and Anglicist Controversy 2. It became illegal for a Santhal to transfer
Select the correct answer using the code land to a non-Santhal.
given below : Select the correct answer using the code
(a) 1 and 2 only given below :
(b) 2 only (a) 1 only
(c) 1 and 3 only (b) 2 only
(d) 1, 2 and 3 (b) Both 1 and 2
(d) Neither 1 nor 2
2. In 1920, which of the following changed its
name to "Swarajya Sabha" ? 6. Economically, one of the results of the British
(a) All India Home Rule League rule in India in the 19th century was the
(b) Hindu Mahasabha (a) increase in the export of Indian
handicrafts
(c) South Indian Liberal Federation
(b) growth in the number of Indian owned
(d) The Servants of India Society
factories
(c) commercialization of Indian agriculture
3. Which among the following events happened
(d) rapid increase in the urban population
earliest ?
(a) Swami Dayanand established Arya
Samaj. 7. Consider the following events :
(b) Dinabandhu Mitra wrote Neeldarpan.

https://telegram.me/CivilServices_UPSC
https://t.me/UPSC_PDF

HISTORY OF INDIA AND INDIAN NATIONAL MOVEMENT

1. The first democratically elected (b) Active involvement of Dalit and Tribal
communist party government formed in a communities of India in the National
State in India. Movement
2. India's then largest bank, imperial Bank (c) Joining of peasant unrest to India's
of India', was renamed 'State Bank of National Movement
India'. (d) Drastic decrease in the cultivation of
3. Air India was nationalised and became plantation crops and commercial crops
the national carrier.
4. Goa became a part of independent India. 12. Who among the following were the founders
Which of the following is the correct of the "Hind Mazdoor Sabha" established in
chronological sequence of the above events 1948?
? (a) B. Krishna Pillai, E.M.S.
(a) 4-1-2-3 Namboodiripad and K.C. George
(b) 3-2-1-4 (b) Jayaprakash Narayan, Deen Dayal
(c) 4-2-1-3 Upadhyay and M.N. Roy
(d) 3-1-2-4 (c) C.P. Ramaswamy Iyer, K. Kamaraj and
Veeresalingam Pantulu
8. He wrote biographies of Mazzini, Garibaldi, (d) Ashok Mehta, T.S. Ramanujam and G.G.
Shivaji and Shrikrishna; stayed in America for Mehta
some time; and was also elected to the
Central Assembly. He was 13. Which one of the following statements does
(a) Aurobindo Ghosh not apply to the system of Subsidiary Alliance
(b) Bipin Chandra Pal introduced by Lord Wellesley ?
(c) Lala Lajpat Rai (a) To maintain a large standing army at
other's expense
(d) Motilal Nehru
(b) To keep India safe from Napoleonic
danger
9. In the Federation established by The
(c) To secure a fixed income for the
Government of India Act of 1935, residuary
Company
powers were given to the
(d) To establish British paramountcy over the
(a) Federal Legislature
Indian States
(b) Governor General
(c) Provincial Legislature
14. Regarding Wood's Dispatch, which of the
(d) Provincial Governors
following statements are true ?
1. Grants-in-Aid system was introduced.
10. The staple commodities of export by the
2. Establishment of universities was
English East India Company from Bengal in
recommended.
the middle of the 18th century were
3. English as a medium of instruction at all
(a) Raw cotton, oil-seeds and opium
levels of education was recommended.
(b) Sugar, salt, zinc and lead
Select the correct answer using the code
(c) Copper, silver, gold, spices and tea given below:
(d) Cotton, silk, saltpetre and opium (a) 1 and 2 only
(b) 2 and 3 only
11. Which one of the following is a very (c) 1 and 3 only
significant aspect of the Champaran
(d) 1, 2 and 3
Satyagraha ?
(a) Active all-India participation of lawyers,
15. Which one of the following foreign travellers
students and women in the National
elaborately discussed about diamonds and
Movement
diamond mines of India ?

https://telegram.me/CivilServices_UPSC
https://t.me/UPSC_PDF

HISTORY OF INDIA AND INDIAN NATIONAL MOVEMENT

(a) Francois Bernier 2. Tyagaraja created several new ragas.


(b) Jean-Baptiste Tavernier 3. Annamacharya and Tyagaraja are
(c) Jean de Thevenot contemporaries.
(d) Abbe Barthelemy Carre 4. Annamacharya kirtanas are devotional
songs in praise of Lord Venkateshwara.
16. With reference to the religious practices in Which of the statements given above are
India, the "Sthanakvasi" sect belongs to correct?
(a) Buddhism (a) 1 and 3 only
(b) Jainism (b) 2 and 4 only
(c) Vaishnavism (c) 1, 2 and 3
(d) Shaivism (d) 2, 3 and 4

17. With reference to Indian history, who among 21. With reference to the cultural history of India,
the following is a future Buddha, yet to come consider the following statements:
to save the world ? 1. White marble was used in making Buland
(a) Avalokiteshvara Darwaza and Khankah at Fatehpur Sikri.
(b) Lokesvara 2. Red sandstone and marble were used in
making Bara Imambara and Rumi
(c) Maitreya
Darwaza at Lucknow.
(d) Padmapani
Which of the statements given above is/ are
correct ?
18. Consider the following pairs :
(a) 1 only
Craft Heritage of (b) 2 only
1. Puthukkuli - Tamil Nadu (c) Both 1 and 2
shawls (d) Neither 1 nor 2
2. Sujni - Maharashtra
embroidery 22. Consider the following pairs :
3. Uppada - Karnataka Tradition State
Jamdani saris
1 Chapchar Kut : Mizoram
Which of the pairs given above is/are correct festival
?
2 Khongjom Parba : Manipur
(a) 1 only ballad
(b) 1 and 2
3 Thang-Ta dance : Sikkim
(c) 3 only
Which of the pairs given above is/are correct
(d) 2 and 3
?
(a) 1 only
19. The well-known painting "Bani Thani"
(b) 1 and 2
belongs to the
(c) 3 only
(a) Bundi school
(d) 2 and 3
(b) Jaipur school
(c) Kangra school
(d) Kishangarh school 2017

20. With reference to cultural history of India, 1. With reference to the difference between the
consider the following statements : culture of Rigvedic Aryans and Indus Valley
1. Most of the Tyagaraja Kritis are people, which of the following statements
devotional songs in praise of Lord is/are correct ?
Krishna.

https://telegram.me/CivilServices_UPSC
https://t.me/UPSC_PDF

HISTORY OF INDIA AND INDIAN NATIONAL MOVEMENT

1. Rigvedic Aryans used the coat of m ail an (a) Ajanta


d h el met in warfare whereas the people (b) Badami
of Indus Valley Civilization did not leave (c) Bagh
any evidence of using them.
(d) Ellora
2. Rigvedic Aryans knew gold, silver am
copper whereas Indus Valley people
5. Consider the following pairs :
knew only copper and iron.
3. Rigvedic Aryans had domesticated the Traditions Communities
horse whereas there is no evidence of 1. Chaliha Sahib - Sindhis
Indus Valley people having been aware Festival
of this animal.
2. Nanda Raj Jaat - Gonds
Select the correct answer using the code Yatra
given below:
3. Wari-Warkari - Santhals
(a) 1 only
(b) 2 and 3 only Which of the pairs given above is/are
correctly matched ?
(c) 1 and 3 only
(a) 1 only
(d) 1, 2 and 3
(b) 2 and 3 only
2. With reference to Manipuri Sankirtana,
consider the following statements : (c) 1 and 3 only
1. It is a song and dance performance. (d) None of the above
2. Cymbals are the only musical
instruments used in the performance. 6. Which of the following is/are famous for Sun
3. It is performed to narrate the life and temples?
deeds of Lord Krishna. 1. Arasavalli
Which of the statements given above is/ are 2. Amarakantak
correct ? 3. Omkareshwar
(a) 1, 2 and 3 Select the correct answer using the code
(b) 1 and 3 only given below:
(c) 2 and 3 only (a) 1 only
(d) 1 only (b) 2 and 3 only
(c) 1 and 3 only
3. With reference to the religious history of (d) 1, 2 and 3
India, consider the following statements:
1. Sautrantika and Sammitiya were the 7. Which one of the following was a very
sects of Jainism. important seaport in the Kakatiya kingdom?
2. Sarvastivadin held that the constituents (a) Kakinada
of phenomena were not wholly (b) Motupalli
momentary, but existed forever in a latent (c) Machilipatnam (Masulipatnam)
form.
(d) Nelluru
Which of the statements given above is/ are
correct ?
8. Who among the following was/were
(a) 1 only
associated with the introduction of Ryotwari
(b) 2 only Settlement in India during the British rule ?
(c) Both 1 and 2 1. Lord Cornwallis
(d) Neither 1 nor 2 2. Alexander Read
3. Thomas Munro
4. The painting of Bodhisattva Padmapani is Select the correct answer using the code
one of the most famous and oft-illustrated given below:
paintings at
(a) 1 only

https://telegram.me/CivilServices_UPSC
https://t.me/UPSC_PDF

HISTORY OF INDIA AND INDIAN NATIONAL MOVEMENT

(b) 1 and 3 only (a) 1 only


(c) 2 and 3 only (b) 1 and 3 only
(d) 1, 2 and 3 (c) 2 and 3 only
(d) 1, 2 and 3
9. The Trade Disputes Act of 1929 provided for
(a) the participation of workers in the 13. In the context of Indian history, the principle
management of industries. of 'Dyarchy (diarchy)' refers to
(b) arbitrary powers to the management to (a) Division of the central legislature into two
quell industrial disputes. houses.
(c) an intervention by the British Court in the (b) Introduction of double government i.e.,
event of a trade dispute, Central and State governments.
(d) a system of tribunals and a ban on (c) Having two sets of rulers; one in London
strikes. and another in Delhi
(d) Division of the subjects delegated to the
10. Consider the following statements : provinces into two categories.
1. The Factories Act, 1881 was passed with
a view to fix the wages of industrial 14. With reference to" Indian freedom struggle,
workers and to allow the workers to form consider the following events :
trade unions. 1. Mutiny in Royal Indian Navy
2. N.M. Lokhande was a pioneer in 2. Quit India Movement launched
organizing the labour movement in British 3. Second Round Table Conference
India.
What is the correct chronological sequence of
Which of the above statements is/are correct the above events?
?
(a) 1-2-3
(a) 1 only
(b) 2-1-3
(b) 2 only
(c) 3-2-1
(c) Both 1 and 2
(d) 3-1-2
(d) Neither 1 nor 2

11. The object of the Butler Committee of 1927 2016


was to
(a) Define the jurisdiction of the Central and 1. Banjaras during the medieval period of Indian
Provincial Governments. history were generally
(b) Define the powers of the Secretary of (a) agriculturists
State for India. (b) warriors
(c) Impose censorship on national press. (c) weavers
(d) Improve the relationship between the (d) traders
Government of India and the Indian
States.
2. Who of the following had first deciphered the
edicts of Emperor Ashoka?
12. Consider the following pairs :
(a) Georg Biihler
1. Radhakanta Deb — First President of (b) James Prinsep
the' British Indian Association
(c) Max Miiller
2. Gazulu Lakshminarasu Chetty —
(d) William Jones
Founder of the Madras Mahajana Sabha
3. Surendranath Banerjee — Founder of the
Indian Association. 3. The 'Swadeshi' and 'Boycott' were adopted
as methods of struggle for the first time
Which of the above pairs is/are correctly
during the
matched?

https://telegram.me/CivilServices_UPSC
https://t.me/UPSC_PDF

HISTORY OF INDIA AND INDIAN NATIONAL MOVEMENT

(a) agitation against the Partition of Bengal (d) None of the statements given above is
(b) Home Rule Movement correct
(c) Non-Cooperation Movement
(d) visit of the Simon Commission to India 8. Consider the following :
1. Calcutta Unitarian Committee
4. With reference to the religious history of 2. Tabernacle of New Dispensation
India, consider the following statements: 3. Indian Reform Association
1. The concept of Bodhisattva is central to Keshab Chandra Sen is associated with the
Hinayana sect of Buddhism. establishment of which of the above?
2. Bodhisattva is a compassionate one on (a) 1 and 3 only
his way to enlightenment. (b) 2 and 3 only
3. Bodhisattva delays achieving his own (c) 3 only
salvation to help all sentient beings on (d) 1, 2 and 3
their path to it.
Which of the statements given above is/are
9. In the context of the history of India, consider
correct?
the following pairs :
(a) 1 only
Term Description
(b) 2 and 3 only
(c) 2 only 1. Eripatti : Land, revenue from
which was set apart for
(d) 1, 2 and 3
the main-tenance of
5. Satya Shodhak Samaj organized the village tank

(a) a movement for upliftment of tribals in 2. Taniyurs : Villages donated to a


Bihar single Brahmin or a
(b) a temple-entry movement in Gujarat group of Brahmins
(c) an anti-caste movement in 3. Ghatikas : Colleges generally
Maharashtra attached to the
(d) a peasant movement in Punjab temples
Which of the pairs given above is/are
6. The Montague-Chelmsford Proposals were correctly matched?
related to (a) 1 and 2
(a) social reforms (b) 3 only
(b) educational reforms (c) 2 and 3
(c) reforms in police administration (d) 1 and 3
(d) constitutional reforms
10. With reference to the economic history of
7. What is/are common to the two historical medieval India, the term 'Araghatta' refers to
places known as Ajanta and Mahabalipuram? (a) bonded labour
1. Both were built in the same period. (b) land grants made to military officers
2. Both belong to the same religious (c) waterwheel used in the irrigation of land
denomination. (d) wasteland converted to cultivated land
3. Both have rock-cut monuments.
Select the correct answer using the code 11. With reference to the cultural history of India,
given below. the memorizing of chronicles, dynastic
(a) 1 and 2 only histories and epic tales was the profession of
(b) 3 only who of the following?
(c) 1 and 3 only (a) Shramana
(b) Parivraajaka

https://telegram.me/CivilServices_UPSC
https://t.me/UPSC_PDF

HISTORY OF INDIA AND INDIAN NATIONAL MOVEMENT

(c) Agrahaarika Which of the statements given above is/ are


(d) Maagadha correct?
(a) 1 only
12. What was the main reason for the split in the (b) 2 only
Indian National Congress at Surat in 1907? (c) Both 1 and 2
(a) Introduction of communalism into Indian (d) Neither 1 nor 2
politics by Lord Minto
(b) 'Extremists' lack of faith in the capacity of 16. Which one of the following books of ancient
the moderates to negotiate with the India has the love story of the son of the
British Government founder of Sunga dynasty?
(c) Foundation of Muslim League (a) Swapnavasavadatta
(d) Aurobindo Ghosh's inability to be elected (b) Malavikagnimitra
as the President of the Indian National (c) Meghadoota
Congress
(d) Ratnavali
13. The plan of Sir Stafford Cripps envisaged that
after the Second World War 17. With reference to the cultural history of
(a) India should be granted complete medieval India, consider the following
independence statements :
(b) India should be partitioned into two 1. Siddhas (Sittars) of Tamil region were
before granting independence monotheistic and condemned idolatry.
(c) India should be made a republic with the 2. Lingayats of Kannada region questioned
condition that she will join the the theory of rebirth and rejected the
Commonwealth caste hierarchy.
(d) India should be given Dominion status Which of the statements given above is/ are
correct?
(a) 1 only
14. Consider the following pairs :
(b) 2 only
Famous place Region
(c) Both 1 and 2
1. Bodhgaya : Baghelkhand (d) Neither 1 nor 2
2. Khajuraho : Bundelkhand
3. Shirdi : Vidarbha 2015
4. Nasik (Nashik) : Malwa
1. Kalamkari painting refers to
5. Tirupati : Rayalaseema (a) a hand-painted cotton textile in South
Which of the pairs given above are correctly India
matched? (b) a handmade drawing on bamboo
(a) 1, 2 and 4 handicrafts in North-East India
(b) 2, 3, 4 and 5 (c) a block-painted woollen cloth in Western
(c) 2 and 5 only Himalayan region of India
(d) 1, 3, 4 and 5 (d) a hand-painted decorative silk cloth in
North-Western India

15. Regarding the taxation system of Krishna


Deva, the ruler of Vijayanagar, consider the 2. With reference to Rowlat Satyagraha, which
following statements : of the following statements is/are correct ?
1. The tax rate on land was fixed depending 1. The Rowlatt Act was based on the
on the quality of the land. recommendations of the 'Sedition
Committee'.
2. Private owners of workshops paid an
industries tax. 2. In Rowlatt Satyagraha, Gandhiji tried to
Utilize the Home Rule League.

10

https://telegram.me/CivilServices_UPSC
https://t.me/UPSC_PDF

HISTORY OF INDIA AND INDIAN NATIONAL MOVEMENT

3. Demonstrations against the arrival of State


Simon Commission coincided with 1. Champaka : Central India
Rowlatt Satyagraha.
2. Durgara : Jammu
Select the correct answer using the code
given below. 3. Kuluta : Malabar
(a) 1 only Which of the above pairs is/are correctly
(b) 1 and 2 only matched?
(c) 2 and 3 only (a) 1 and 2
(d) 1, 2 and 3 (b) 2 only
(c) 1 and 3
3. Who of the following was/were economic (d) 3 only
critic? critics of colonialism in India?
1. Dadabhai Naoroji 7. Which of the following kingdoms were
2. G. Subramania Iyer associated with the life of the Buddha?
3. R. C. Dutt 1. Avanti
Select the correct answer using the code 2. Gandhara
given below. 3. Kosala
(a) 1 only 4. Magadha
(b) 1 and 2 only Select the correct answer using the code
(c) 2 and 3 only given below.
(d) 1, 2 and 3 (a) 1, 2 and 3
4. With reference to Congress Socialist Party, (b) 2 and 3 only
consider the following statements: (c) 1, 3 and 4
1. It advocated the boycott of British goods (d) 3 and 4 only
and evasion of taxes.
2. It wanted to establish the dictatorship of 8. Consider the following:
proletariat. The arrival of Babur into India led to the
3. It advocated, separate electorate for 1. introduction of gunpowder in the
minorities and oppressed classes. subcontinent
Which of the statements given above is/ are 2. introduction of the arch and dome in the
correct? region's architecture
(a) 1 and 2 only 3. establishment of Timurid dynasty in the
(b) 3 only region
(c) 1, 2 and 3 Select the correct answer using the code
(d) None given below.
(a) 1 and 2 only
5. The Government of India Act of 1919 clearly (b) 3 only
defined (c) 1 and 3 only
(a) the separation of power between the (d) 1, 2 and 3
judiciary and the legislature
(b) the jurisdiction of the central and 9. Who of the following organized a march on
provincial governments the Tanjore coast to break the Salt Law in
(c) the powers of the Secretary of State for April 1930?
India and the Viceroy (a) V. O. Chidambararn Pillai
(d) None of the above (b) C. Rajagopalachari
(c) K. Karnaraj
6. Consider the following pairs : (d) Annie Besant
Medieval Indian Present Region

11

https://telegram.me/CivilServices_UPSC
https://t.me/UPSC_PDF

HISTORY OF INDIA AND INDIAN NATIONAL MOVEMENT

10. Who of the following founded a new city on (2) Emergence of administrative structure
the south bank of a tributaiy to river Krishna based on control and possession of land
and undertook to rule his new kingdom as the (3) Creation of lord-vassal relationship
agent of a deity to whom all the land south of between the feudal lord and his overlord
the river Krishna was supposed to belong? Select the correct answer using the code
(a) Amoghavarsha-I given below.
(b) BallaIa-II (a) 1 and 2 only
(c) Harihara-I (b) 2 and 3 only
(d) Prataparudra-II (c) 3 only
(d) 1, 2 and 3
11. Consider the following statements
1. The first woman President of the Indian 15. Which one of the following movements has
National Congress was Sarojini Naidu. contributed to a split in the Indian National
2. The first Muslim President of the Indian Congress resulting in the emergence of
National Congress was Badruddin Tyabji. 'moderates' and 'extremists'?
Which of the statements given above is/ are (a) Swadeshi Movement
correct? (b) Quit India Movement
(a) 1 only (c) Non-Cooperation Movement
(b) 2 only (d) Civil Disobedience Movement
(c) Both 1 and 2
(d) Neither 1 nor 2
2014
12. With reference to the Cabinet Mission, which
of the following statements is/are correct? 1. In medieval India, the designations
1. It recommended a federal government. 'Mahattara' and Pattakila' were used for
(a) military officers
2. It enlarged the powers of the Indian
courts. (b) village headmen
3. It provided for more Indians in the ICS. (c) specialists in Vedic rituals
Select the correct answer using the code (d) chiefs of craft guilds
given below.
(a) 1 only 2. Which of the following Kingdoms were
(b) 2 and 3 associated with the life of the Buddha?
(c) 1 and 3 1. Avanti
(d) None 2. Gandhara
13. With reference to the cart and archeological 3. Kosala
history of India, which one among the 4. Magadha
following was made earliest? Select the correct answer using the code
(a) Lingaraja Temple at Bhubaneswar given below.
(b) Rock-cut Elephant at Dhauli (a) 1, 2 and 3
(c) Rock-cut Monuments at Mahabalipuram (b) 2 and 4
(d) Varaha Image at Udayagiri (c) 3 and 4 only
(d) 1, 3 and 4
14. With reference to Indian history, which of the
following is/are the essential element/ 3. The Partition of Bengal made by Lord Curzon
elements of the feudal system? in 1905 lasted until
(1) A very strong centralized political (a) the First World War when Indian troops
authority and a very weak provincial or were needed by the British and the
local political authority partition was ended

12

https://telegram.me/CivilServices_UPSC
https://t.me/UPSC_PDF

HISTORY OF INDIA AND INDIAN NATIONAL MOVEMENT

(b) King George V abrogated Curzon's Act at Buddha’s


the Royal Durbar in Delhi in 1911 Mahapannirvana with
numerous celestial
(c) Gandhiji launched his Civil Disobedience
musicians above and the
Movement
sorrowful figures of his
(d) the Partition of India in 1947 when East followers below
Bengal became East Pakistan
2. A huge image of Varaha : Mount Abu
Avatar (boar incarnation)
4. The 1929 Session of Indian National of Vishnu, as he rescues
Congress is of significance in the history of Goddess Earth from the
the Freedom Movement because the deep and chaotic waters,
(a) attainment of Self-Government was sculpted on rock
declared as the objective of the Congress
3. “Arjuna’s Penance”/ : Mamallapuram
(b) attainment of Poorna Swaraj was “Descent of Ganga”
adopted as the goal of the Congress sculpted on the surface
(c) Non-Cooperation Movement was of huge boulders
launched
(d) decision to participate in the Round Table Which of the pairs given above is/are
Conference in London was taken correctly matched?
(a) 1 and 2 only
5. With reference to the famous Sattriya dance, (b) 3 only
consider the following statements (c) 1 and 3 only
1. Sattriya is a combination of music, dance (d) 1, 2 and 3
and drama.
2. It is a centuries-old living tradition of 8. The Ghadr (Ghadar) was a
Vaishnavites of Assam. (a) revolutionary association of Indians with
3. It is based on classical Ragas and Talas headquarters at San Francisco
of devotional songs composed by (b) nationalist organization operating from
Tulsidas, Kabir and Mirabai. Singapore
Which of the statements given above is/ are (c) militant organization with headquarters at
correct? Berlin
(a) 1 only (d) communist movement for India's freedom
(b) 1 and 2 only with headquarters at Tashkent
(c) 2 and 3 only
(d) 1, 2 and 3 9. With reference to India's culture and tradition,
what is 'Kalaripayattu?
6. Chaitra 1 of the national calendar based on (a) It is an ancient Bhakti cult of Shaivism
the Saka Era corresponds to which one of the still prevalent in some parts of South
following dates of the Gregorian calendar in a India
normal year of 365 days? (b) It is an ancient style bronze and
(a) 22nd March (or 21st March) brasswork still found in southern part of
(b) 15th May (or 16th May) Coromandel area
(c) 31st March (or 30th March) (c) It is an ancient form of dance-drama and
a living tradition in the northern part of
(d) 21st April (or 20th April)
Malabar
(d) It is an ancient martial art and a living
7. With reference to the Indian history of art and
tradition in some parts of South India
culture, consider the following pairs:
Famous work of sculpture Site
10. Consider the following pairs
1. A grand image of : Ajanta 1. Garba : Gujarat

13

https://telegram.me/CivilServices_UPSC
https://t.me/UPSC_PDF

HISTORY OF INDIA AND INDIAN NATIONAL MOVEMENT

2. Mohiniattam : Odisha 1. To disclaim any intention to annex Indian


States
3. Yakshagana : Karnataka
2. To place the Indian administration under
Which of the pairs given above is/are the British Crown
correctly matched?
3. To regulate East India Company's trade
(a) 1 only with India
(b) 2 and 3 only Select the correct answer using the code
(c) 1 and 3 only given below.
(d) 1, 2 and 3 (a) 1 and 2 only
(b) 2 only
11. With reference to Buddhist history, tradition (c) 1 and 3 only
and culture in India, consider the following (d) 1, 2 and 3
pairs :
Famous shrine Location
15. Ibadat Khana at Fatehpur Sikri was
1. Tabo monastery : Spiti Valley
and temple (a) the mosque for the use of Royal Family
complex (b) Akbar's private prayer chamber
2. Lhotsava : Zanskar Valley (c) the hall in which Akbar held discussions
Lhakhang
with scholars of various religions
temple Nako
3. Alchi temple : Ladakh (d) the room in which the nobles belonging
Complex to different religions gathered to discuss
Which of the pairs given above is/are religious affairs
correctly matched?
(a) 1 only 16. Consider the following towns of India:
(b) 2 and 3 only 1. Bhadrachalam
(c) 1 and 3 only 2. Chanderi
(d) 1, 2 and 3 3. Kancheepuram
4. Karnal
12. Consider the following statements Which of the above are famous for the
1. 'Bijak' is a composition of the teachings of production of traditional sarees/ fabric?
Saint Dadu Dayal. (a) 1 and 2 only
2. The Philosophy of Pushti Marg was (b) 2 and 3 only
propounded by Madhvacharya. (c) 1, 2 and 3
Which of the statements given above is/ are (d) 1, 3 and 4
correct?
(a) 1 only 17. The Radcliffe Committee was appointed to
(b) 2 only (a) solve the problem of minorities in India
(c) Both 1 and 2 (b) give effect to the Independence Bill
(d) Neither 1 nor 2 (c) delimit the boundaries between India and
Pakistan
13. A community of people called Manganiyars (d) enquire into the riots in East Bengal
is well-known for their
(a) martial arts in North-East India 18. With reference to the cultural history of India,
(b) musical tradition in North-West India the term 'Panchayatan' refers to
(c) classical vocal music in South India (a) an assembly of village elders
(d) pietra dura tradition in Central India (b) a religious sect
(c) a style of temple construction
14. What was/were the object/objects of Queen (d) an administrative functionary
Victoria's Proclamation (1858) ?

14

https://telegram.me/CivilServices_UPSC
https://t.me/UPSC_PDF

HISTORY OF INDIA AND INDIAN NATIONAL MOVEMENT

19. Which one of the following pairs does not


form part of the six systems of Indian 5. Which of the following characterizes/
Philosophy? characterize the people of Indus Civilization?
(a) Mimamsa and Vedanta 1. They possessed great palaces and
(b) Nyaya and Vaisheshika temples.
(c) Lokayata and Kapalika 2. They worshipped both male and female
(d) Sankhya and Yoga deities.
3. They employed horse-drawn chariots in
warfare.
2013 Select the correct statement/statements
1. Some Buddhist rock-cut caves are called using the codes given below.
Chaityas, while the others are called Viharas. (a) 1 and 2 only
What is the difference between the two? (b) 2 only
(a) Vihara is a place of worship, while (c) 1,2 and 3
Chaitya is the dwelling place of the
(d) None of the statements given above is
monks
correct
(b) Chaitya is a place of worship, while
Vihara is the dwelling place of the monks
6. Which of the following statements is/are
(c) Chaitya is the stupa at the far end of the
applicable to fain doctrine?
cave, while Vihara is the hall axial to it
1. The surest way of annihilating Karma is
(d) There is no material difference between
to practice penance.
the two
2. Every object, even the smallest particle
has a soul.
2. Which one of the following describes best the
3. Karma is the bane of the soul and must
concept of Nirvana in Buddhism?
be ended.
(a) The extinction of the flame of
Select the correct answer using the codes
(b) The complete annihilation of self given below.
(c) A state of bliss and rest (a) 1 only
(d) A mental stage beyond all (b) 2 and 3 only
comprehension desire
(c) 1 and 3 only
(d) 1, 2 and 3
3. The people of India agitated against the
arrival of Simon Commission because
7. With reference to the history of Indian rock-
(a) Indians never wanted the review of the
cut, architecture, consider the following
working of the Act of 1919
statements :
(b) Simon Commission recommended the
1. The caves at Badami are the 'oldest
abolition of Dyarchy (Diarchy) in the
surviving rock-cut caves in India.
Provinces
2. The Barabar rock-cut caves were
(c) there was no indian member in the Simon
originally made for Ajivikas by Emperor
Commission
Chandragupta Maurya.
(d) the Simon Commission suggested the
3. At Ellora, eaves were made for different
partition of the country
faiths.
Which of the statements given above is/ are
4. Quit India Movement was launched in correct?
response to
(a) 1 only
(a) Cabinet Mission Plan
(b) 2 and 3 only
(b) Cripps Proposals
(c) 3 only
(c) Simon Commission Report
(d) 1, 2 and 3
(d) Waveil Plan

15

https://telegram.me/CivilServices_UPSC
https://t.me/UPSC_PDF

HISTORY OF INDIA AND INDIAN NATIONAL MOVEMENT

8. The Chinese traveller Yuan Chwang (Hiuen (a) One leg is bent and the body is slightly
Tsang) who visited india recorded the general but oppositely curved at waist and neck
conditions and culture of India at that time, In (b) Facial expressions, hand gestures and
this context, which of the following make-up are combined to symbolize
statements is/are correct? certain epic or historic characters
1. The roads and river-routes were (c) Movements of body, face and hands are
completely immune from robbery. used to express oneself or to tell a story
2. As regards punishment for offencea, (d) A little smile, slightly curved waist and
ordeals by fire, water and poison were certain hand gestures are emphasized to
the instruments for determining the express the feelhgs of love or eroticism
innocence or guilt of a person.
3. The tradesmen had to pay duties at 12. Annie Besant was
ferries and barrier stations.
1. responsible for starting the Home Rule
Select the correct answer using the codes Movement
given below.
2. the founder of the Theosophical Society
(a) 1 only
3. once the President of the Indian National
(b) 2 and 3 only Congress
(c) 1 and 3 only Select the correct statement / statements
(d) 1, 2 and 3 using the codes given below.
(a) 1 only
9. With reference to Indian History, the (b) 2 and 3 only
Members of the Constituent Assembly from (c) 1 and 3 only
the Provinces were
(d) 1, 2 and 3
(a) directly elected by the people of those
Provinces
13. The Ilbert Bill controversy was related to the
(b) nominated by the Indian National
(a) imposition of certain restrictions to carry
Congress and the Muslim League
arms by the Indians
(c) elected by the Provincial Legislative
(b) imposition of restrictions en newspapers
Assemblies
and magazines published in Indian
(d) selected by the Government for their
languages
expertise in constitutional matters
(c) removal of disqualifications imposed on
the Indian magistrates with regard to the
10. The demand for the Tebhaga Peasant trial of the Europeans
Movement in Bengal was for
(d) removal of a duty on imported cotton
(a) the reduction of the share of the cloth
landlords from one-half of the crop to
one-third
14. Consider the following Bhakti Saints:
(b) the grant of ownership of land to
1. Dadu Dayal
peasants as they were the actual
cultivators of the land 2. Guru Nanak
(c) the uprooting of Zamindsri system and 3. Tyagaraja
the end of serfdom Who among the above was/were preaching when
(d) writing off all peasant debts the Lodi dynasty fell and Babur took over?
(a) 1 and 3
11. Its the context of cultural history: of India, a (b) 2 only
pose in dance and dramatics called (c) 2 and 3
tribhanga' has been a favourite of Indian
(d) 1 and 2
artists from ancient times till today. Which
one of the following statements best
describes this pose? 15. Consider the following historical places:

16

https://telegram.me/CivilServices_UPSC
https://t.me/UPSC_PDF

HISTORY OF INDIA AND INDIAN NATIONAL MOVEMENT

1. Ajanta Caves 1. Dhrupad originated and developed in the


2. Lepakshi Temple Rajput kingdoms during the Mughal
3. Sanchi Stupa period.
Which of the above places is/are also known 2. Dhrupad is primarily a devotional and
for mural paintings? spiritual music.
(a) 1 only 3. Dhrupad Alap uses Sanskrit syllables
from Mantras.
(b) 1 and 2 only
Select the correct answer using the codes
(c) 1, 2 and 3
given below:
(d) None
(a) 1 and 2 only
(b) 2 and 3 only
16. With reference to the history of philosophical
(c) 1, 2 and 3
thought in India, consider the following
statements regarding Sankhya school: (d) None of the above is correct
1. Sankhya does not accept the theory of
rebirth or transmigration of soul. 3. How do you distinguish between Kuchipudi
2. Sankhya holds that it is the self and Bharatanatyam dances?
knowledge that leads to liberation and 1. Dancers occasionally speaking dialogues
not any exterior influence or agent. is found in Kuchipudi dance but not in
Which of the statements given above is/ are Bharatanatyam.
correct? 2. Dancing on the brass plate by keeping
(a) 1 only the feet on its edges is a feature of
Bharatanatyam but Kuchipudi dance
(b) 2 only
does not have such a form of
(c) Both 1 and 2
movements.
(d) Neither 1 nor 2
Which of the statements given above is/ are
correct?
2012 (a) 1 only
(b) 2 only
(c) Both 1 and 2
1. Consider the following statements :
(d) Neither 1 nor 2
The most effective contribution made by
Dadabhai Naoroji to the cause of Indian
National Movement was that he 4. With reference to the religious history of
1. exposed the economic exploitation of medieval India, the Sufi mystics were known
India by the British to pursue which of the following practices?
2. interpreted the ancient Indian texts and 1. Meditation and control of breath
restored the self-confidence of Indians 2. Severe ascetic exercises in a lonely
3. stressed the need for eradication of all place
the social evils before anything else 3. Recitation of holy songs to arouse a state
Which of the statements given above is/ are of ecstasy in their audience
correct? Select the correct answer using the codes
(a) 1 only given below :
(b) 2 and 3 only (a) 1 and 2 only
(c) 1 and 3 only (b) 2 and 3 Only
(d) 1, 2 and 3 (c) 3 only
(d) 1, 2 and 3
2. With reference to Dhrupad, one of the major
traditions of India that has been kept alive for 5. The Rowlatt Act aimed at :
centuries, which of the following statements (a) compulsory economic support to war
are correct? efforts

17

https://telegram.me/CivilServices_UPSC
https://t.me/UPSC_PDF

HISTORY OF INDIA AND INDIAN NATIONAL MOVEMENT

(b) imprisonment without trial and summary 2. It denied the need for a priestly class for
procedures for trial interpreting the religious texts.
(c) suppression of the Khilafat Movement 3. It popularized the doctrine that the Vedas
(d) imposition of restrictions freedom of the are infallible,
press Select the correct answer using the codes
given below :
6. The Lahore Session of the Indian National (a) 1 only
Congress (1929) is very important in history, (b) 1 and 2 only
because (c) 3 only
1. the Congress passed a resolution (d) 1, 2 and 3
demanding complete independence
2. the rift between the extremists and 10. Consider the following
moderates was resolved in that Session
1. Assessment of land revenue on the basis
3. a resolution was passed rejecting the of nature of the soil and the quality of
two-nation theory in that Session crops
Which of the statements given above is/ are 2. Use of mobile cannons in warfare
correct?
3. Cultivation of tobacco and red chillies.
(a) 1 only
Which of the above was/were introduced into
(b) 2 and 3 India by the English?
(c) 1 and 3 (a) 1 only
(d) None of the above (b) 1 and 2
(c) 2 and 3
7. Lord Buddha's image is sometimes shown (d) None
with the hand gesture called 'Bhumisparsha
Mudra'. It symbolizes
11. With reference to the guilds (Shreni) of
(a) Buddha's calling of the Earth to watch
ancient India that played a very important role
over Mara and to prevent Mara from
in the country's economy, which of the
disturbing his meditation
following statements is/are correct?
(b) Buddha's calling of the Earth to witness
1. Every guild was registered with the
his purity and chastity despite the
central authority of the State and the king
temptations of Mara
was the chief administrative authority on
(c) Buddha's reminder to his followers that them.
they all arise from the Earth and finally
2. The wages, rules of work, standards and
dissolve into the Earth, and thus this life
prices were fixed by the guild.
is transitory
3. The guild had judicial powers over its
(d) Both the statements (a) and (b) are
own members.
Correct in this context
Select the correct answer using the codes
given below:
8. The religion of early Vedic Aryans was
(a) 1 and 2 only
primarily of :
(b) 3 only
(a) Bhakti
(c) 2 and 3 only
(b) image worship and Yajnas
(d) 1, 2 and 3
(c) worship of nature and Yajnas
(d) worship of nature and Bhakti
12. The distribution of powers between the
Centre and the States in the Indian
9. Which of the following statements is/are Constitution is based on the scheme provided
correct regarding Brahmo Samaj? in the
1. It opposed idolatry. (a) Morley-Minto Reforms, 1909
(b) Montagu-Chelmsford Act, 1919

18

https://telegram.me/CivilServices_UPSC
https://t.me/UPSC_PDF

HISTORY OF INDIA AND INDIAN NATIONAL MOVEMENT

(c) Government of India Act, 1935 16. The Congress ministries resigned in the
(d) Indian Independence Act, 1947 seven provinces in 1939, because
(a) the Congress could not form ministries in
13. With reference to the scientific progress of the other four provinces
ancient India, which of the statements given (b) emergence of a 'left wing' in the
below are correct? Congress made the working of the
1. Different kinds of specialized surgical ministries impossible
instruments were in common use by 1st (c) there were widespread communal
century AD. disturbances in their provinces
2. Transplant of internal organs in the (d) None of the statements (a), (b) and (c)
human body had begun by the beginning given above is correct
of 3rd century AD.
3. The concept of sine of an angle was 17. Which of the following is/are the principal
known in 5th century AD. feature(s) of the Government of India Act,
4. The concept of cyclic quadrilaterals was 1919 ?
known in 7th century AD. 1. Introduction of dyarchy in the executive
Select the correct answer using the codes government of the provinces
given below : 2. Introduction of separate communal
(a) 1 and 2 only electorates for Muslims
(b) 3 and 4 only 3. Devolution of Legislative authority by the
(c) 1, 3 and 4 only centre to the provinces
(d) 1, 2, 3 and 4 Select the correct answer using the codes
given below :
(a) 1 only
14. With reference to the history of ancient India,
which of the following was/were common to (b) 2 and 3 only
both Buddhism and Jainism? (c) 1 and 3 only
1. Avoidance of extremities of penance and (d) 1, 2 and 3
enjoyment
2. Indifference to the authority of the Vedas 18. During Indian freedom struggle, the National
3. Denial of efficacy of rituals Social Conference was formed. What was the
Select the correct answer using the codes reason for its formation?
given below: (a) Different social reform groups or
(a) 1 only organizations of Bengal region united to
form a single body to discuss the issues
(b) 2 and 3 only
of larger interest and to prepare
(c) 1 and 3 only
appropriate petitions/ representations to
(d) 1, 2 and 3 the government
(b) Indian National Congress did not want to
15. The Nagara, the Dravida and the Vesara are include social reforms in its deliberations
the and decided to form a separate body for
(a) three main racial groups of the Indian such a purpose
subcontinent (c) Behramji Malabari and M. G. Ranade
(b) three main linguistic divisions into which decided to bring together all the social
the languages of India can be classified reform groups of the country under one
(c) three main styles of Indian temple organization
architecture (d) None of the statements (a), (b) and (c)
(d) three main musical Gharanas prevalent given above is correct in this context
in India
19. Which of the following parties were
established by Dr. B. R. Ambedkar?

19

https://telegram.me/CivilServices_UPSC
https://t.me/UPSC_PDF

HISTORY OF INDIA AND INDIAN NATIONAL MOVEMENT

1. The Peasants and Workers Party of India (d) None of the (a), (b) and (c) above.
2. All India Scheduled Castes Federation
3. The Independent Labour Party 2. Which one of the following observations is
Select the correct answer using the codes not true about the Quit India Movement of
given below : 1942 ?
(a) 1 and 2 only (a) It was a non-violent movement.
(b) 2 and 3 only (b) It was led by Mahatma Gandhi
(c) 1 and 3 only (c) It was a spontaneous movement
(d) 1, 2 and 3 (d) It did not attract the labour class in
general.
20. Mahatma Gandhi undertook fast unto death
in 1932, mainly because 3. Which amongst the following provided a
(a) Round Table Conference failed to satisfy common factor for tribal insurrection in India
Indian political aspirations in the 19th century ?
(b) Congress and Muslim League had (a) Introduction of a new system of land
differences of opinion revenue and taxation of tribal products
(c) Ramsay Macdonald announced the (b) Influence of foreign religious missionaries
Communal Award in tribal areas
(d) None of the statements (a), (b) and (c) (c) Rise of a large number of money lenders,
given above is correct in this context traders and revenue farmers as
middlemen in tribal areas
(d) The complete disruption of the old
21. With reference to Ryotwari Settlement,
agrarian order to the tribal communities
consider the following statements :
1. The rent was paid directly by the
peasants to the Government. 4. India maintained its early cultural contacts
and trade links with Southeast Asia across
2. The Government gave Pattas to the
the Bay of Bengal. For this pre-eminence of
Ryots.
early maritime history of Bay of Bengal, which
3. The lands were surveyed and assessed
of the following could be the most convincing
before being taxed.
explanation/ explanations ?
Which of the statements given above is/ are
(a) As compared to other countries, India
correct?
had a better ship-building technology in
(a) 1 only ancient and medieval times.
(b) 1 and 2 only (b) The rulers of southern India always
(c) 1, 2 and 3 patronized traders, brahmin priests and
(d) None buddhist monks in this context.
(c) Monsoon winds across the Bay of Bengal
facilitated sea voyages.
2011
(d) Both (a) and (b) are convincing
explanations in this context.
1. The tendency for increased litigation was
visible after the introduction of the land 5. With reference to the period of colonial rule in
settlement system of Lord Cornwallis in 1793. India, "Home Charges" formed an important
The reason for this is normally traced to part of drain of wealth from India. Which of
which of the following provisions? the following funds constituted "Home
(a) Making Zamindar's position stronger vis- Charges"?
a-vis the ryot. 1. Funds used to support the India Office in
(b) Making East India Company an overlord London.
of Zamindars. 2. Funds used to pay salaries and pensions
(c) Making judicial system more efficient. of British personnel engaged in India.

20

https://telegram.me/CivilServices_UPSC
https://t.me/UPSC_PDF

HISTORY OF INDIA AND INDIAN NATIONAL MOVEMENT

3. Funds used for waging wars outside (d) All the statements (a), (b) and (c) are
India by the British. correct in this context
Select the correct answer using the codes
given below: 9. With reference to Indian freedom struggle,
(a) 1 only Usha Mehta is well-known for
(b) 1 and 2 only (a) Running the secret Congress Radio in
(c) 2 and 3 only the wake of Quit India Movement
(d) 1, 2 and 3 (b) Participating in the Second Round Table
Conference
6. What was the reason for Mahatma Gandhi to (c) Leading a contigent of Indian National
organize a satyagraha on behalf of the Army
peasants of Kheda ? (d) Assisting in the formation of Interim
1. The Administration did not suspend the Government under Pandit Jawahalal
land revenue collection in spite of a Nehru
drought.
2. The Administration proposed to introduce 10. With reference to the period of India freedom
Permanent Settlement in Gujarat. struggle, which of the following was/were
Which of the statements given above is/ are recommended by the Nehru report?
correct? 1. Complete Independence for India.
(a) 1 only 2. Joint electorates for reservation of seats
(b) 2 only for minorities.
(c) Both 1 and 2 3. Provision of fundamental rights for the
people of India in the Constitution.
(d) Neither 1 nor 2
Select the correct answer using the codes
given below :
7. What was the purpose with which Sir William
(a) 1 only
Wedderburn and W.S. Caine had set up the
Indian Parliamentary Committee in 1893 ? (b) 2 and 3 only
(a) To agitate for Indian political reforms in (c) 1 and 3 only
the House of Commons (d) 1, 2 and 3
(b) To campaign for the entry of Indians into
the Imperial Judiciary 11. The Jain philosophy holds that the world is
(c) To facilitate a discussion on India's created and maintained by
Independence in the British Parliament (a) Universal Law
(d) To agitate for the entry of eminent (b) Universal Truth
Indians into the British Parliament (c) Universal Faith
(d) Universal Soul
8. Mahatma Gandhi said that some of his
deepest convictions were reflected in a book 12. Regarding the Indus Valley Civilization,
titled, "Unto this Last" and the book consider the following statements:
transformed his life. What was the message
1. It was predominantly a secular civilization
from the book that transformed Mahatma
and the religious element, though
Gandhi?
present, did not dominate the scene.
(a) Uplifting the oppressed and poor is the
2. During this period, cotton was used for
moral responsibility of an educated man
manufacturing textiles in India.
(b) The good of individual is contained in the
Which of the statements given above is/ are
good of all
correct?
(c) The life of celibacy and spiritual pursuit
(a) 1 only
are essential for a noble life
(b) 2 only
(c) Both 1 and 2

21

https://telegram.me/CivilServices_UPSC
https://t.me/UPSC_PDF

HISTORY OF INDIA AND INDIAN NATIONAL MOVEMENT

(d) Neither 1 nor 2 "The Way Out". Which one of the following
was a proposal in this : pamphlet?
13. The "dharma" and "rita" depict a central idea (a) The establishment of a "War Advisory
of ancient Vedic civilization of India. In this Council" composed of representatives of
context, consider the following statements: British India and the Indian States
1. Dharma was a conception of obligations (b) Reconstitution of the Central Executive
and of the discharge of one's duties to
Council in such a way that all its
oneself and to others
members, except the Governor General
2. Rita was the fundamental moral law
and the Commander-in-Chief should be
governing the functioning of the universe
and all it contained. Indian leaders
Which of the statements given above is/ are (c) Fresh elections to the Central and
correct? Provincial Legislatures to be held at the
(a) 1 only end of 1945 and the Constitution making
(b) 2 only body to be convened as soon as possible
(c) Both 1 and 2 (d) A solution for the constitutional deadlock
(d) Neither 1 nor 2
4. There are only two known examples of cave
2010 paintings of the Gupta period in ancient India.
One of these is paintings of Ajanta caves.
Where is the other surviving example of
1. With reference to Simon Commission's Gupta paintings ?
recommendations, which one of the following
(a) Bagh caves
statements is correct?
(b) Ellora caves
(a) It recommended the replacement of
(c) Lomas Rishi cave
diarchy with responsible government in
the provinces (d) Nasik caves
(b) It proposed the setting up of inter-
provincial council under the Home 5. Who among the following Governor Generals
Department created the Covenanted Civil Service of India
(c) It suggested the abolition of bicameral which later came to be known as the Indian
legislature at the Centre Civil Service ?
(d) It recommended the creation of Indian (a) Warren Hastings
Police Service with a provision for (b) Wellesley
increased pay and allowances for British (c) Cornwallis
recruits as compared to Indian recruits (d) William Bentinck

2. Four resolutions were passed at the famous 6. What was the immediate cause for the launch
Calcutta session of Indian National Congress of the Swadeshi movement ?
in 1906. The question of either retention OR (a) The partition of Bengal done by Lord
of rejection of these four resolutions became Curzon
the cause of a split in Congress at the next
(b) A sentence of 18 months rigorous
Congress session held in Surat in 1907.
imprisonment imposed on Lokmanya
Which one of the following was not one of
Tilak
those resolutions?
(c) The arrest and deportation of Lala Lajpat
(a) Annulment of partition of Bengal
Rai and Ajit Singh; and passing of the
(b) Boycott Punjab Colonization Bill
(c) National education (d) Death sentence pronounced on the
(d) Swadeshi Chapekar brothers

3. After Quit India Movement, C. 7. Consider the following statements :


Rajagopalachari issued a pamphlet entitled

22

https://telegram.me/CivilServices_UPSC
https://t.me/UPSC_PDF

https://telegram.me/CivilServices_UPSC
https://t.me/UPSC_PDF

https://telegram.me/CivilServices_UPSC
https://t.me/UPSC_PDF

ANSWERS - HISTORY OF INDIA AND INDIAN NATIONAL MOVEMENT

ANSWERS OF HISTORY OF INDIA AND INDIAN


NATIONAL MOVEMENT

2019 13. (c) To secure a fixed income for the


Company
14. (a) 1 and 2 only
1. (c) Sohgaura
15. (b) Jean-Baptiste Tavernier
2. (a) Kanganahalli
16. (b) Jainism
3. (d) 1, 2 and 3
17. (c) Maitreya
4. (a) It was considered a source of income
for the State, a sort of tax paid by the 18. (a) 1 only
people. 19. (d) Kishangarh school
5. (d) Vijayanagara 20. (b) 2 and 4 only
6. (a) 1 only 21. (a) 1 only
7. (d) Neither 1 nor 2 22. (b) 1 and 2
8. (a) Tansen was the title given to him by
Emperor Akbar.
9. (c) Jahangir 2017
10. (d) Neither 1 nor 2
11. (a) 1 and 2 only 1. (a) 1 only
12. (b) 1 and 3 only 2. (b) 1 and 3 only
13. (d) 1, 2 and 3 3. (b) 2 only
14. (b) The major aim of land reforms was 4. (a) Ajanta
providing agricultural land to all the 5. (a) 1 only
landless. 6. (a) 1 only
15. (a) 1 and 2 only 7. (c) Machilipatnam (Masulipatnam)
16. (c) Both 1 and 2 8. (c) 2 and 3 only
17. (d) 2 and 3 9. (d) a system of tribunals and a ban on
strikes.
2018 10. (b) 2 only
11. (d) Improve the relationship between the
1. (d) 1, 2 and 3 Government of India and the Indian
2. (a) All India Home Rule League States.
3. (b) Dinabandhu Mitra wrote Neeldarpan. 12. (b) 1 and 3 only
4. (b) 2 only 13. (d) Division of the subjects delegated to
the provinces into two categories.
5. (c) Both 1 and 2
14. (c) 3-2-1
6. (c) commercialization of Indian
agriculture
7. (b) 3-2-1-4 2016
8. (c) Lala Lajpat Rai
9. (b) Governor General 1. (d) traders
10. (d) Cotton, silk, saltpetre and opium 2. (b) James Prinsep
11. (c) Joining of peasant unrest to India’s 3. (a) agitation against the Partition of
National Movement Bengal
12. (d) Ashok Mehta, T.S. Ramanujam and 4. (b) 2 and 3 only
G.G. Mehta 5. (c) an anti-caste movement in
Maharashtra

25

https://telegram.me/CivilServices_UPSC
https://t.me/UPSC_PDF

ANSWERS - HISTORY OF INDIA AND INDIAN NATIONAL MOVEMENT

6. (d) constitutional reforms 6 (a) 22nd March (or 21st March)


7. (b) 3 only 7 (c) 1 and 3 only
8. (b) 2 and 3 only 8 (a) revolutionary association of Indians
9. (d) 1 and 3 with headquarters at San Francisco
10. (c) waterwheel used in the irrigation of 9 (d) It is an ancient martial art and a living
land tradition in some parts of South India
11. (d) Maagadha 10 (c) 1 and 3 only
12. (b) ‘Extremists’ lack of faith in the 11 (c) 1 and 3 only
capacity of the moderates to 12 (d) Neither 1 nor 2
negotiate with the British 13 (b) musical tradition in North-West India
Government 14 (a) 1 and 2 only
13. (d) India should be given Dominion 15 (c) the hall in which Akbar held
status discussions with scholars of various
14. (c) 2 and 5 only religions
15. (c) Both 1 and 2 16 (b) 2 and 3 only
16. (b) Malavikagnimitra 17 (c) delimit the boundaries between India
17. (b) 2 only and Pakistan
18 (c) a style of temple construction
2015 19 (c) Lokayata and Kapalika

1. (a) a hand-painted cotton textile in South 2013


India
2. (b) 1 and 2 only 1 (b) Chaitya is a place of worship, while
3. (d) 1, 2 and 3 Vihara is the dwelling place of the
4. (a) 1 and 2 only monks
5. (b) the jurisdiction of the central and 2 (c) A state of bliss and rest
provincial governments 3 (c) there was no indian member in the
Simon Commission
6. (b) 2 only
4 (b) Cripps Proposals
7. (d) 3 and 4 only
5 (b) 2 only
8. (c) 1 and 3 only
6 (d) 1, 2 and 3
9. (b) C. Rajagopalachari
7 (c) 3 only
10. (c) Harihara-I
8 (b) 2 and 3 only
11. (b) 2 only 9 (c) elected by the Provincial Legislative
12. (d) None Assemblies
13. (b) Rock-cut Elephant at Dhauli 10 (a) the reduction of the share of the
14. (b) 2 and 3 only landlords from one-half of the crop to
15. (a) Swadeshi Movement one-third
11 (a) One leg is bent and the body is
slightly but oppositely curved at waist
2014 and neck
12 (c) 1 and 3 only
1 (b) village headmen 13 (c) removal of disqualifications imposed
2 (c) 3 and 4 only on the Indian magistrates with regard
to the trial of the Europeans
3 (b) King George V abrogated Curzon’s
Act at the Royal Durbar in Delhi in 14 (b) 2 only
1911 15 (b) 1 and 2 only
4 (b) attainment of Poorna Swaraj was 16 (c) Both 1 and 2
adopted as the goal of the Congress
5 (b) 1 and 2 only

26

https://telegram.me/CivilServices_UPSC
https://t.me/UPSC_PDF

ANSWERS - HISTORY OF INDIA AND INDIAN NATIONAL MOVEMENT

2012 10 (d) None


1 (a) 1 only All the above mentioned items were
popular in India even before the
Dadabhai Naoroji is also known as the advent of the British. Use of mobile
“First Economic Thinker of India”. In canons in warfare was introduced by
his book “Poverty and Un-British Rule Babur, and cultivation of tobacco and
in India” he has described elaborately red chillies was started by the
the drain of wealth from India. Portuguese in India.
2 (b) 2 and 3 only 11 (c) Statement 2 and 3 are correct.
Dhrupad is the oldest surviving form of
Indian classical music and traces its Kautilya mentions in Arthashastea
origin to the chanting of the Vedic regarding the methods devised for the
hymns and Mantras during the Vedic regulation of the guilds. But the king
age. Most Dhrupads are religious in was not the chief administrative
nature, praising Hindu gods, although authority.
some texts praise the kings also. 12 (c) Government of India Act, 1935.
3 (a) 1 only 13 (c) 1, 3 and 4 only
Dancing on the brass plate by keeping Sushruta, who was the contemporary
the feet on its edges is a feature of of Kanishka, wrote a book “Sushruta
Kuchipudi dance. Samhita”, which was about surgical
4 (d) 1, 2 and 3 instruments and procedures.
The Sufis propagated mystic union of 14 (b) 2 and 3 only
individual with the God. The Chisti Jainism believed in extremism
Sufis had many things in common with 15 (c) three main styles of Indian temple
the Hindu Yogis. architecture.
5 (b) imprisonment without trial and These are the three styles of temple
summary procedures for trial. architecture. The Nagara style was
The Rowlatt Act came in March 1919. prevalent in North India, e.g.,
According to the Act, an Indian could Kandarya Mahadev Temple of
be imprisoned if he was in possession Khajuraho. The Dravida style was in
of arms without license. He could not South India, e.g. Brihadeshwar temple
defend himself in a law-court. at Tanjore.
6 (a) 1 only 16 (d) None of the statements (a), (b) and (c)
The Lahore Session was presided given above is correct
over by J.L. Nehru, which, for the first The Congress ministries resigned in
time, passed a resolution demanding 1939 because the British government
complete independence. declared India a party to the World
7 (b) Buddha’s calling of the Earth to War – II without consulting its leaders.
witness his purity and chastity despite 17 (c) 1 and 3 only
the temptations of Mara. Dyarchy was introduced by the
Among Buddhist symbols, Gautam Government of India Act, 1919 in
Buddha used the Bhumisparsha provinces whereby subjects were
gesture to summon the Earth Godess, classified into two categories :
Sthavara. This gesture signified that (i) Reserved
Buddha had withstood all the (ii) Transferred
temptations put before him by Mara, 18 (d) None of the statements (a), (b) and (c)
the God of Evil. given above is correct in this context
8 (c) worship of nature and Yajnas M.G. Ranade was the founder of the
9 (b) 1 and 2 statements are correct. National Social Conference. It was not
Brahmo Samaj laid emphasis on formed by the Congress.
monotheism and opposed idolatory as 19 (b) 2 and 3 only
well as criticized Brahmins or priestly B.R. Ambedkar established the All
class. India Scheduled Castes Federation in

27

https://telegram.me/CivilServices_UPSC
https://t.me/UPSC_PDF

ANSWERS - HISTORY OF INDIA AND INDIAN NATIONAL MOVEMENT

1946 and the Independent Labour 4. (d) Both (a) and (b) are convincing
Party in 1936 (The Peasants and explanations in this context.
Workers Party of India was a Marxist Indian shipping industry was well
political party in Maharashtra. The established both in ancient and
party was founded in 1949, having its medieval times. The priests and
roots from the pre-independence monks who visited to South-East
period and had around 10,000 Asia to preach, later on settled there.
members). 5. (d) 1, 2 and 3
20 (c) Ramsay Macdonald announced the All wars waged in defence of India
Communal Award were put under Home Charges.
The British Prime Minister Ramsay Salaries and expenses to Indian
Macdonald, on 16th Aug. 1932, offices were, of course, put under.
announced the creation of separate ‘Home Charges’ to be paid by
electorates for the Depressed Indians.
Classes. Gandhiji began a fast unto 6. (a) 1 only
death on this issue of separate
In 1918, due to a severe drought in
electorate on 20th Sept., and was able
Kheda district of Gujarat, the
to secure an agreement with B.R.
peasants were not able to pay the
Ambedkar (Poona Past) by which the
normal land revenue. Hence, they
Award was modified.
wanted a remission or suspension of
21 (c) 1, 2 and 3 are correct statements. land revenue, which they got with
The Ryotwari Settlement was Gandhiji’s help.
introduced by Reed and Munro in the 7. (a) To agitate for Indian political reforms
presidency towns of Bombay and in the House of Commons.
Madras, whereby the Government
Sir William Wedderburn, a retired
made a settlement directly with the
English civil servant, was actively
peasants (Ryots).
involved in Congress politics,
becoming its president twice.
2011 Subsequently, he became a liberal
member of the British Parliament,
1. (d) None of the (a), (b) and (c) above. where he set up the Indian
The Permanent Settlement gave Parliamentary Committee and acted
formal land ownership titles to the as its chairman between 1893 and
Zamindars. This formalitisation of 1900 to voice India’s grievances in
land titles made a flurry of legal the House of Commons
petitions. 8. (d) All the statements (a), (b) and (c) are
2. (a) It was a non-violent movement. correct in this context
The movement of 1942, launched by “Unto this Last”, was an essay on
Gandhi, was initially a non-violent economy written in 1860 by an
movement like earlier Gandhian English Catholic Socialist, John
movements. It became a violent Ruskin. Gandhiji was tremendously
movement later on due to heavy influenced by this book.
repression by the British. 9. (a) Running the secret Congress Radio
3. (d) The complete disruption of the old in the wake of Quit India Movement.
agrarian order to the tribal Usha Mehta was involved in running
communities the secret Congress Radio in the
Though the factors mentioned in (a), wake of the Quit India Movement in
(b) and (c) also played important role 1942.
in most of the tribal revolts of the 19th 10. (b) 2 and 3 only
century, the disruption of old agrarian The Nehru Report (1928) demanded
order of the tribal communities was Dominion Status and not Complete
the most important cause. Independence. Purna Swaraj
(Complete Independence) was

28

https://telegram.me/CivilServices_UPSC
https://t.me/UPSC_PDF

ANSWERS - HISTORY OF INDIA AND INDIAN NATIONAL MOVEMENT

demanded at the Lahore Session of Moreover, they wanted to restricting


1929. the movement of Boycott to the
11. (a) Universal Law British goods only. The extremists
The Jain Philosophy is dualistic in wanted to extend the movement to
nature, holding that the world the rest of India and to include not
consists of two eternal, co-existing only the goods, but also the British
but independent categories, viz. Jiva educational system and anything
and Ajiva. There is no separate which can be identified with the
creator or preserver, except the British.
Universal (Natural) Law. 3. (d) A solution for the constitutional
12. (c) Both 1 and 2 deadlock
The Harappans, though had their In March 1944, C. Rajagopalachari
own religious beliefs, as evident from evolved a formula for the Congress,
the archaeological evidence, but they with the full approval of Gandhiji for
did not allow religion to dominate the Congress (Muslim League). The
their public life. Secondly, they were scheme involved :
probably the first in the world to (i) The League would endorse the
produce cotton. demand for independence and
13. (c) Both 1 and 2 Co-operate with the Congress
The term “Dharma” means “law in forming a provisional
requires everyone to be dutiful and government for the transitional
law-abiding”, and in of time it period.
developed into the “Varnashrama (ii) At the end of the war, a
Dharma”. The term “Rita”, meaning plebiscite of all the inhabitants
“cosmic order required nature and in the Muslim-majority areas in
morality to function properly”. The the north-west and the north-
Vedic god Varuna is supposed to be east would decide whether or
upholder of this “Rita”. not they should form a separate
state.
(iii) In the event of separation,
2010
agreements would be made for
Defence, Communications and
1. (a) It recommended the replacement of other essential matters.
diarchy with responsible government
in the provinces (iv) These terms were to be binding
The Simon Commission was to only in case of transfer by
enquire into the working of the England of full powr and
system of the government, the responsibility for the
growth of education and the government of India.
development of representative However, the constitutional deadlock
institution in British India, and continued.
matters connected therewith. Further 4. (a) Bagh caves
it was set up to report as to weather Bagh caves in Dhar district of
and to what extent it is desirable to Madhya Pradesh were built by
establish the principle of responsible Dattaka, a Buddhist monk during the
government or to extend, modify or Gupta period.
restrict the degree of responsible Lomas Rishi cave (Barabar, Bihar)
government then existing therein. and Nashik caves were built before
2. (a) Annulment of partition of Bengal the Gupta period.
In the years between 1905 and 1907, Ellora caves were built after the
great differences arose among the Gupta period.
Moderates and the Extremists. The 5. (c) Cornwallis
Moderates wanted to confine to the
Swadeshi and Boycott Movements Lord Cornwallis (1786 - 1793) was
only in the province of Bengal. the founder of the Indian Civil

29

https://telegram.me/CivilServices_UPSC
https://t.me/UPSC_PDF

ANSWERS - HISTORY OF INDIA AND INDIAN NATIONAL MOVEMENT

Service as well as the Police business community, it strengthened


Administration. the hands of the moderators, within
6. (a) The partition of Bengal done by Lord the Congress, like Bhulabhai Desai
Curzon and G.B. Pant, who put pressure on
In July, 1905 Lord Curzon (1899 - Nehru to tone down his socialist
1905) announced, that the province views.
of Bengal would be partitioned. He 10. (c) Nagarjuna
gave the reason that Bengal was too Tyagaraja was one of the greatest
big to be administered from a single composers of Carnatic music. He,
capital. The nationalist leaders along with his contemporaries,
however, realized the hidden motive Muthuswami Dikshitar and Shyama
of Curzon and launched Swadeshi Shastri, formed the Trinity of Carnatic
Movement to oppose it. music. He was a prolific composer
7. (b) 2 only and highly influential in the
development of the South Indian
Raj Kumar Shukla, a peasant leader
classical music tradition. Tyagraja
from Champaran, met Gandhiji at the
composed thousands of devotional
Lucknow sSession of the Congress
compositions, most of them in praise
(1916) and invited Gandhiji to come
of Lord Rama.
to Champaran and investigate the
11. (c) Pandit Nehru and Maulana Azad
problem of peasants. There were
Cripps Mission was an attempt in late
several nationalists who
March 1942 by the British
accompanied him, viz.,
Government to secure Indian co-
J. B. Kriplani, Rajendra Prasad, operation and support for their efforts
Narhari Parikh, Mahadev Desai in World War-II. The Mission was
and Mazhar ul Haque headed by Sir Stafford Cripps, a
8. (c) Lord Cornwallis was alarmed at the senior left-using politician and
extent of power concentrated in the government minister.
District Collector and felt that such 12. (b) Pandit Jawaharlal Nehru
absolute power was undesirable in The Karachi Session of the Congress
one person (1931) was one of the historic
Cornwallis’ judicial reforms took the sessions. It endorsed the Gandhi-
final shape by 1793 and were Irwin Pact of 5th March, 1931. A
embodied in the famous Cornwallis resolution on Fundamental Rights
Code. The new reforms were based and Economic Planning Programme
on the principle of separation of were formulated by Nehru.
powers. The Cornwallis Code 13. (a) He wanted to avenge the expulsion
divested the Collector of all judicial by Marathas of his viceroy Timur
and magisterial powers and left him Shah from Lahore
with the duty of administration of In March 1758, Raghunath Rao, the
revenue. A new officer called the Maratha leader, crossed into the
District Judge was created to preside Punjab and drove away Timur Shah,
over the District Civil Court. The Ahmad Shah Abdali’s son and agent,
District Judge was also given out of the Punjab.
magisterial and police functions. 14. (a) 1 only
9. (a) 1 only The first to come in India were the
The “Bombay Manifesto” signed in Portuguese.
1936 by 21 Bombay businessmen, 15. (a) 1 only
contained an open indictment of The rulers who invaded India
Nehru’s preaching of socialist ideals, accepted Buddhism, e.g. Kanishka.
which were deemed prejudicial to Though Gupta, e.g., rulers accepted
private property, and to the peace Hinduism but they did not oppose
and prosperity of the country. Buddhism.
Although it did not evoke support
from any other section of the

30

https://telegram.me/CivilServices_UPSC
https://t.me/UPSC_PDF

UPSC
PRELIMINARY EXAMINATION
(PAPER-I)
GENERAL STUDIES

PREVIOUS YEARS QUESTIONS

(YEAR 2010 TO 2019)

THEME  INDIAN AND WORLD


GEOGRAPHY

https://telegram.me/CivilServices_UPSC
https://t.me/UPSC_PDF

INDIAN AND WORLD GEOGRAPHY

(a) 1 only
2019
(b) 2 and 3 only
(c) 3 only
1. Consider the following statements:
(d) 1, 2 and 3
1. Agricultural soils release nitrogen
oxides into environment.
5. Consider the following States:
2. Cattle release ammonia into
environment. 1. Chhattisgarh
3. Poultry industry releases reactive 2. Madhya Pradesh
nitrogen compounds into 3. Maharashtra
environment. 4. Odisha
Which of the statements given above is/are With reference to the States mentioned
correct? above, in terms of percentage of forest
(a) 1 and 3 only cover to the total area of State, which one
(b) 2 and 3 only of the following is the correct ascending
order?
(c) 2 only
(a) 2-3-1-4
(d) 1, 2 and 3
(b) 2-3-4-1
(c) 3-2-4-1
2. What is common to the places known as
Aliyar, Isapur and Kangsabati? (d) 3-2-1-4
(a) Recently discovered uranium
deposits 6. Consider the following pairs:
(b) Tropical rain forests Sea : Bordering
(c) Underground cave systems country
(d) Water reservoirs 1. Adriatic Sea : Albania
2. Black Sea : Croatia
3. Why are dewdrops not formed on a cloudy
3. Caspian Sea : Kazakhstan
night?
4. Mediterranean : Morocco
(a) Clouds absorb the radiation released
from the Earth's surface. Sea

(b) Clouds reflect back the Earth's 5. Red Sea : Syria


radiation. Which of the pairs given above are
(c) The Earth's surface would have low correctly matched?
temperature on cloudy nights. (a) 1, 2 and 4 only
(d) Clouds deflect the blowing wind to (b) 1, 3 and 4 only
ground level. (c) 2 and 5 only
(d) 1, 2, 3, 4 and 5
4. Consider the following statements:
1. Coal sector was nationalized by the 7. Among the following, which one is the
Government of India under Indira largest exporter of rice in the world in the
Gandhi. last five years?
2. Now, coal blocks are allocated on (a) China
lottery basis.
(b) India
3. Till recently, India imported coal to
(c) Myanmar
meet the shortages of domestic
(d) Vietnam
supply, but now India is self-sufficient
in coal production.
Which of the statements given above is/are
correct?

https://telegram.me/CivilServices_UPSC
https://t.me/UPSC_PDF

INDIAN AND WORLD GEOGRAPHY

8. Consider the following pairs: 3. Area of cotton cultivation is more


Glacier : River than that of sugarcane.
4. Area under sugarcane cultivation has
1. Bandarpunch : Yamuna
steadily decreased.
2. Bara Shigri : Chenab
Which of the statements given above are
3. Milam : Mandakini correct?
4. Siachen : Nubra (a) 1 and 3 only
5. Zemu : Manas (b) 2, 3 and 4 only

Which of the pairs given above are (c) 2 and 4 only


correctly matched? (d) 1, 2, 3 and 4
(a) 1, 2 and 4
(b) 1, 3 and 4 13. Consider the following statements about
(c) 2 and 5 Particularly Vulnerable Tribal Groups
(PVTGs) in India:
(d) 3 and 5
1. PVTGs reside in 18 States and one
Union Territory.
9. Which one of the following National Parks
2. A stagnant or declining population is
lies completely in the temperate alpine
one of the criteria for determining
zone?
PVTG status.
(a) Manas National Park
3. There are 95 PVTGs officially notified
(b) Namdapha National Park in the country so far.
(c) Neora Valley National Park 4. Irular and Konda Reddi tribes are
(d) Valley of Flowers National Park included in the list of PVTGs.
Which of the statements given above are
st
10. On 21 June, the Sun correct?
(a) does not set below the horizon at the (a) 1, 2 and 3
Arctic Circle (b) 2, 3 and 4
(b) does not set below the horizon at (c) 1, 2 and 4
Antarctic Circle (d) 1, 3 and 4
(c) shines vertically overhead at noon on
the Equator
(d) shines vertically overhead at the 2018
Tropic of Capricorn
1. Which one of the following is an artificial
11. Which one of the following groups of plants lake?
was domesticated in the 'New World' and (a) Kodaikanal (Tamil Nadu)
introduced into the 'Old World? (b) Kolleru (Andhra Pradesh)
(a) Tobacco, cocoa and rubber (c) Nainital (Uttarakhand)
(b) Tobacco, cotton and rubber (d) Renuka (Himachal Pradesh)
(c) Cotton, coffee and sugarcane
(d) Rubber, coffee and wheat 2. With reference to agricultural soils, consider
the following statements :
12. With reference to the cultivation of Kharif 1. A high content of organic matter in soil
crops in India in the last five years, drastically reduces its water holding
consider the following statements : capacity.
1. Area under rice cultivation is the 2. Soil does not play any role in the sulphur
highest. cycle.
2. Area under the cultivation of jowar is
more than that of oilseeds.

https://telegram.me/CivilServices_UPSC
https://t.me/UPSC_PDF

INDIAN AND WORLD GEOGRAPHY

3. Irrigation over a period of time can (c) 2 and 3 (d) 3 and 4


contribute to the salinization of some
agricultural lands. 6. Consider the following statements :
Which of the statements given above is/ are 1. In India, State Governments do not have
correct ? the power to auction non-coal mines.
(a) 1 and 2 only 2. Andhra Pradesh and Jharkhand do not
(b) 3 only have gold mines.
(c) 1 and 3 only 3. Rajasthan has iron ore mines.
(d) 1, 2 and 3 Which of the statements given above is/ are
correct ?
3. Consider the following statements : (a) 1 and 2 (b) 2 only
1. The Barren Island volcano is an active (c) 1 and 3 (d) 3 only
volcano located in the Indian territory.
2. Barren Island lies about 140 km east of 7. Among the following cities, which one lies on
Great Nicobar. a longitude closest to that of Delhi?
3. The last time the Barren Island volcano (a) Bengaluru (b) Hyderabad
erupted was in 1991 and it has remained (c) Nagpur (d) Pune
inactive since then.
Which of the statements given above is/ are
correct ? 2017
(a) 1 only (b) 2 and 3
(c) 3 only (d) 1 and 3 1. At one of the places in India, if you stand on
the seashore and watch the sea, you will find
4 Consider the following pairs : that the sea water recedes from the shore
line a few kilometres and comes back to the
Regions sometimes Country
shore, twice a day, and you can actually walk
mentioned in news
on the sea floor when the water recedes. This
1. Catalonia - Spain unique phenomenon is seen at
2. Crimea - Hungary (a) Bhavnagar
3. Mindanao - Philippines (b) Bheemunipatnam
(c) Chandipur
4. Oromia - Nigeria
(d) Nagapattinam
Which of the pairs given above are correctly
matched?
2. With reference to river Teesta, consider the
(a) 1, 2 and 3
following statements :
(b) 3 and 4 only
1. The source of river Teesta is the same as
(c) 1 and 3 only that of Brahmaputra but it flows through
(d) 2 and 4 only Sikkim,
2. River Rangeet originates in Sikkim and it
5. Consider the following pairs : is a tributary of river Teesta.
Towns sometimes Country 3. River Teesta flows into Bay of Bengal on
mentioned in news the border of India and Bangladesh.
1. Aleppo - Syria Which of the statements given above is/ are
2. Kirkuk - Yemen correct ?
3. Mosul - Palestine (a) 1 and 3 only
4. Mazar-i-sharif - Afghanistan (b) 2 only
Which of the pairs given above are correctly (c) 2 and 3 only
matched?
(d) 1, 2 and 3
(a) 1 and 2 (b) 1 and 4

https://telegram.me/CivilServices_UPSC
https://t.me/UPSC_PDF

INDIAN AND WORLD GEOGRAPHY

3. If you travel by road from Kohima to (a) 1 only (b) 2 only


Kottayam, what is the minimum number of (c) Both 1 and 2 (d) Neither 1 nor 2
States within India through which you can
travel, including the origin and the destination
7. Which of the following practices can help in
?
water conservation in agriculture?
(a) 6
1. Reduced or zero tillage of the land
(b) 7
2. Applying gypsum before irrigating the
(c) 8 field
(d) 9 3. Allowing crop residue to remain in the
field
4. Mediterranean Sea is a border, following Select the correct answer using the code
countries ? given below :
1. Jordan (a) 1 and 2 only (b) 3 only
2. Iraq (c) 1 and 3 only (d) 1, 2 and 3
3. Lebanon
4. Syria 8. Which of the following is geographically
Select the correct answer using the code closest to Great Nicobar ?
given below: (a) Sumatra
(a) 1, 2 and 3 only (b) Borneo
(b) 2 and 3 only (c) Java
(c) 3 and 4 only (d) Sri Lanka
(d) 1, 3 and 4 only

2016
5. Consider the following statements :
1. In India, the Himalayas are spread over 1. Which of the following is/are tributary/
five States only. tributaries of Brahmaputra?
1. Dibang
2. Western Ghats are spread over five
States only. 2. Kameng
3. Pulicat Lake is spread over two States 3. Lohit
only. Select the correct answer using the code
Which of the statements given above is/ are given below.
correct ? (a) 1 only
(a) 1 and 2 only (b) 2 and 3 only
(b) 3 only (c) 1 and 3 only
(c) 2 and 3 only (d) 1, 2 and 3
(d) 1 and 3 only
2. With reference to 'Red Sanders', sometimes
6. With reference to 'Indian, Ocean Dipole seen in the news, consider the following
(IOD)' sometimes mentioned in the news statements :
while forecasting Indian monsoon, which of 1. It is a tree species found in a part of
the following statements is/are correct? South India.
1. IOD phenomenon is characterised by a 2. It is one of the most important trees in the
difference in sea surface temperature tropical rain forest areas of South India.
between tropical Western Indian Ocean Which of the statements given above is /are
and tropical Eastern Pacific Ocean. correct?
2. An IOD phenomenon can influence an El (a) 1 only
Nino's impact on the monsoon. (b) 2 only
Select the correct answer using the code (c) Both 1 and 2
given below:

https://telegram.me/CivilServices_UPSC
https://t.me/UPSC_PDF

INDIAN AND WORLD GEOGRAPHY

(d) Neither 1 nor 2


7. Recently, which of the following States has
3. What is/are the purpose/purposes of 'District explored the possibility of constructing an
Mineral Foundations' in India? artificial inland port to be connected to sea by
1. Promoting mineral exploration activities in a long navigational channel?
mineral-rich districts (a) Andhra Pradesh
2. Protecting the interests of the persons (b) Chhattisgarh
affected by mining operations (c) Karnataka
3. Authorizing State Governments to issue (d) Rajasthan
licences for mineral exploration
Select the correct answer using the code
given below.
2015
(a) 1 and 2 only
(b) 2 only 1. Which one of the following countries of
(c) 1 and 3 only South-West Asia does not open out to the
Mediterranean Sea?
(d) 1, 2 and 3
(a) Syria
(b) Jordan
4. In which of the following regions of India are
shale gas resources found? (c) Lebanon

1. Cambay Basin (d) Israel

2. Cauvery Basin
3. Krishna-Godavari Basin 2. "Each day is more or less the same, the
morning is clear and bright with -a sea
Select the correct answer using the code
breeze; as the Sun climbs high In the sky,
given below.
heat mounts up, dark clouds form, then rain
(a) 1 and 2 only comes with thunder and lightning. But rain is
(b) 3 only soon over." Which of the following regions is
(c) 2 and 3 only described in the, above passage?
(d) 1, 2 and 3 (a) savannah
5. Which of the following is/are the (b) Equatorial
advantage/advantages of practising drip (c) Monson
irrigation? (d) Mediterranean
1. Reduction in weed
2. Reduction in soil salinity 3. In the South Atlantic and South-Eastern
3. Reduction in soil erosion Pacific regions in tropical latitudes, cyclone
Select the correct answer using the code does not originate. What is the reason?
given below. (a) Sea surface temperatures are low
(a) 1 and 2 only (b) Inter-Tropical Convergence Zone seldom
(b) 3 only occurs
(c) 1 and 3 only (c) Coriolis force is too weak
(d) None of the above is an advantage of (d) Absence of land in those regions
practising drip irrigation
4. Which one of the following pairs of States of
6. Recently, linking of which of the following India indicates the easternmost and
rivers was undertaken? westernmost State?
(a) Cauvery and Tungabhadra (a) Assam and Rajasthan
(b) Godavari and Krishna (b) Arunachal Pradesh and Rajasthan
(c) Mahanadi and Sone (c) Assam and Gujarat
(d) Narmada and Tapti (d) Arunachal Pradesh and Gujarat

https://telegram.me/CivilServices_UPSC
https://t.me/UPSC_PDF

INDIAN AND WORLD GEOGRAPHY

9. Which one of the following National Parks


5. What explains the eastward flow of the has a climate that varies from tropical to
equatorial counter-current subtropical, temperate and arctic?
(a) The Earth's rotation on its axis (a) Khangchendzonga National Park
(b) Convergence of the two equatorial (b) Nandadevi National Park
currents (c) Neora Valley National Park
(c) Difference in salinity, of water (d) Namdapha National Park
(d) Occurrence of the' belt of calm near the
equator 10. In India, the steel production industry requires
the import of
6. Consider the following pairs : (a) saltpetre
Place of Location (b) rock phosphate
Pilgrimage (c) coking coal
(d) All of the above
1. Srisailam : Nallamala Hills

2. Omkareshwar : Satmala Hills 11. Consider the following statements


1. The Accelerated Irrigation Benefits
3. Pushkar : Mahadeo Hills Programme was launched during 1996—
97 to provide loan assistance to poor
Which of the above pairs is/are correctly
farmers.
matched?
2. The Command Area Development
(a) 1 only
Programme was launched in 1974—75
(b) 2 and 3 only for the development of water-use
(c) 1 and 3 only efficiency.
(d) 1, 2 and 3 Which of the statements given above is/ are
correct?
7. The area known as 'Golan Heights' (a) 1 only
sometimes appears in the news in the (b) 2 only
context of the events related to (c) Both 1 and 2
(a) Central Asia (d) Neither 1 nor 2
(b) Middle East
(c) South-East Asia 12. Consider the following statements
(d) Central Africa 1. The winds which blow between 30°N and
60°S latitudes throughout the year are
8. Consider the following rivers: known as westerlies.
1. Vamandhara 2. The moist air masses that cause winter
2. Indravati rains in North-Western region of India are
3. Pranahita part of westerlies.

4. Pennar Which of the statements given above is/ are


correct ?
Which of the above are tributaries of
Godavari? (a) 1 only

(a) 1, 2 and 3 (b) 2 only

(b) 2, 3 and 4 (c) Both 1 and 2

(c) 1, 2 and 4 (d) Neither 1 nor 2

(d) 2 and 3 only


13. Tides occur in the oceans and seas due to.
which among the following?
1. Gravitational force of the Sun

https://telegram.me/CivilServices_UPSC
https://t.me/UPSC_PDF

INDIAN AND WORLD GEOGRAPHY

2. Gravitational force of the Moon (a) 1 only


3. Centrifugal force of the Earth (b) 2 only
Select the correct answer using the code (c) Both 1 and 2
given below. (d) Neither 1 nor 2
(a) 1 only
(b) 2 and 3 only 3. With reference to 'Changpa' community of
(c) 1 and 3 only India, consider the following statements :
(d) 1, 2 and 3 1. They live mainly in the State of
Uttarakhand.
14. In which of the following activities are Indian 2. They rear the Pashmina goats that yield
Remote Sensing (IRS) satellites used? a fine wool.
(1) Assessment of crop productivity 3. They are kept in the category of
(2) Locating groundwater resources Scheduled Tribes.
(3) Mineral exploration Which of the statements given above is/ are
correct?
(4) Telecommunications
(a) 1 only
(5) Traffic studies
(b) 2 and 3 only
Select the correct answer using the code
given below. (c) 3 only
(a) 1, 2 and 3 only (d) 1, 2 and 3
(b) 4 and 5 only
(c) 1 and 2 only 4. In the context of food and nutritional security
of India, enhancing the 'Seed Replacement
(d) 1, 2, 3, 4 and 5
Rates' of various crops helps in achieving the
food production targets of the future. But
2014 what is/are the constraint/ constraints in its
wider/ greater implementation?
1. Every year, a monthlong ecologically
1. There is no National Seeds Policy in
important campaign/festival is held during
place.
which certain communities/ tribes plant
saplings of fruit-bearing trees. Which of the 2. There is no participation of private sector
following are such communities/ tribes? seed companies in the supply of quality
seeds of vegetables and planting
(a) Bhutia and Lepcha
materials of horticultural crops.
(b) Gond and Korku
3. There is a demand-supply gap regarding
(c) Irula and Toda
quality seeds in case of low value and
(d) Sahariya and Agariya high volume crops.
Select the correct answer using the code
2. With reference to two non- conventional given below.
energy sources called 'coalbed methane' and (a) 1 and 2
'shale gas', consider the following statements
(b) 3 only
:
(c) 2 and 3
1. Coalbed methane is the pure methane
(d) None
gas extracted from coal seams, while
shale gas is a mixture of propane and
butane only that can be extracted from 5. Consider the following pairs:
finegrained sedimentary rocks. Region Well-known for
2. In India, abundant coalbed methane
the production of
sources exist, but so far no shale gas
sources have been found. 1. Kinnaur : Areca nut
Which of the statements given above is/ are
correct? 2. Mewat : Mango

https://telegram.me/CivilServices_UPSC
https://t.me/UPSC_PDF

INDIAN AND WORLD GEOGRAPHY

3. Coromandel : Soya bean (b) Nicobar and Sumatra


(c) Maldives and Lakshadweep
Which of the above pairs is/are correctly
(d) Sumatra and Java
matched ?
(a) 1 and 2 only
9. Consider the following pairs :
(b) 3 only
Programme / Ministry
(c) 1, 2 and 3
(d) None Project

1. Drought-Prone : Ministry of
6. Consider the following pairs: Agriculture
National Cities connected Area Programme
Highway
2. Desert : Ministry of
Development Environment
Programme and Forests
1. NH 4 : Chennai and Hyderabad
3. National : Ministry of
2. NH 6 : Mumbai and Kolkata Watershed Rural
Development Development
3. NH 15 : Ahmedabad and Jodhpur Project for
Rainfed Areas
Which of the above pairs is/are correctly
matched? Which of the above pairs is/are correctly
(a) 1 and 2 only matched ?
(b) 3 only (a) 1 and 2 only
(c) 1, 2 and 3 (b) 3 only
(d) None (c) 1, 2 and 3
(d) None

7. What are the significances of a practical


approach to sugarcane production known as 10. Consider the following pairs :
'Sustainable Sugarcane Initiative'? 1. Dampa Tiger : Mizoram Reserve
1. Seed cost is very low in this compared to 2. Gumti Wildlife : Sikkim Sanctuary
the conventional method of cultivation. 3. Saramati Peak : Nagaland
2. Drip irrigation can be practiced very Which of the above pairs is/are correctly
effectively in this. matched?
3. There is no application of (a) 1 only
chemical/inorganic fertilizers at all in this. (b) 2 and 3 only
4. The scope for intercropping is more in (c) 1 and 3 only
this compared to the conventional
(d) 1, 2 and 3
method of cultivation.
Select the correct answer using the code
11. Which of the following phenomena might
given below.
have influenced the evolution of organisms?
(a) 1 and 3 only
1. Continental drift
(b) 1, 2 and 4 only
2. Glacial cycles
(c) 2, 3 and 4 only
Select the correct answer using the code
(d) 1, 2, 3 and 4
given below.
(a) 1 only
8. Which one of the following pairs of islands is
(b) 2 only
separated from each other by the 'Ten
(c) Both 1 and 2
Degree Channel'?
(d) Neither 1 nor 2
(a) Andaman and Nicobar

https://telegram.me/CivilServices_UPSC
https://t.me/UPSC_PDF

INDIAN AND WORLD GEOGRAPHY

12. In India, the problem of soil erosion is (b) Black Sea and Mediterranean Sea
associated with which of the following? (c) Gulf of Suez and Mediterranean Sea
1. Terrace cultivation (d) Gulf of Aqaba and Dead Sea
2. Deforestation
3. Tropical climate 17. What is the correct sequence of occurrence
Select the correct answer using the code of the following cities in South-East Asia as
given below. one proceeds from south to north?
(a) 1 and 2 only 1. Bangkok
(b) 2 only 2. Hanoi
(c) 1 and 3 only 3. Jakarta
(d) 1, 2 and 3 4. Singapore
Select the correct answer using the code
13. The seasonal reversal of winds is the typical given below.
characteristic of (a) 4-2-1-3
(a) Equatorial climate (b) 3-2-4-1
(b) Mediterranean climate (c) 3-4-1-2
(c) Monsoon climate (d) 4-3-2-1
(d) All of the above climates
18. What are the benefits of implementing the
14. Consider the following rivers 'Integrated Watershed Development
1. Barak Programme' ?
2. Lohit 1. Prevention of soil runoff
3. Subansiri 2. Linking the country's perennial rivers with
seasonal rivers
Which of the above flows/flow through
Arunachal Pradesh? 3. Rainwater harvesting and recharge of
groundwater table
(a) 1 only
4. Regeneration of natural vegetation
(b) 2 and 3 only
Select the correct answer using the code
(c) 1 and 3 only
given below.
(d) 1, 2 and 3
(a) 1 and 2 only
(b) 2, 3 and 4 only
15. Consider the following pairs :
(c) 1, 3 and 4 only
Region Well-known for the
production of (d) 1, 2, 3 and 4

1. Kinnaur : Areca nut


2013
2. Mewat : Mango

3. Coromandel : Soya bean 1. Consider the following pairs :


National Park River flowing
Which of the above pairs are correctly through the Park
matched?
(a) 1 and 2 1. Corbett National : Ganga
(b) 2 and 3 Park
(c) 3 and 4
2. Kaziranga : Manas
(d) 2 and 4 National Park

16. Turkey is located between 3. Silent Valley : Kaveri


National Park
(a) Black Sea and Caspian Sea

10

https://telegram.me/CivilServices_UPSC
https://t.me/UPSC_PDF

INDIAN AND WORLD GEOGRAPHY

Which of the above pairs is/are correctly (a) Abyssiinian : Arabia


matched? Plateau
(a) 1 and 2
(b) Atlas : North –Western
(b) 3 only
Moutains Africa
(c) 1 and 3
(d) None (c) Guiana : South-western
Highlands Africa
2. Consider the following organisms
(d) Okavango : Patagonia
1. Agaricus Basin
2. Nostoc
3. Spirogyra
Which of the above is/are used as biofertilizer 6. Consider the following :
/ biofertilizers? 1. Star tortoise
(a) 1 and 2 2. Monitor lizard
(b) 2 only 3. Pygmy hog
(c) 2 and 3 4. Spider monkey
(d) 3 only Which of the above are naturally found in
India ?
3. Which of the following adds/add nitrogen to (a) 1, 2 and 3 only
the soil? (b) 2 and 3 only
1. Excretion of urea by animals (c) 1 and 4 only
2. Burning of coal by man (d) 1, 2, 3 and 4
3. Death of vegetation
Select the correct answer using the codes 7. Consider the following animals :
given below. 1. Sea cow
(a) 1 only 2. Sea horse
(b) 2 and 3 only 3. Sea lion
(c) 1 and 3 only Which of the above is/are mammal/
(d) 1, 2 and 3 mammals?
(a) 1 only
4. In which of the following States is lion-tailed (b) 1 and 3 only
macaque found in its natural habitat? (c) 2 and 3 only
1. Tamil Nadu (d) 1, 2 and 3
2. Kerala
3. Karnataka 8. Variations in the length of daytime and
4. Andhra Pradesh nighttime from season to season are due to
Select the correct answer using the codes (a) the earth's rotation on its axis
given below. (b) the earth's revolution round the sun in an
(a) 1, 2 and 3 only elliptical manner
(b) 2 only (c) latitudinal position of the place
(c) 1, 3 and 4 only (d) revolution of the earth on a tilted axis
(d) 1, 2, 3 and 4
9. The Narmada river flows to the west, while
5. Which one of the following pairs is correctly most other large peninsular rivers flow to the
matched? east. Why?
Geographical Region 1. It occupies a linear rift valley.
Feature

11

https://telegram.me/CivilServices_UPSC
https://t.me/UPSC_PDF

INDIAN AND WORLD GEOGRAPHY

2. It flows between the Vindhyas and the (b) low flat plains, close to stream courses,
Satpuras. liable to flooding
3. The land slopes to the west from Central (c) scrublands, liable to spread of weed
India, growth
Select the correct answer using the codes (d) None of the above
given below.
(a) 1 only (b) 2 and 3 14. Consider the following fauna of India
(c) 1 and 3 (d) None 1. Gharial
2. Leatherback turtle
10. On the planet earth, most of the freshwater 3. Swampdeer
exists as ice caps and glaciers. Out of the Which of the above is/are endangered?
remaining freshwater, the largest proportion
(a) 1 and 2 only
(a) is found in atmosphere as moisture and
(b) 3 only
clouds
(c) 1, 2 and 3
(b) is found in freshwater lakes and rivers
(d) None
(c) exists as groundwater
(d) exists as soil moisture
15. The most important fishing grounds of the
world are found in the regions where
11. Consider the following pairs :
(a) warm and cold atmospheric currents
1. Nokrek Bio sphere : Reserve Garo Hills meet
2. Logtak (Loktak) Lake : Range (b) rivers drain out large amounts of
3. Namdapha National Park : Dafla Hills freshwater into the sea
Which of the above pairs is/are correctly (c) warm and cold oceanic currents meet
matched? (d) continental shelf is undulating
(a) 1 only
(b) 2 and 3 only 16. Which of the following is/arc unique
(c) 1, 2 and 3 characteristic/characteristics of equatorial
(d) None forests?
1. Presence of tall, closely sat trees with
12. Consider the following: crowns forming a continuous canopy
1. Electromagnetic radiation 2. Co-existence of a large number of
2. Geothermal energy species
3. Gravitational force 3. Presence of numerous varieties of
epiphytes
4. Plate movements
Select the correct answer using the codes
5. Rotation of the earth
given below.
6. Revolution of the earth
(a) 1 only
Which of the above are responsible for
(b) 2 and 3 only
bringing dynamic changes on the, surface of
the earth? (c) 1 and 3 only
(a) 1, 2, 3 and 4 only (d) 1, 2 and 3
(b) 1, 3, 5 and 6 only
(c) 2, 4, 5 and 6 only 17. The annual range of temperature in the
interior of the continents is high as compared
(d) 1, 2, 3, 4, 5 and 6
to coastal areas. What is/are the
reason/reasons?
13. Contour bunding is a method of soil
1. Thermal difference between land and
conservation used in
water
(a) desert margins, liable to strong wind
action

12

https://telegram.me/CivilServices_UPSC
https://t.me/UPSC_PDF

INDIAN AND WORLD GEOGRAPHY

2. Variation in altitude between continents (c) 2 and 3


and oceans (d) None
3. Presence of strong winds in the interior
4. Heavy rains in the interior as compared 21. Consider the following crops
to coasts 1. Cotton
Select the correct answer using the codes 2. Groundnut
given below.
3. Rice
(a) 1 only
4. Wheat
(b) 1 and 2 only
Which of these are Kharif crops?
(c) 2 and 3 only
(a) 1 and 4 (b) 2 and 3 only
(d) 2, 3 and 4
(c) 1, 2 and 3 (d) 2, 3 and 4

18. Which of the following is/are the


22. "Climate is extreme, rainfall is scanty and the
characteristic/characteristics of Indian coal?
people used to be nomadic herders."
1. High ash content
The above statement best describes which of
2. Low sulphur content the following regions?
3. Low ash fusion temperature (a) African Savannah
Select the correct answer using the codes (b) Central Asian Steppe
given below.
(c) North American Prairie
(a) 1 and 2 only
(d) Siberian Tundra
(b) 2 only
(c) 1 and 3 only
23. During a thunderstorm, the thunder in the
(d) 1, 2 and 3 skies is produced by the
1. meeting of cumulonimbus clouds in the
19. Which of the following statements regarding sky
laterite soils of India are correct? 2. lightning that separates the nimbus
1. They are generally red in colour. clouds
2. They are rich in nitrogen and potash. 3. violent upward movement of air
3. They are well-developed in Rajasthan and water particles
and UP. Select the correct answer using the codes
4. Tapioca and cashew nuts grow well on given below.
these soils. (a) 1 only
Select the correct answer using the codes (b) 2 and 3
given below. (c) 1 and 3
(a) 1, 2 and 3 (d) None of the above produces the thunder
(b) 2, 3 and 4
(c) 1 and 4
(d) 2 and 3 only
2012

20. Consider the following statements: 1. Despite having large reserves of coal, why
does India import millions of tonnes of coal?
1. Natural gas occurs in the Gondwana
beds. 1. It is the policy of India to save its own
2. Mica occurs in abundance in Kodarma coal reserves for future, and import it
from other countries for the present use.
3. Dharwars are famous for petroleum.
2. Most of the power plants in India are
Which of the statements given above is/ are
coal-based and they are not able to get
correct?
sufficient supplies of coal from within the
(a) 1 and 2 country.
(b) 2 only

13

https://telegram.me/CivilServices_UPSC
https://t.me/UPSC_PDF

INDIAN AND WORLD GEOGRAPHY

3. Steel companies need large quantity of (a) 1 only


coking coal which has to be imported. (b) 2 only
Which of the statements given above is/ are (c) Both 1 and 2
correct? (d) Neither 1 nor 2
(a) 1 only
(b) 2 and 3 only 5. If National Water Mission is properly and
(c) 1 and 3 only completely implemented, how will it impact
(d) 1, 2 and 3 the country?
1. Part of the water needs of urban areas
2. A person stood alone in a desert on a dark will be met through recycling of waste-
night and wanted to reach his village which water.
was situated 5 km east of the point where he 2. The water requirements of coastal cities
was standing. He had no instruments to find with inadequate alternative sources of
the direction but he located the polestar. The water will be met by adopting appropriate
most convenient way now to reach his village technologies that allow for the use of
is to walk in the ocean water.
(a) direction facing the polestar 3. All the rivers of Himalayan origin will be
(b) direction opposite to the polestar linked to the rivers of peninsular India.
(c) direction keeping the polestar to his left 4. The expenses incurred by farmers for
(d) direction keeping the polestar to his right digging bore-wells and for installing
motors and pump-sets to draw
groundwater will be completely
3. Recently, there has been a concern over the
reimbursed by the Government.
short supply of a group of elements called
Select the correct answer using the codes
'rare earth metals'. Why?
given below :
1. China, which is the largest producer of
(a) 1 only (b) 1 and 2 only
these elements, has imposed some
restrictions on their export. (c) 3 and 4 only (d) 1, 2, 3 and 4
2. Other than China, Australia, Canada and
Chile, these elements are not found in 6. Which of the following is/are cited by the
any Country. scientists as evidence/ evidences for the
3. Rare earth metals are essential for the continued expansion of universe?
manufacture of various kinds of electronic 1. Detection of microwaves in space
items and there is a growing demand for 2. Observation of redshift phenomenon in
these elements. space
Which of the statements given above is/ are 3. Movement of asteroids in space
correct? 4. Occurrence of supernova explosions in
(a) 1 only space
(b) 2 and 3 only Select the correct answer using the codes
(c) 1 and 3 only given below :
(d) 1, 2 and 3 (a) 1 and 2
(b) 2 only
4. Consider the following statements : (c) 1, 3 and 4
1. The duration of the monsoon decreases (d) None of the above can be cited as
from southern India to northern India. evidence
2. The amount of annual rainfall in the
northern plains of India decreases from 7. Which of the following is the chief
east to west. characteristic of 'mixed farming?
Which of the statements given above is/ are (a) Cultivation of both cash crops and food
correct? crops

14

https://telegram.me/CivilServices_UPSC
https://t.me/UPSC_PDF

INDIAN AND WORLD GEOGRAPHY

(b) Cultivation of two or more crops in the recorded more in Gujarat as compared to
same field other States.
(c) Rearing of animals and cultivation of 2. In India, the total geographical area of
crops together coastal wetlands is larger than that of
(d) None of the above inland wetlands.
Which of the statements given above is/ are
8. A particular State in India has the following correct?
characteristics : (a) 1 only
1. It is located on the same latitude which (b) 2 only
passes through northern Rajasthan. (c) Both 1 and 2
2. It has over 80% of its area under forest (d) Neither 1 nor 2
cover.
3. Over 12% of forest cover constitutes 12. Consider the following crops of India :
Protected Area Network in this State. 1. Groundnut
Which one among the following States has all 2. Sesamum
the above characteristics?
3. Pearl millet
(a) Arunachal Pradesh
Which of the above is/are predominantly
(b) Assam rainfed crop/crops?
(c) Himachal Pradesh (a) 1 and 2 only
(d) Uttarakhand (b) 2 and 3 only
(c) 3 only
9. Consider the following crops of India : (d) 1, 2 and 3
1. Cowpea
2. Green gram 13. When you travel in Himalayas, you will see
3. Pigeon pea the following :
Which of the above is/are used as pulse, 1. Deep gorges
fodder and green manure? 2. U-turn river courses
(a) 1 and 2 only 3. Parallel mountain ranges
(b) 2 only 4. Steep gradients causing land- sliding
(c) 1 and 3 only Which of the above can be said to be the
(d) 1, 2 and 3 evidences for Himalayas being young fold
mountains ?
10. Consider the following factors : (a) 1 and 2 only
1. Rotation of the Earth (b) 1, 2 and 4 only
2. Air pressure and wind (c) 3 and 4 only
3. Density of ocean water (d) 1, 2, 3 and 4
4. Revolution of the Earth
Which of the above factors influence the 14. Normally, the temperature decreases with the
ocean currents? increase in height from the Earth's surface,
(a) 1 and 2 only because
(b) 1, 2 and 3 1. the atmosphere can be heated upwards
(c) 1 and 4 only from the Earth's surface
(d) 2, 3 and 4 2. there is more moisture in the upper
atmosphere
3. the air is less dense in the upper
11. With reference to the wetlands of India,
atmosphere
consider the following statements :
Select the correct answer using the codes
1. The Country's total geographical area
given below :
under the category of wetlands is

15

https://telegram.me/CivilServices_UPSC
https://t.me/UPSC_PDF

INDIAN AND WORLD GEOGRAPHY

(a) 1 only (c) 2 and 3


(b) 2 and 3 only (d) 3 only
(c) 1 and 3 only
(d) 1, 2 and 3 3. If a tropical rain forest is removed, it does not
regenerate quickly as compared to a tropical
15. The acidification of oceans is increasing. Why deciduous forest. This is because.
is this phenomenon a cause of concern? (a) the soil of rain forest is deficient in
1. The growth and survival of calcareous nutrients
phytoplankton will be adversely affected. (b) propagules of the trees in a rain forest
2. The growth and survival of coral reefs will have poor viability
be adversely affected. (c) the rain forest species are slow-growing
3. The survival of some animals that have (d) exotic species invade the fertile soil of
phytoplanktonic larvae will be adversely rain forest
affected.
4. The cloud seeding and formation of 4. The Himalayan Range is very rich in species
clouds will be adversely affected. diversity. Which one among the following is
Which of the statements given above is / are the most appropriate reason for this
correct? phenomenon?
(a) 1, 2 and 3 only (a) It has a high rainfall that supports
(b) 2 only luxuriant vegetative growth
(c) 1 and 3 only (b) It is a confluence of different
biogeographical zones
(d) 1, 2, 3 and 4
(c) Exotic and invasive species have not
been introduced in this region
2011 (d) It has less human interference

1. India is regarded as a country with 5. La Nina is suspected to have caused recent


"Demographic Dividend". This is due to floods in Australia. How is La Nina different
(a) Its high population in the age group from EI Nino?
below 15 years. 1. La Nina is characterised by unusually
(b) Its high population in the age group of 15- cold ocean temperature in equatorial
64 years. Indian Ocean whereas EI Nino is
characterised by unusually warm ocean
(c) Its high population in the age group
above 65 years. temperature in the equatorial Pacific
Ocean.
(d) Its high total population.
2. El Nino has adverse effect on south west
monsoon of India, but La Nina has no
2. In the context of ecosystem productivity, effect on monsoon climate.
marine upwelling zones are important as they
Which of the statements given above is/ are
increase the marine productivity by bringing
correct?
the
(a) 1 only
1. decomposer microorganisms to the
(b) 2 only
surface.
(c) Both 1 and 2
2. nutrients to the surface
(d) Neither 1 nor 2
3. bottom-dwelling organisms to the
surface.
Which of the statements given above is/ 6. With reference to micro-irrigation, which of
correct? the following statements is/are correct?
(a) 1 and 2 1. Fertilizer/ nutrient loss can be reduced.
(b) 2 only

16

https://telegram.me/CivilServices_UPSC
https://t.me/UPSC_PDF

INDIAN AND WORLD GEOGRAPHY

2. It is the only means of irrigation in dry 1. Southern hemisphere has less landmass
land farming. as compared to northern hemisphere.
3. In some areas of farming, receding of 2. Coriolis force is higher in southern
ground water table can be checked. hemisphere as compared to northern
Select the correct answer using the codes hemisphere.
given below:- Which of the statements given above is/ are
(a) 1 only correct?
(b) 2 and 3 only (a) 1 only
(c) 1 and 3 only (b) 2 only
(d) 1, 2 and 3 (c) Both 1 and 2
(d) Neither 1 nor 2
7. Among the following States, which one has
the most suitable climatic conditions for the
11. Between India and East Asia, the navigation-
cultivation of a large variety of orchids with
time and distance can be greately reduced by
minimum cost of production, and can develop
which of the following?
an export oriented industry in this field ?
(a) Andhra Pradesh 1. Deepeing the Malacca straits between
(b) Arunachal Pradesh Malaysia and Indonesia.

(c) Madhya Pradesh 2. Opening a new canal across the


(d) Uttar Pradesh Kraisthmus between the Gulf of Siam and
Andaman Sea.

8. Which one of the following is not a site for in- Which of the statements given above is/ are
situ method of conservation of flora? correct?
(a) Biosphere Reserve (a) 1 only
(b) Botanical Garden (b) 2 only
(c) National Park (c) Both 1 and 2
(d) Wildlife Sanctuary (d) Neither 1 nor 2

9. The 2004 Tsunami made people realize that 12. The lower Gangetic plain is characterised by
mangroves can serve as a reliable safety humid climate with high temperature
hedge against coastal calamities. How do
throughout the year. Which one among the
mangroves function as a safety hedge?
following pairs of crops is most suitable for
(a) The mangrove swamps separate the
this region ?
human settlements from the sea by a
wide zone in which people neither live (a) Paddy and Cotton
nor venture out (b) Wheat and Jute
(b) The mangroves provide both food and (c) Paddy and Jute
medicines which people are in need of (d) Wheat and Cotton
after any natural disaster
(c) The mangrove trees are tall with dense
13. What could be the main reason/ reasons for
canopies and serve as an excellent
shelter during a cyclone or tsunami the formation of African and Eurasian desert
(d) The magrove trees do not get uprooted belt ?
by storms and tides because of their 1. It is located in the sub-tropical high
extensive roots pressure cells.
2. It is under the influence of warm ocean
10. Westerlies in southern hemisphere are currents.
stronger and persistent than in northern Which of the statements given above is/ are
hemisphere. Why? correct in this context?

17

https://telegram.me/CivilServices_UPSC
https://t.me/UPSC_PDF

INDIAN AND WORLD GEOGRAPHY

(a) 1 only (b) Montane subtropical forest


(b) 2 only (c) Temperate forest
(c) Both 1 and 2 (d) Tropical rain forest
(d) Neither 1 nor 2

2. A new type of El Nino called El Nino Modoki


14. The Brahmaputra, Irrawady and Mekong appeared in the news. In this context,
rivers originate in Tibet and flow through consider the following statements:
narrow and parallel mountain ranges in their 1. Normal El Nino forms in the Central
upper reaches. Of these rivers, Brahmaputra Pacific ocean whereas El-Nino Modoki
makes a "U" turn in its course to flow into forms in Eastern Pacific ocean.
India. This "U" turn is due to 2. Normal EI Nino results in diminished
(a) Uplift of folded Himalayan series. hurricanes in the Atlantic ocean but El
Nino Modoki results in a greater number
(b) Syntaxial bending of geologically young
of hurricanes with greater frequency.
Himalayas
Which of the statements given above is/ are
(c) Geo-tectonic disturbance in the tertiary correct ?
folded mountain chains
(a) 1 only (b) 2 only
(d) Both (a) and (b) above (c) Both 1 and 2 (d) Neither .1 nor 2

15. A state in India has the following 3. Though coffee and tea both are cultivated on
characteristics: hill slopes, there is some difference between
1. Its northern part is arid and semi-arid. them regarding their cultivation. In this
context, consider the following statements:
2. Its central part produces cotton
1. Coffee plant requires a hot and humid
3. Cultivation of cash crops is predominant
climate of tropical areas whereas tea can
over food crops.
be cultivated in both tropical and
Which one of the following states has all of subtropical areas.
the above characteristics? 2. Coffee is propagated by seeds but tea is
(a) Andhra Pradesh propagated by stem cuttings only.
(b) Gujarat Which of the statements given above is/ are
(c) Karnataka correct ?
(a) 1 only
(d) Tamil Nadu
(b) 2 only
(c) Both 1 and 2
2010 (d) Neither 1 nor 2

1. A geographic area with an altitude or 400 4. Consider the following statements:


meters has following characteristics: 1. The boundaries of a National Park are
Month J F M A M J J A S O N D defined by legislation.
Average 31 31 31 31 30 30 29 28 29 29 30 21
Maximum 2. A Biosphere Reserve is declared to
Temp. °C conserve a few specific species of flora
Average 21 21 21 21 21 21 20 20 20 20 20 20
Minimum
and fauna.
Temp. °C 3. In a Wildlife Sanctuary, limited biotic
Rainfall 51 85 188 158 139 121 134 168 185 221 198 86
(mm)
interference is permitted.
If this geographic area were to have a natural Which of the statements given above is/are
forest, which one of the following would it correct ?
most likely be? (a) 1 only
(a) Moist temperate coniferous forest (b) 2 and 3 only

18

https://telegram.me/CivilServices_UPSC
https://t.me/UPSC_PDF

INDIAN AND WORLD GEOGRAPHY

(c) 1 and 3 only (a) They convert atmospheric methane into


(d) 1, 2 and 3 ammonia which the crop plants can
absorb readily
5. With reference to soil conservation, consider (b) They induce the crop plants to produce
the following practices : the enzymes which help convert
atmospheric nitrogen to nitrates
1. Crop rotation
(c) They have the mechanism to convert
2. Sand fences
atmospheric nitrogen into a form that the
3. Terracing
crop plants can absorb readily
4. Wind breaks
(d) They induce the roots of the crop plants
Which of the above are considered to absorb the soil nitrates in larger
appropriate methods for soil conservation in quantities
India?
(a) 1, 2 and 3 only
9. Given below are the names of four energy
(b) 2 and 4 only crops. Which one of them can be cultivated
(c) 1, 3 and 4 only for ethanol ?
(d) 1, 2, 3 and 4 (a) Jatropha (b) Maize
(c) Pongamia (d) Sunflower
6. Following are the characteristics of an area in
India: 10. Consider the following pairs :
1. Hot and humid climate Protected area Well-known for
2. Annual rainfall 200 cm
1. Bhiterkanika, - Salt Water
3. Hill slopes up to an altitude of 1100 Orissa Crocodile
metres Desert National Great Indian
2. -
4. Annual range of temperature 15°C to Park, Rajasthan Bustard
30°C 3. Eravikulam, Kerala - Hoolak Gibbon
Which one among the following crops are you
Which of the pairs given above is/are
most likely to find in the area described
correctly matched?
above ?
(a) 1 only
(a) Mustard
(b) 1 and 2 only
(b) Cotton
(c) 2 only
(c) Pepper
(d) 1, 2 and 3
(d) Virginia tobacco

11. In India, which type of forest among the


7. An objective of the National Food Security
following occupies the largest area ?
Mission is to increase the production of
(a) Montane Wet Temperate Forest
certain crops through area expansion and
productivity enhancement in a sustainable (b) Sub-tropical Dry Evergreen Forest
manner in the identified districts of the (c) Tropical Moist Deciduous Forest
country. What are those crops? (d) Tropical Wet Evcergreen Forest
(a) Rice and wheat only
(b) Rice, wheat and pulses only 12. Tamil Nadu is a leading. producer of mill-
(c) Rice, wheat, pulses and oil seeds only made cotton yam in the country. What could
(d) Rice, wheat, pulses, oil seeds and be the reason ?
vegetables 1. Black cotton soil is the predominant type
of soil in the State.
8. Which feature of some species of blue-green 2. Rich pool of skilled labour is available.
algae helps promote them as bio-fertilizers? Which of the above is/are the correct reasons
?
(a) 1 only

19

https://telegram.me/CivilServices_UPSC
https://t.me/UPSC_PDF

INDIAN AND WORLD GEOGRAPHY

(b) 2 only
(c) Both 1 and 2 18. With reference to the river Luni, which one of
(d) Neither 1 nor 2 the following statements is correct?
(a) It flows into Gulf of Khambhat
13. Consider the following statements : (b) It flows into Gulf of Kuchchh
1. On the planet Earth, the fresh water (c) It flows into Pakistan and merges with a
available for use amounts to about less tributary of Indus
than 1% of the total water found. (d) It is lost in the marshy land of the Rann of
2. Of the total fresh water found on the Kuchchh
planet Earth 95% is bound up in polar ice
caps and glaciers. 19. Which one of the following pairs is not
Which of the statements given above is/ are correctly matched?
correct ? Dam/Lake River
(a) 1 only (b) 2 only
(a) Govind Sagar : Satluj
(c) Both 1 and 2 (d) Neither 1 nor 2
(b) Kolleru Lake : Krishna
(c) Ukai Reservoir : Tapi
14. Rivers that pass through Himachal Pradesh
are (d) Wular Lake : Jhelum
(a) Beas and Chenab only
(b) Beas and Ravi only 20. A geographic region has the following distinct
(c) Chenab, Ravi and Satluj only characteristics:
(d) Beas, Chenab, Ravi, Satluj and Yamuna 1. Warm and dry climate
2. Mild and wet winter
15. When you travel in certain parts of India, you 3. Evergreen oak trees
will notice red soil. What is the main reason The above features are the distinct
for this colour ? characteristics of which one of the following
(a) Abundance of magnesium regions ?
(b) Accumulated humus (a) Mediterranean
(c) Presence of ferric-oxides (b) Eastern China
(d) Abundance of phosphates (c) Central Asia
(d) Atlantic coast of North America
16. Which one of the following is the appropriate
reason for considering the Gondwana rocks 21. As a result of their annual survey, the
as most important of rock systems of India ? National Geographic Society and an
(a) More than 90% of limestone reserves of international polling firm GlobeScan gave
India are found in them India top rank in Greendex 2009 score. What
(b) More than 90% of India's coal reserves is this score ?
are found in them (a) It is a measure of efforts made by
(c) More than 90% of fertile black cotton different countries in adopting
soils are spread over them technologies for reducing carbon footprint
(d) None of the reasons given above is (b) It is a measure of environmentally
appropriate in this context sustainable consumer behaviour in
different countries
(c) It is an assessment of programmes/
17. Which one of the following can one come
schemes undertaken by different
across if one travels through the Strait of
countries for improving the conservation
Malacca ?
of natural resources.
(a) Bali (b) Brunei
(d) It is an index showing the volume of.
(c) Java (d) Singapore
carbon credits sold by different countries

20

https://telegram.me/CivilServices_UPSC
https://t.me/UPSC_PDF

INDIAN AND WORLD GEOGRAPHY

(a) Net area sown 25%; forests 33%; other


22. What causes wind to deflect toward left in the areas 42% .
Southern hemisphere ? (b) Net area sown 58%; forests 17%; other
(a) Temperature areas 25%
(b) Magnetic field (c) Net area sown 43%; forests 29%; other
(c) Rotation of the earth areas 28%
(d) Pressure (d) Net area sown 47%; forests 23%; other
areas 30%

23. If there were no Himalayan ranges, what


would have been the most likely geographical 26. In India, during the last decade the total
impact on India ? cultivated land for which one of the following
crops has remained more or less stagnant ?
1. Much of the country would experience
the cold waves from Siberia. (a) Rice (b) Oilseeds
2. Indo-gangetic plain would be devoid of (c) Pulses (d) Sugarcane
such extensive alluvial soils.
3. The pattern of monsoon would be 27. Due to their extensive rice cultivation, some
different from what it is at present. regions may be contributing to global
Which of the statements given above is/ are warming. To what possible reason/ reasons
correct? is this attributable ?
(a) 1 only (b) 1 and 3 only 1. The anaerobic conditions associated with
rice cultivation cause the emission of
(c) 2 and 3 only (d) 1, 2 and,3
methane.
2. When nitrogen based fertilizers are used,
24. The latitudes that pass through Sikkim also
nitrous oxide, is emitted from the
pass through
cultivated soil.
(a) Rajasthan
Which of the statements given above is/ are
(b) Punjab correct?
(c) Himachal Pradesh (a) 1 only
(d) Jammu & Kashmir (b) 2 only
(c) Both 1 and 2
25. The approximate representation of land use (d) Neither 1 nor 2
classification in India is '

21

https://telegram.me/CivilServices_UPSC
https://t.me/UPSC_PDF

INDIAN AND WORLD GEOGRAPHY

ANSWERS OF INDIAN AND WORLD GEOGRAPHY

2019 2. (a) 1 only


3. (b) 2 only
1. (d) 1, 2 and 3 4. (d) 1, 2 and 3
2. (d) 1, 2 and 3 5. (c) 1 and 3 only
3. (b) Clouds reflect back the Earth's 6. (b) Godavari and Krishna
radiation. 7. (d) Rajasthan
4. (a) 1 only
5. (c) 3-2-1-4 2015
6. (b) 1, 3 and 4 only
7. (b) India 1. (b) Jordan
8. (a) 1, 2 and 4 2. (b) Equatorial
9. (d) Valley of Flowers National Park 3. (a) Sea surface temperatures are low
10. (a) does not set below the horizon at the 4. (d) Arunachal Pradesh and Gujarat
Arctic Circle 5. (a) The Earth’s rotation on its axis
11. (a) Tobacco, cocoa and rubber 6. (a) 1 only
12. (a) 1 and 3 only 7. (b) Middle East
13. (c) 1, 2 and 4 8. (d) 2 and 3 only
9. (d) Namdapha National Park
2018 10. (c) coking coal
11. (b) 2 only
1. (a) Kodaikanal (Tamil Nadu) 12. (b) 2 only
2. (b) 3 only 13. (d) 1, 2 and 3
3. (a) 1 only 14. (a) 1, 2 and 3 only
4. (c) 1 and 3 only
5. (b) 1 and 4
2014
6. (d) 3 only
7. (a) Bengaluru
1 (b) Gond and Korku
2 (a) 1 only
2017
3 (b) 2 and 3 only
4 (b) 3 only
1. (c) Chandipur 5 (d) None
2. (b) 2 only 6 (d) None
3. (b) 7 7 (b) 1, 2 and 4 only
4. (c) 3 and 4 only 8 (a) Andaman and Nicobar
5. (b) 3 only 9 (d) None
6. (b) 2 only 10 (c) 1 and 3 only
7. (d) 1, 2 and 3 11 (c) Both 1 and 2
8. (a) Sumatra 12 (b) 2 only
13 (c) Monsoon climate
14 (b) 2 and 3 only
2016 15 (c) 3 and 4
16 (b) Black Sea and Mediterranean Sea
1. (d) 1, 2 and 3 17 (c) 3–4–1–2

22

https://telegram.me/CivilServices_UPSC
https://t.me/UPSC_PDF

INDIAN AND WORLD GEOGRAPHY

18 (c) 1, 3 and 4 only 13 (c) 3 and 4 only


14 (c) 1 and 3 only
2013 15 (d) 1, 2, 3 and 4

1 (d) None 2011


2 (b) 2 only
3 (c) 1 and 3 only 1. (b) Its high population in the age group
4 (a) 1, 2 and 3 only of 15-64 years.
5 (b) Atlas Moutains : North –Western 2. (b) 2 only
Africa 3. (a) the soil of rain forest is deficient in
6 (a) 1, 2 and 3 only nutrients.
7 (b) 1 and 3 only 4. (b) It is a confluence of different
8 (d) revolution of the earth on a tilted axis biogeographical zones.
9 (a) 1 only 5. (d) Neither 1 nor 2
10 (c) exists as groundwater 6. (d) 1, 2 and 3
11 (a) 1 only 7. (b) Arunachal Pradesh
8. (b) Botanical Garden.
12 (d) 1, 2, 3, 4, 5 and 6
9. (d) The magrove trees do not get
13 (a) desert margins, liable to strong wind uprooted by storms and tides
action because of their extensive roots
14 (c) 1, 2 and 3 10. (a) 1 only
15 (c) warm and cold oceanic currents 11. (c) Both 1 and 2
meet 12. (c) Paddy and Jute
16 (d) 1, 2 and 3 13. (a) 1 only
17 (a) 1 only 14. (d) Both (a) and (b) above
18 (a) 1 and 2only 15. (b) Gujarat
19 (c) 1 and 4
20 (b) 2 only 2010
21 (c) l, 2 and 3
22 (d) Siberian Tundra 1. (d) Tropical rain forest
23 (d) None of the above produces the 2. (b) 2 only
thunder
3. (c) Both 1 and 2
24 (a) 1 and 3 only
4. (c) 1 and 3 only
5. (c) 1, 3 and 4 only
2012 6. (c) Pepper
7. (b) Rice, wheat and pulses only
1 (b) 2 and 3 only 8. (c) They have the mechanism to convert
2 (c) direction keeping the polestar to his atmospheric nitrogen into a form that
left
the crop plants can absorb readily
3 (c) 1 and 3 only
9. (b) Maize
4 (c) Both 1 and 2
10. (a) 1 only
5 (b) 1 and 2 only
11. (c) Tropical Moist Deciduous Forest
6 (a) 1 and 2
12. (b) 2 only
7 (c) Rearing of animals and cultivation of
crops together 13. (c) Both 1 and 2
8 (a) Arunachal Pradesh 14. (d) Beas, Chenab, Ravi, Satluj and
9 (a) 1 and 2 only Yamuna
10 (b) 1, 2 and 3 15. (c) Presence of ferric-oxides
11 (c) Both 1 and 2 16. (b) More than 90% of India's coal
12 (c) 3 only reserves are found in them

23

https://telegram.me/CivilServices_UPSC
https://t.me/UPSC_PDF

INDIAN AND WORLD GEOGRAPHY

17. (d) Singapore


18. (d) It is lost in the marshy land of the
Rann of Kuchchh
19. (b) Kolleru Lake : Krishna
20. (a) Mediterranean
21. (b) It is a measure of environmentally
sustainable consumer behaviour in
different countries
22. (c) Rotation of the earth
23. (d) 1, 2 and 3
24. (a) Rajasthan
25. (d) Net area sown 47%; forests 23%;
other areas 30%
26. (c) Pulses
27. (c) Both 1 and 2

24

https://telegram.me/CivilServices_UPSC
https://t.me/UPSC_PDF

UPSC
PRELIMINARY EXAMINATION
(PAPER-I)
GENERAL STUDIES

PREVIOUS YEARS QUESTIONS

(YEAR 2010 TO 2019)

THEME  INDIAN ECONOMY


(ECONOMIC AND SOCIAL
DEVELOPMENT)

https://telegram.me/CivilServices_UPSC
https://t.me/UPSC_PDF

INDIAN ECONOMY (ECONOMIC AND SOCIAL DEVELOPMENT)

2019
6. Which one of the following is not the most
likely measure the Government/ RBI takes
1. Atal Innovation Mission is set up under the to stop the slide of Indian rupee?
(a) Department of Science and (a) Curbing imports of non-essential
Technology goods and promoting exports
(b) Ministry of Labour and (b) Encouraging Indian borrowers to
Employment issue rupee denominated Masala
(c) NITI Aayog Bonds
(d) Ministry of Skill Development and (c) Easing conditions relating to external
Entrepreneurship commercial borrowing
(d) Following an expansionary monetary
2. The Global Competitiveness Report is policy
published by the
(a) International Monetary Fund 7. Consider the following statements :
(b) United Nations Conference on Trade The Reserve Bank of India's recent
and Development directives relating to 'Storage of Payment
(c) World Economic Forum System Data', popularly known as data
(d) World Bank diktat, command the payment system
providers that
3. In a given year in India, official poverty lines 1. they shall ensure that entire data
are higher in some States than in others relating to payment systems
because operated by them are stored in a
(a) poverty rates vary from State to State system only in India
(b) price levels vary from State to State 2. they shall ensure that the systems
(c) Gross State Product varies from are owned and operated by public
State to State sector enterprises
(d) quality of public distribution varies 3. they shall submit the consolidated
from State to State system audit report to the
Comptroller and Auditor General of
4. Consider the following statements : India by the end of the calendar year
1. Purchasing Power Parity (PPP) Which of the statements given above is/are
exchange rates are calculated by correct?
comparing the prices of the same (a) 1 only
basket of goods and services in (b) 1 and 2 only
different countries. (c) 3 only
2. In terms of PPP dollars, India is the (d) 1, 2 and 3
sixth largest economy in the world.
Which of the statements given above is 8. The money multiplier in an economy
/are correct? increases with which one of the following?
(a) 1 only (a) Increase in the cash reserve ratio
(b) 2 only (b) Increase in the banking habit of the
(c) Both 1 and 2 population
(d) Neither 1 nor 2 (c) Increase in the statutory liquidity ratio
(d) Increase in the population of the
5. Among the agricultural commodities country
imported by India, which one of the
following accounts for the highest imports 9. The Service Area Approach was
in terms of value in the last five years? implemented under the purview of
(a) Spices (a) Integrated Rural Development
(b) Fresh fruits Programme
(c) Pulses (b) Lead Bank Scheme
(d) Vegetable oils

https://telegram.me/CivilServices_UPSC
https://t.me/UPSC_PDF

INDIAN ECONOMY (ECONOMIC AND SOCIAL DEVELOPMENT)

(c) Mahatma Gandhi National Rural 1. Ad Hoc Committees set up by the


Employment Guarantee Scheme Parliament
(d) National Skill Development Mission 2. Parliamentary Department Related
Standing Committees
10. Consider the following statements : 3. Finance Commission
1. Most of India's external debt is owed 4. Financial Sector Legislative Reforms
by governmental entities. Commission
2. All of India's external debt is 5. NITI Aayog
denominated in US dollars. Select the correct answer using the code
Which of the statements given above is/are given below.
correct? (a) 1 and 2
(a) 1 only (b) 1, 3 and 4
(b) 2 only (c) 3, 4 and 5
(c) Both 1 and 2 (d) 2 and 5
(d) Neither 1 nor 2
15. With reference to India's Five-Year Plans,
11. Which of the following is not included in the which of the following statements is/are
assets of a commercial bank in India? correct?
(a) Advances 1. From the Second Five-Year Plan,
(b) Deposits there was a determined thrust
(c) Investments towards substitution of basic and
(d) Money at call and short notice capital good industries.
2. The Fourth Five-Year Plan adopted
12. In the context of India, which of the the objective of correcting the earlier
following factors is/are contributor/ trend of increased concentration of
contributors to reducing the risk of a wealth and economic power.
currency crisis? 3. In the Fifth Five-Year Plan, for the
1. The foreign currency earnings of first time, the financial sector was
India's IT sector included as an integral part of the
2. Increasing the government Plan.
expenditure Select the correct answer using the code
3. Remittances from Indians abroad given below.
Select the correct answer using the code (a) 1 and 2 only
given below. (b) 2 only
(a) 1 only (c) 3 only
(b) 1 and 3 only (d) 1, 2 and 3
(c) 2 only
(d) 1, 2 and 3 16. Consider the following statements :
As per the Industrial Employment (Standing
13. Which of the following is issued by Orders) Central (Amendment) Rules, 2018
registered foreign portfolio investors to 1. if rules for fixed-term employment are
overseas investors who want to be part of implemented, it becomes easier for
the Indian stock market without registering the firms/companies to lay off
themselves directly? workers
(a) Certificate of Deposit 2. no notice of termination of
(b) Commercial Paper employment shall be necessary in
(c) Promissory Note the case of temporary workman
(d) Participatory Note Which of the statements given above is
/are correct?
14. In India, which of the following review the (a) 1 only
independent regulators in sectors like (b) 2 only
telecommunications, insurance, electricity, (c) Both 1 and 2
etc.? (d) Neither 1 nor 2

https://telegram.me/CivilServices_UPSC
https://t.me/UPSC_PDF

INDIAN ECONOMY (ECONOMIC AND SOCIAL DEVELOPMENT)

(b) interest cost only


17. With reference to Asian Infrastructure (c) procurement incidentals and
investment Bank (AIIB), consider the distribution cost
following statements: (d) procurement incidentals and charges
1. AIIB has more than 80 member for godowns
nations.
2. India is the largest shareholder in 22. In the context of any country, which one of
AIIB. the following would be considered as part
3. AIIB does not have any members of its social capital?
from outside Asia. (a) The proportion of literates in the
Which of the statements given above is/are population
correct? (b) The stock of its buildings, other
(a) 1 only infrastructure and machines
(b) 2 and 3 only (c) The size of population in the working
(c) 1 and 3 only age group
(d) 1, 2 and 3 (d) The level of mutual trust and
harmony in the society
18. What was the purpose of Inter-Creditor
Agreement signed by Indian banks and
financial institutions recently? 2018
(a) To lessen the Government of India's
perennial burden of fiscal deficit and 1. With reference to Pradhan Mantri Kaushal
current account deficit Vikas Yojana, consider the following
(b) To support the infrastructure statements :
projects of Central and State
1. It is the flagship scheme of the Ministry of
Governments Labour and Employment.
(c) To act as independent regulator in
2. It, among other things, will also impart
case of applications for loans of Rs
training in soft skills, entrepreneurship,
50 crore or more
financial and digital literacy.
(d) To aim at faster resolution of
stressed assets of Rs 50 crore or 3. It aims to align the competencies of the
more which are under consortium unregulated workforce of the country to
the National Skill Qualification
19. The Chairmen of public sector banks are Framework.
selected by the Which of the statements given above is/ are
(a) Banks Board Bureau correct ?
(b) Reserve Bank of India (a) 1 and 3 only
(c) Union Ministry of Finance (b) 2 only
(d) Management of concerned bank (c) 2 and 3 only
(d) 1, 2 and 3
20. Which one of the following is not a sub-
index of the World Bank's 'Ease of Doing
Business Index'? 2. Which one of the following statements
(a) Maintenance of law and order correctly describes the meaning of legal
(b) Paying taxes tender money ?
(c) Registering property (a) The money which is tendered in courts of
(d) Dealing with construction permits law to defray the fee of legal cases
(b) The money which a creditor is under
21. The economic cost of food grains to the compulsion to accept in settlement of his
Food Corporation of India is Minimum claims
Support Price and bonus if any) paid to the (c) The bank money in the form of cheques,
farmers plus drafts, bills of exchange, etc.
(a) transportation cost only

https://telegram.me/CivilServices_UPSC
https://t.me/UPSC_PDF

INDIAN ECONOMY (ECONOMIC AND SOCIAL DEVELOPMENT)

(d) The metallic money in circulation in a 7. Consider the following statements :


country 1. The Reserve Bank of I ndia manages and
services Government of India Securities
3. If a commodity is provided free to the public but not any State Government Securities.
by the Government, then 2. Treasury bills are issued by the
(a) the opportunity cost is zero. Government of India and there are no
(b) the opportunity cost is ignored. treasury bills issued by the State
Governments.
(c) the opportunity cost is transferred from
the consumers of the product to the tax- 3. Treasury bills offer are issued at a
paying public. discount from the par value.
(d) the opportunity cost is transferred from Which of the statements given above is/ are
the consumers of the product to the correct?
Government. (a) 1 and 2 only
(b) 3 only
4. Increase in absolute and per capita real GNP (c) 2 and 3 only
do not connote a higher level of economic (d) 1, 2 and 3
development, if
(a) industrial output fails to keep pace with 8. Consider the following
agricultural output.
1. Areca nut
(b) agricultural output fails to keep pace with 2. Barley
industrial output.
3. Coffee
(c) poverty and unemployment increase.
(d) imports grow faster than exports. 4. Finger millet
5. Groundnut
5. Consider the following statements : 6. Sesamum
Human capital formation as a concept is 7. Turmeric
better explained in terms of a process which The Cabinet Committee on Economic Affairs
enables has announced the Minimum Support Price
1. individuals of a country to accumulate for which of the above ?
more capital. (a) 1, 2, 3 and 7 only
2. increasing the knowledge, skill levels and (b) 2, 4, 5 and 6 only
capacities of the people of the country.
(c) 1, 3, 4, 5 and 6 only
3. accumulation of tangible wealth.
(d) 1, 2, 3, 4, 5, 6 and 7
4. accumulation of intangible wealth.
Which of the statements given above is/ are
9. With reference to the governance of public
correct?
sector banking in India, consider the following
(a) 1 and 2 statements :
(b) 2 only 1. Capital infusion into public sector banks
(c) 2 and 4 by the Government of India has steadily
(d) 1, 3 and 4 increased in the last decade.
2. To put the public sector banks in order,
6. Despite being a high saving economy, capital the merger of associate banks with the
formation may not result in significant parent State Bank of India has been
increase in output due to affected.
(a) weak administrative machinery Which of the statements given above is/ are
(b) illiteracy correct ?
(c) high population density (a) 1 only
(d) high capital-output ratio (b) 2 only
(c) Both 1 and 2

https://telegram.me/CivilServices_UPSC
https://t.me/UPSC_PDF

INDIAN ECONOMY (ECONOMIC AND SOCIAL DEVELOPMENT)

(d) Neither 1 nor 2 (d) The incentive given by the Government


to merchants for promoting digital
10. Consider the following items : payments by their customers through
Point of Sale (PoS) machines and debit
1. Cereal grains hulled
cards.
2. Chicken eggs cooked
3. Fish processed and canned
13. With reference to India's decision to levy an
4. Newspapers containing advertising
equalization tax of 6% on online
material
advertisement services offered by non-
Which of the above items is/are exempted resident entities, which of the following
under GST (Goods and Services Tax)? statements is/are correct ?
(a) 1 only 1. It is introduced as a part of the Income
(b) 2 and 3 only Tax Act.
(c) 1, 2 and 4 only 2. Non-resident entities that offer
(d) 1, 2, 3 and 4 advertisement services in India can claim
a tax credit in their home country under
11. As per the NSSO 70th Round "Situation the "Double Taxation Avoidance
Assessment Survey of Agricultural Agreements".
Households", consider the following Select the correct answer using the code
statements : given below :
1. Rajasthan has the highest percentage (a) 1 only
share of agricultural households among (b) 2 only
its rural households. (c) Both 1 and 2
2. Out of the total agricultural households in (d) Neither 1 nor 2
the country, a little over 60 percent
belong to OBCs.
14. Consider the following statements :
3. In Kerala, a little over 60 percent of
1. The Fiscal Responsibility and Budget
agricultural households reported to have
Management (FRBM) Review Committee
received maximum income from sources
Report has recommended a debt to GDP
other than agricultural activities.
ratio of 60% for the general (combined)
Which of the statements given above is/ are government by 2023, comprising 40% for
correct? the Central Government and 20% for the
(a) 2 and 3 only State Governments.
(b) 2 only 2. The Central Government has domestic
(c) 1 and 3 only liabilities of 21 % of GDP as compared to
(d) 1, 2 and 3 that of 49% of GDP of the State
Governments.

12. Which one of the following best describes the 3. As per the Constitution of India, it is
term "Merchant Discount Rate" sometimes mandatory for a State to take the Central
seen in news ? Government's consent for raising any
loan if the former owes any outstanding
(a) The incentive given by a bank to a
liabilities to the latter.
merchant for accepting payments through
debit cards pertaining to that bank. Which of the statements given above is/ are
correct ?
(b) The amount paid back by banks to their
customers when they use debit cards for (a) 1 only
financial transactions for purchasing (b) 2 and 3 only
goods or services. (c) 1 and 3 only
(c) The charge to a merchant by a bank for (d) 1, 2 and 3
accepting payments from his customers
through the bank's debit cards.

https://telegram.me/CivilServices_UPSC
https://t.me/UPSC_PDF

INDIAN ECONOMY (ECONOMIC AND SOCIAL DEVELOPMENT)

15. Which one of the following links all the ATMs 2. While a chip-pin debit card has four
in India ? factors of authentication, BHIM app has
(a) Indian Banks' Association only two factors of authentication.
(b) National Securities Depository Limited Which of the statements given above is/ are
(c) National Payments Corporation of India correct ?
(d) Reserve Bank of India (a) 1 only
(b) 2 only
16. Which of the following is/are the aim/ aims of (c) Both land 2
"Digital India" Plan of the Government of (d) Neither 1 nor 2
India?
1. Formation of India's own Internet 19. Consider the following statements :
companies like China did. 1. The quantity of imported edible oils is
2. Establish a policy framework to more than the domestic production of
encourage overseas multinational edible oils in the last five years.
corporations that collect Big Data to build 2. The Government does not impose any
their large data centres within our customs duty on all the imported edible
national geographical boundaries. oils as a special case.
3. Connect many of our villages to the Which of the statements given above is/ are
Internet and bring Wi-Fi to many of our correct ?
schools, public places and major tourist (a) 1 only
centres.
(b) 2 only
Select the correct answer using the code
(c) Both 1 and 2
given below :
(d) Neither 1 nor 2
(a) 1 and 2 only
(b) 3 only
(c) 2 and 3 only 2017
(d) 1, 2 and 3
1. Which of the following statements is/are
17. Consider the following statements : correct regarding the Monetary Policy
1. Capital Adequacy Ratio (CAR) is the Committee (MPC) ?
amount that banks have to maintain in 1. It decides the RBI's benchmark interest
the form of their own funds to offset any rates.
loss that banks incur if the account- 2. It is a 12-member body including the
holders fail to repay dues. Governor of RBI and is reconstituted
2. CAR is decided by each individual bank. every year.
Which of the statements given above is/ are 3. It functions under the chairmanship of the
correct? Union Finance Minister.
(a) 1 only Select the correct answer using the code
(b) 2 only given below :
(c) Both 1 and 2 (a) 1 only
(d) Neither 1 nor 2 (b) 1 and 2 only
(c) 3 only
18. With reference to digital payments, consider (d) 2 and 3 only
the following statements :
1. BHIM app allows the user to transfer 2. Who among the following can join the
money to anyone with a UPI-enabled National Pension System (NPS) ?
bank account. (a) Resident Indian citizens only
(b) Persons of age from 21 to 55 only

https://telegram.me/CivilServices_UPSC
https://t.me/UPSC_PDF

INDIAN ECONOMY (ECONOMIC AND SOCIAL DEVELOPMENT)

(c) All State Government employees joining (b) 2 only


the services after the date of notification (c) Both 1 and 2
by the respective State Governments (d) Neither 1 nor 2
(d) All Central Government employees
including those of Armed Forces joining
6. What is the purpose of setting up of Small
the services on or after 1st April, 2004
Finance Banks (SFBs) in India ?
1. To supply credit to small business units.
3. Consider the following statements :
2. To supply credit to small and marginal
1. The Standard Mark of Bureau of Indian farmers.
Standards (BIS) is mandatory for
3. To encourage young entrepreneurs to set
automotive tyres and tubes.
up business particularly in rural areas.
2. AGMARK is a quality Certification Mark
Select the correct answer using the code
issued by the Food and Agriculture
given below :
Organisation (FAO).
(a) 1 and 2 only
Which of the statements given above is/ are
(b) 2 and 3 only
correct ?
(c) 1 and 3 only
(a) 1 only
(d) 1, 2 and 3
(b) 2 only
(c) Both 1 and 2
7. Which of the following is a most likely
(d) Neither 1 nor 2
consequence of implementing the 'Unified
Payments Interface (UPI)' ?
4. What is/are the advantage/advantages of
(a) Mobile wallets will not be necessary for
implementing the 'National Agriculture Market
online payments.
Scheme'?
(b) Digital currency will totally replace the
1. It is a pan-India electronic trading portal
physical currency in about two decades.
for agricultural commodities.
(c) FDI inflows will drastically increase.
2. It provides the farmers access to
(d) Direct transfer of subsidies to poor
nationwide market, with prices
people will become very effective.
commensurate with the quality of their
produce.
Select the correct answer using the code 8. With reference to 'National Investment and
given below : Infrastructure Fund', which of the following
statements is/are correct ?
(a) 1 only
1. It is an organ of NITI Aayog.
(b) 2 only
2. It has a corpus of Rs. 4,00,000 crore at
(c) Both 1 and 2
present.
(d) Neither 1 nor 2
Select the correct answer using the code
given below :
5. With reference to the 'National Intellectual
(a) 1 only
Property Rights Policy', consider the following
(b) 2 only
statements :
(c) Both 1 and 2
1. It reiterates India's commitment to the
Doha Development Agenda and the (d) Neither 1 nor 2
TRIPS Agreement.
2. Department of Industrial Policy and 9. The Global Infrastructure Facility is a/an
Promotion is the nodal agency for (a) ASEAN initiative to upgrade infrastructure
regulating intellectual property rights in in Asia and financed by credit from the
India. Asian Development Bank.
Which of the above statements is/are correct ? (b) World Bank collaboration that facilitates
(a) 1 only the preparation and structuring of
complex infrastructure Public-Private

https://telegram.me/CivilServices_UPSC
https://t.me/UPSC_PDF

INDIAN ECONOMY (ECONOMIC AND SOCIAL DEVELOPMENT)

Partnerships (PPPs) to enable 1. Share of agriculture in GDP increased


mobilization of private sector and enormously.
institutional investor capital. 2. Share of India's exports in world trade
(c) Collaboration among the major banks of increased.
the world working with the OECD and 3. FDI inflows increased.
focused on expanding the set of 4. India's foreign exchange reserves
infrastructure projects that have the increased enormously.
potential to mobilize private investment.
Select the correct answer using the codes
(d) UNCTAD funded initiative that seeks to given below :
finance and facilitate infrastructure
(a) 1 and 4 only
development in the world.
(b) 2, 3 and 4 only
(c) 2 and 3 only
10. With reference to 'National Skills Qualification
(d) 1, 2, 3 and 4
Framework (NSQF)', which of the statements
given below is/are correct ?
1. Under NSQF, a learner can acquire the 13. Consider the following statements
certification for competency only through 1. National Payments Corporation of India
formal learning. (NPCI) helps in promoting the financial
2. An outcome expected from the inclusion in the country.
implementation of NSQF is the mobility 2. NPCI has launched RuPay, a card,
between vocational and general payment scheme.
education. Which of the statements given above is/ are
Select the correct answer using the code correct ?
given below : (a) 1 only
(a) 1 only (b) 2 only
(b) 2 only (c) Both 1 and 2
(c) Both 1 and 2 (d) Neither 1 nor 2
(d) Neither 1 nor 2
14. What is/are the most likely advantages of
11. Consider the following statements : implementing 'Goods and Services Tax
The nation-wide 'Soil Health Card Scheme' (GST)'?
aims at 1. It will replace multiple taxes collected by
1. expanding the cultivable area under multiple authorities and will thus create a
irrigation. single market in India.
2. enabling the banks to assess the 2. It will drastically reduce the 'Current
quantum of loans to be granted to Account Deficit' of India and will enable it
farmers on the basis of soil quality, to increase its foreign exchange reserves
3. checking the overuse of fertilizers in 3. It will enormously increase the growth
farmlands. and size of economy of India and will
enable it to overtake China in the near
Which of the above statements is/are correct
future.
?
Select the correct answer using the code
(a) 1 and 2 only
given below :
(b) 3 only
(a) 1 only
(c) 2 and 3 only
(b) 2 and 3 only
(d) 1, 2 and 3
(c) 1 and 3 only
(d) 1, 2 and 3
12. Which of the following has/have occurred in
India after its liberalization of economic
policies in 1991 ? 15. Consider the following statements:

https://telegram.me/CivilServices_UPSC
https://t.me/UPSC_PDF

INDIAN ECONOMY (ECONOMIC AND SOCIAL DEVELOPMENT)

1. Tax revenue as a percent of GDP of India


has steadily increased in the last decade. 3. India's ranking in the 'Ease of Doing Business
2. Fiscal deficit as a percent of GDP of India Index' is sometimes seen in the news. Which
has steadily increased in the last decade. of the following has declared that ranking?
Which of the statements given above is/ are (a) Organization for Economic
correct ? Cooperation and Development (OECD)
(a) 1 only (b) World Economic Forum
(b) 2 only (c) World Bank
(c) Both 1 and 2 (d) World Trade Organization (WTO)
(d) Neither 1 nor 2
4. What is/are the purpose/purposes of the
'Marginal Cost of Funds based Lending Rate
2016 (MCLR)' announced by RBI?
1. These guidelines help improve the
1. With reference to 'Stand Up India Scheme', transparency in the methodology followed
which of the following statements is/are by banks for determining the interest
correct? rates on advances.
(1) Its purpose is to promote 2. These guidelines help ensure availability
entrepreneurship among SC/ST and of bank credit at interest rates which are
women entrepreneurs. fair to the borrowers as well as the banks.
(2) It provides for refinance through SIDBI. Select the correct answer using the code
Select the correct answer using the code given below.
given below. (a) 1 only
(a) 1 only (b) 2 only
(b) 2 only (c) Both 1 and 2
(c) Both 1 and 2 (d) Neither 1 nor 2
(d) Neither 1 nor 2
5. With reference to 'Initiative for Nutri-tional
2. The term 'Core Banking Solutions' is Security through Intensive Millets Promotion',
sometimes seen in the news. Which of the which of the following statements is/are
following statements best describes/ describe correct?
this term? 1. This initiative aims to demonstrate the
1. It is a networking of a bank's branches improved production and post-harvest
which enables customers to operate their technologies, and to demonstrate value
accounts from any branch of the bank on addition techniques, in an integrated
its network regardless of where they manner, with cluster approach.
open their accounts. 2. Poor, small, marginal and tribal farmers
2. It is an effort to increase RBI's control have larger stake in this scheme.
over commercial banks through 3. An important objective of the scheme is
computerization. to encourage farmers of commercial
3. It is a detailed procedure by which a bank crops to shift to millet cultivation by
with huge non-performing assets is taken offering them free kits of critical inputs of
over by another bank. nutrients and microirrigation equipment.
Select the correct answer using the code Select the correct answer using the code
given below. given below.
(a) 1 only (a) 1 only
(b) 2 and 3 only (b) 2 and 3 only
(c) 1 and 3 only (c) 1 and 2 only
(d) 1, 2 and 3 (d) 2 and 3

10

https://telegram.me/CivilServices_UPSC
https://t.me/UPSC_PDF

INDIAN ECONOMY (ECONOMIC AND SOCIAL DEVELOPMENT)

(c) Maharashtra
6. With reference to 'Financial Stability and (d) Uttar Pradesh
Development Council', consider the following
statements : 10. 'SWAYAM', an initiative of the
1. It is an organ of NITI Aayog. Government of India, aims at
2. It is headed by the Union Finance (a) promoting the Self Help Groups in rural
Minister. areas
3. It monitors macroprudential super-vision (b) providing financial and technical
of the economy. assistance to young start-up
Which of the statements given above is/ are entrepreneurs
correct? (c) promoting the education and health of
(a) 1 and 2 only adolescent girls
(b) 3 only (d) providing affordable and quality
(c) 2 and 3 only education to the citizens for free
(d) 1, 2 and 3
11. What is/are the purpose/purposes of
Government's 'Sovereign Gold Bond
7. With reference to 'Agenda 21', sometimes Scheme' and 'Gold Monetization Scheme'?
seen in the news, consider the following
1. To bring the idle gold lying with Indian
statements:
households into the economy
1. It is a global action plan for sustainable
2. To promote FDI in the gold and jewellery
development.
sector
2. It originated in the World Summit on
3. To reduce India's dependence on gold
Sustainable Development held in
imports
Johannesburg in 2002.
Select the correct answer using the code
Which of the statements given above is/ are
given below.
correct?
(a) 1 only
(a) 1 only
(b) 2 and 3 only
(b) 2 only
(c) 1 and 3 only
(c) Both 1 and 2
(d) 1, 2 and 3
(d) Neither 1 nor 2

12. Pradhan Mantri MUDRA Yojana is aimed at


8. The term 'Base Erosion and Profit Shifting' is
(a) bringing the small entrepreneurs into
sometimes seen in the news in the context of
formal financial system
(a) mining operation by multinational
(b) providing loans to poor farmers for
companies in resource-rich but backward
cultivating particular crops
areas
(c) providing pensions to old and destitute
(b) curbing of the tax evasion by
persons
multinational companies
(d) funding the voluntary organizations
(c) exploitation of genetic resources of a
involved in the promotion of skill
country by multinational companies
development and employment generation
(d) lack of consideration of environmental
costs in the planning and implementation
of developmental projects 13. 'Global Financial Stability Report' is prepared
by the
(a) European Central Bank
9. Recently, India's first 'National Investment
and Manufacturing Zone' was proposed to be (b) International Monetary Fund
set up in (c) International Bank for
(a) Andhra Pradesh Reconstruction and Development
(b) Gujarat

11

https://telegram.me/CivilServices_UPSC
https://t.me/UPSC_PDF

INDIAN ECONOMY (ECONOMIC AND SOCIAL DEVELOPMENT)

(d) Organization for Economic Select the correct answer using the code
Cooperation and Development given below.
(a) 1 only
14. Regarding 'Atal Pension Yojana', which of the (b) 2 and 3 only
following statements is/are correct? (c) 1, 2 and 3
1. It is a minimum guaranteed pension (d) 1 and 3 only
scheme mainly targeted at unorganized
sector workers.
18. There has been a persistent deficit budget
2. Only one member of a family can join the year after year. Which action/ actions of the
scheme. following can be taken by the Government to
3. Same amount of pension is guaranteed reduce the deficit?
for the spouse for life after subscriber's 1. Reducing revenue expenditure
death.
2. Introducing new welfare schemes
Select the correct answer using the code
3. Rationalizing subsidies
given below.
4. Reducing import duty
(a) 1 only
Select the correct answer using the code
(b) 2 and 3 only
given below.
(c) 1 and 3 only
(a) 1 only
(d) 1, 2 and 3
(b) 2 and 3 only
(c) 1 and 3 only
15. The term 'Regional Comprehensive
(d) 1, 2, 3 and 4
Economic Partnership' often appears in the
news in the context of the affairs of a group of
countries known as 19. The establishment of 'Payment Banks' is
being allowed in India to promote financial
(a) G20
inclusion. Which of the following statements
(b) ASEAN
is/are correct in this context?
(c) SCO
1. Mobile telephone companies and
(d) SAARC supermarket chains that are owned and
controlled by residents are eligible to be
16. Consider the following statements : promoters of Payment Banks.
1. The Sustainable Development Goals 2. Payment Banks can issue both credit
were first proposed in 1972 by a global cards and debit cards.
think tank called the 'Club of Rome'. 3. Payment Banks cannot undertake
2. The Sustainable Development Goals lending activities.
have to be achieved by 2030. Select the correct answer using the code
Which of the statements given above is/ are given below.
correct? (a) 1 and 2 only
(a) 1 only (b) 1 and 3 only
(b) 2 only (c) 2 only
(c) Both 1 and 2 (d) 1, 2 and 3
(d) Neither 1 nor 2
20. Which one of the following is a purpose of
17. Which of the following is/are the 'UDAY', a scheme of the Government?
indicator/indicators used by IFPRI to compute (a) Providing technical and financial
the Global Hunger Index Report? assistance to start-up entre-preneurs in
1. Undernourishment the field of renewable sources of energy
2. Child stunting (b) Providing electricity to every household in
3. Child mortality the country by 2018

12

https://telegram.me/CivilServices_UPSC
https://t.me/UPSC_PDF

INDIAN ECONOMY (ECONOMIC AND SOCIAL DEVELOPMENT)

(c) Replacing the coal-based power plants (c) Indian Rupee


with natural gas, nuclear, solar, wind and (d) Renminbi
tidal power plants over a period of time
(d) Providing for financial turnaround and 25. With reference to the International Monetary
revival of power distribution companies and Financial Committee (I MFC), consider
the following statements :
21. With reference to 'IFC Masala Bonds', 1. IMFC discusses matters of concern
sometimes seen in the news, which of the affecting the global economy, and
statements given below is/are correct? advises the International Monetary Fund
(1) The International Finance Corporation, (IMF) on the direction of its work.
which offers these bonds, is an arm of 2. The World Bank participates as observer
the World Bank. in IMFC's meetings.
(2) They are the rupee-denominated bonds Which of the statements given above is/ are
and are a source of debt financing for the correct?
public and private sector. (a) 1 only
Select the correct answer using the code (b) 2 only
given below.
(c) Both 1 and 2
(a) 1 only
(d) Neither 1 nor 2
(b) 2 only
(c) Both 1 and 2
26. 'Rashtriya Garima Abhiyaan' is a national
(d) Neither 1 nor 2 campaign to
(a) rehabilitate the homeless and destitute
22. In the context of which of the following do you persons and provide them with suitable
sometimes find the terms 'amber box, blue sources of livelihood
box and green box' in the news? (b) release the sex workers from their
(a) WTO affairs practice and provide them with alternative
(b) SAARC affairs sources of livelihood
(c) UNFCCC affairs (c) eradicate the practice of manual
(d) India-EU negotiations on FTA scavenging and rehabilitate the manual
scavengers
23. Which of the following is/are included in the (d) release the bonded labourers from their
capital budget of the Government of India? bondage and rehabilitate them
1. Expenditure on acquisition of assets like
roads, buildings, machinery, etc. 27. Which of the following best describes the
2. Loans received from foreign governments term 'import cover', sometimes seen in the
news?
3. Loans and advances granted to the
States and Union Territories (a) It is the ratio of value of imports to the
Gross Domestic Product of a country
Select the correct answer using the code
given below. (b) It is the total value of imports of a country
in a year
(a) 1 only
(c) It is the ratio between the value of
(b) 2 and 3 only
exports and that of imports between two
(c) 1 and 3 only
countries
(d) 1, 2 and 3
(d) It is the number of months of imports that
could be paid for by a country's
24. Recently, which one of the following international reserves
currencies has been proposed to be added to
the basket of IMF's SDR?
28. With reference to 'Pradhan Mantri Fasal Bima
(a) Rouble Yojana', consider the following statements :
(b) Rand

13

https://telegram.me/CivilServices_UPSC
https://t.me/UPSC_PDF

INDIAN ECONOMY (ECONOMIC AND SOCIAL DEVELOPMENT)

1. Under this scheme, farmers will have to (a) Controlling the inflation in India is the
pay a uniform premium of two percent for responsibility of the Government of India
any crop they cultivate in any season of only
the year. (b) The Reserve Bank of India has no role in
2. This scheme covers post-harvest losses controlling the inflation
arising out of cyclones and unseasonal (c) Decreased money circulation helps in
rains. controlling the inflation
Which of the statements given above is/ are (d) Increased money circulation helps in
correct? controlling the inflation
(a) 1 only
(b) 2 only 4. The substitution of steel for wooden ploughs
(c) Both 1 and 2 in agriculture production is an example of
(d) Neither 1 nor 2 (a) labour-augmenting technological
progress
(b) capital-augmenting technological
2015 progress
(c) capital-reducing 'technological progress
1. With reference to Indian economy, consider (d) None of the above
the following statements :
1. The rate of growth of Real Gross 5. The problem of International liquidity is
Domestic Product has steadily increased related to the non-availability of
in the last decade.
(a) goods and services
2. The Gross Domestic Product at market
(b) gold and silver
prices (in, rupees) has steadily increased
in the last decade. (c) dollars and other hard currencies
Which of the statements given above is/ are (d) exportable surplus
correct?
(a) 1 only 6. Which one of the following best describes the
(b) 2 only main objective of 'Seed Village Concept'?
(c) Both 1 and 2 (a) Encouraging the farmers to use their own
farm seeds and discouraging them to buy
(d) Neither 1 nor 2
the seeds from others
(b) Involving the farmers for training in
2. With reference to Indian economy, consider quality seed production and thereby to
the following.
make available quality seeds, to others at
1. Bank rate appropriate time and affordable cost
2. Open market operations (c) Earmarking some villages exclusively for
3. Public debt the production of certified seeds
4. Public revenue (d) Identifying the villages and technology
Which of the above is/are component/ and seed companies entrepreneurs in
components of Monetary Policy? providing them finance to set up
(a) 1 only
(b) 2, 3 and 4 7. There has been a persistent deficit budget
(c) 1 and 2 year after year. Which of the following actions
(d) 1, 3 and 4 can be taken by the government to reduce
the deficit?
1. Reducing revenue expenditure
3. With reference to inflation in India, which of
2. Introducing new welfare schemes
me following statements is correct?
3. Rationalizing subsidies
4. Expanding industries

14

https://telegram.me/CivilServices_UPSC
https://t.me/UPSC_PDF

INDIAN ECONOMY (ECONOMIC AND SOCIAL DEVELOPMENT)

Select the correct answer using the code 1. Slowing economic growth rate
given below. 2. Less equitable distribution of national
(a) 1 and 3 only income
(b) 2 and 3 only Select the correct answer using the code
(c) 1 only given below.
(d) 1, 2, 3 and 4 (a) 1 only
(b) 2 only
8. With reference to the Indian Renewable (c) Both 1 and 2
Energy Development Agency Limited (I (d) Neither 1 nor 2
REDA), which of the following statements
is/are correct 12. In the 'Index of Eight Core Industries', which
1. It is a Public Limited Government one of the following is given the highest
Company. weight?
2. It is a Non-Banking Financial Company. (a) Coal production
Select the correct answer using the code (b) Electricity generation
given below (c) Fertilizer production
(a) 1 only (d) Steel production
(b) 2 only
(c) Both 1 and 2 13. The Fair and Remunerative Price (FRP) of
(d) Neither 1 nor 2 sugarcane is approved by the
(a) Cabinet Committee on Economic Affairs.
9. 'Pradhan Mantri Jan-Dhan Yojana' has been (b) Commission for 'Agricultural Costs and
launched for Prices
(a) providing housing loan to poor people at (c) Directorate of Marketing and Inspection,
cheaper interest rates Ministry of Agriculture
(b) promoting women's Self-Help Groups in (d) Agricultural Produce Market Committee
backward areas
(c) promoting financial inclusion in the 14. Which one of the following issues the 'Global
country Economic Prospects' report periodically?
(d) providing financial help to the (a) The Asian Development Bank
marginalized communities
(b) The European Bank for
Reconstruction and Development
10. With reference to the Fourteenth Finance (c) The US Federal Reserve Bank
Commission, which of the following
(d) The World Bank
statements is/are correct?
1. It has increased the share of States in the
15. When the Reserve Bank of India reduces the
central divisible pool from 32 percent to
Statutory Liquidity Ratio by 50 basis points,
42 percent.
which of the following is likely to happen.?
2. It has made recommendations
(a) India's GDP growth rate increases
concerning sector-specific grants.
drastically
Select the correct answer using the code
(b) Foreign Institutional Investors may bring
given below.
more capital into our country
(a) 1 only
(c) Scheduled Commercial Banks may cut
(b) 2 only
their lending rates
(c) Both 1 and 2
(d) It may drastically reduce the liquidity to
(d) Neither 1 nor 2 the banking system

11. A decrease in tax to GDP ratio of a country


indicates which of the following?

15

https://telegram.me/CivilServices_UPSC
https://t.me/UPSC_PDF

INDIAN ECONOMY (ECONOMIC AND SOCIAL DEVELOPMENT)

16. In India, markets in agricultural products are


2014
regulated under the
(a) Essential Commodities Act, 1955
(b) Agricultural Produce Market Committee 1. The sales tax you pay while purchasing a
Act enacted by States toothpaste is a
(a) tax imposed by the Central Government
(c) Agricultural Produce (Grading and
Marking) Act, 1937 (b) tax imposed by the Central Government
but collected by the State Government
(d) Food Products Order, 1956 and Meat
and Food Products Order, 1973 (c) tax imposed by the State Government but
collected by the Central Government

17. Which of the following brings 'out: the (d) tax imposed and collected by the State
Government
'Consumer Price Index Number for Industrial
Workers?
(a) The Reserve Bank of India 2. What does venture capital mean?
(b) The Department of Economic Affairs (a) A short-term capital provided to industries
(c) The Labour Bureau (b) A long-term start-up capital provided to
new entrepreneurs
(d) The Department of Personnel and
Training (c) Funds provided to industries at times of
incurring losses

18. The terms 'Agreement on Agriculture', (d) Funds provided for replacement
renovation of industries
'Agreement on the Application of Sanitary
and Phytosanitary Measures' and 'Peace
th
Clauee' appear in the news frequently in the 3. The main objective of the 12 Five-Year Plan
context of the affairs of the is
(a) Food and Agriculture Organization (a) inclusive growth and poverty reduction
(b) United Nations Framework conference on (b) inclusive and sustainable growth
Climate Change (c) sustainable and inclusive growth to
(c) World Trade Organization reduce unemployment
(d) United Nations Environment Programme (d) faster, sustainable and more inclusive
growth
19. Convertibility of rupee implies
(a) being able to convert rupee notes into 4. With reference to Balance of Payments,
gold which of the following constitutes/ constitute
the Current Account?
(b) allowing the value of rupee to be fixed by
market forces 1. Balance of trade
(c) freely permitting the conversion of rupee 2. Foreign assets
to other currencies and vice versa 3. Balance of invisibles
(d) developing an international market for 4. Special Drawing Rights
currencies in India Select the correct answer using the code
given below.
20. The Government of India has established (a) 1 only
NITI Aayog to replace the (b) 2 and 3
(a) Human Rights Commission (c) 1 and 3
(b) Finance Commission (d) 1, 2 and 4
(c) Law Commission
(d) Planning Commission 5. The terms 'Marginal Standing Facility Rate'
and 'Net Demand and Time Liabilities',
sometimes appearing in news, are used in
relation to

16

https://telegram.me/CivilServices_UPSC
https://t.me/UPSC_PDF

INDIAN ECONOMY (ECONOMIC AND SOCIAL DEVELOPMENT)

(a) banking operations (d) The World Bank


(b) communication networking
(c) military strategies 9. With reference to Union Budget, which of the
(d) supply and demand of agricultural following is/are covered under Non-Plan
products Expenditure?
1. Defence expenditure
6. What is/are the facility/facilities the 2. Interest payments
beneficiaries can get from the services of 3. Salaries and pensions
Business Correspondent (Bank Saathi) in 4. Subsidies
branchless areas? Select the correct answer using the code
1. It enables the beneficiaries to draw their given below.
subsidies and social security benefits in (a) 1 only
their villages.
(b) 2 and 3 only
2. It enables the beneficiaries in the rural
(c) 1, 2, 3 and 4
areas to make deposits and withdrawals.
(d) None
Select the correct answer using the code
given below.
10. If the interest rate is decreased in an
(a) 1 only
economy, it will
(b) 2 only
(a) decrease the consumption expenditure
(c) Both 1 and 2
in the economy
(d) Neither 1 nor 2
(b) increase the tax collection of the
Government
7. In the context of Indian economy, which of (c) increase the investment expenditure
the following is/are the purpose/ purposes of in the economy
'Statutory Reserve Requirements'?
(d) increase the total savings in the economy
1. To enable the Central Bank to control the
amount of advances the banks can
create 2013
2. To make the people's deposits with
banks safe and liquid
1. 'Economic Justice' as one of the objectives of
3. To prevent the commercial banks from the Indian Constitution has been provided in
making excessive profits
(a) the Preamble and the Fundamental
4. To force the banks to have sufficient vault Rights
cash to meet their day-to-day
(b) the Preamble and the Directive Principles
requirements
of State Policy
Select the correct answer using the code
(c) the Fundamental Rights and the Directive
given below.
Principles of State Policy
(a) 1 only
(d) None of the above
(b) 1 and 2 only
(c) 2 and 3 only
2. The balance of payments of a country is a
(d) 1, 2, 3 and 4 systematic record of
(a) all import and export transactions of a
8. Which of the following organizations brings country during a given period of time,
out the publication known as 'World normally a year
Economic Outlook'? (b) goods exported from a country during a
(a) The International Monetary Fund year
(b) The United Nations Development (c) economic transaction between the
Programme government of one country to another
(c) The World Economic Forum

17

https://telegram.me/CivilServices_UPSC
https://t.me/UPSC_PDF

INDIAN ECONOMY (ECONOMIC AND SOCIAL DEVELOPMENT)

(d) capital movements from one country to (d) 1, 2 and 3


another
7. The Government enacted the Panchayat
3. The Reserve Bank of India regulates the Extension to Scheduled Areas (PESA) Act in
commercial banks in matters of 1996. Which one of the following is not
1. liquidity of assets identified as its objective?
2. branch expansion (a) To provide self-governance
3. merger of banks (b) To recognize traditional rights
4. winding-up of banks (c) To create autonomous regions in tribal
Select the correct answer using the codes areas
given below : (d) To free tribal people from exploitation
(a) 1 and 4 only
(b) 2, 3 and 4 only 8. Priority Sector Lending by banks in India
(c) 1, 2 and 3 only constitutes the lending to
(d) 1, 2, 3 and 4 (a) agriculture
(b) micro and small enterprises
4. An increase in the Bank Rate generally (c) weaker sections
indicates that the (d) All of the above
(a) market rate of interest is likely to fall
(b) Central Bank is no longer making loans 9. Which one among the following industries is
to commercial banks the maximum consumer of water in India?
(c) Central Bank is following an easy money (a) Engineering
policy (b) Paper and pulp
(d) Central Bank is following a tight money (c) Textiles
policy (d) Thermal power

5. In India, deficit financing is used for raising 10. To obtain full benefits of demographic
resources for dividend, what should India do?
(a) economic development (a) Promoting skill development
(b) redemption of public debt (b) Introducing more social security schemes
(c) adjusting the balance of payments (c) Reducing infant mortality rate
(d) reducing the foreign debt (d) Privatization of higher education

6. With reference to the usefulness of the by- 11. A rise in general level of prices may be
products of sugar industry, which of the caused by
following statements is/are correct? 1. an increase in the money supply
1. Bagasse can be used as biomass fuel for 2. a decrease in the aggregate level of
the generation of energy. output
2. Molasses can be used as one of the 3. an increase in the effective demand
feedstock's for the production of synthetic
Select the correct answer using the codes
chemical fertilizers.
given below:
3. Molasses can be used for the production
(a) 1 only
of ethanol,
(b) 1 and 2 only
Select the correct answer using the codes
(c) 2 and 3 only
given below.
(d) 1, 2 and 3
(a) 1 only
(b) 2 and 3 only
(c) 1 and 3 only

18

https://telegram.me/CivilServices_UPSC
https://t.me/UPSC_PDF

https://telegram.me/CivilServices_UPSC
https://t.me/UPSC_PDF

https://telegram.me/CivilServices_UPSC
https://t.me/UPSC_PDF

INDIAN ECONOMY (ECONOMIC AND SOCIAL DEVELOPMENT)

(a) 2 and 3 only Select the correct answer using the codes
(b) 1 and 2 only given below :
(c) 1 and 3 only (a) 1, 2, 3 and 4
(d) 1, 2 and 3 (b) 2 and 4 only
(c) 1 and 3 only
5. Under which of the following circumstances (d) 1, 2 and 3 only
may 'capital gains' arise?
1. When there is an increase in the sales of 8. Consider the following statements :
a product The price of any currency in international
2. When there is a natural increase in the market is decided by the
value of the property owned 1. World Bank
3. When you purchase a painting and there 2. demand for goods/services provided by
is a growth in its value due to increase in the country concerned
its popularity. 3. stability of the government of the
Select the correct answer using the codes concerned country
given below : 4. economic potential of the country in
(a) 1 only question
(b) 2 and 3 only Which of the statements given above are
(c) 2 only correct?
(d) 1, 2 and 3 (a) 1, 2, 3 and 4
(b) 2 and 3 only
6. Which of the following measures would result (c) 3 and 4 only
in an increase in the money supply in the (d) 1 and 4 only
economy?
1. Purchase of government securities from 9. The basic aim of Lead Bank Scheme is that
the public by the Central Bank
(a) big banks should try to open offices in
2. Deposit of currency in commercial banks each district
by the public
(b) there should be stiff competition among
3. Borrowing by the government from the the various nationalized banks
Central Bank
(c) individual banks should adopt particular
4. Sale of government securities to the districts for intensive development
public by the Central Bank
(d) all the banks should make intensive
Select the correct answer using the codes efforts to mobilize deposits
given below :
(a) 1 only
10. The Multi-dimensional Poverty Index
(b) 2 and 4 only developed by Oxford Poverty and Human
(c) 1 and 3 Development Initiative with UNDP support
(d) 2, 3 and 4 covers which of the following?
1. Deprivation of education, health, assets
7. Which of the following would include Foreign and services at household level
Direct Investment in India? 2. Purchasing power parity at national level
1. Subsidiaries of foreign companies in 3. Extent of budget deficit and GDP growth
India rate at national level
2. Majority foreign equity holding in Indian Select the correct answer using the codes
companies given below :
3. Companies exclusively financed by (a) 1 only
foreign companies (b) 2 and 3 only
4. Portfolio investment (c) 1 and 3 only

21

https://telegram.me/CivilServices_UPSC
https://t.me/UPSC_PDF

INDIAN ECONOMY (ECONOMIC AND SOCIAL DEVELOPMENT)

(d) 1, 2 and 3 14. In India, in the overall Index of Industrial


Production, the Indices of Eight Core
11. Which of the following is/are among the Industries have a combined weight of
noticeable features of the recommendations 37.90%.
of the Thirteenth Finance Commission? Which of the following are among those Eight
1. A design for the Goods and Services Core Industries?
Tax, and a compensation package linked 1. Cement
to adherence to the proposed design 2. Fertilizers
2. A design for the creation of lakhs of jobs 3. Natural gas
in the next ten years in consonance with 4. Refinety products
India's demographic dividend 5. Textiles
3. Devolution of a specified share of central Select the correct answer using the codes
taxes to local bodies as grants given below :
Select the correct answer using the codes (a) 1 and 5 only
given below :
(b) 2, 3 and 4 only
(a) 1 only
(c) 1, 2, 3 and 4 only
(b) 2 and 3 only
(d) 1, 2, 3, 4 and 5
(c) 1 and 3 only
(d) 1, 2 and 3
2011
12. What is/are the recent policy initiative(s) of
Government of India to promote the growth of 1. India has experienced persistent and high
manufacturing sector? food inflation in the recent past. What could
1. Setting up of National Investment and be the reasons ?
Manufacturing Zones 1. Due to a gradual switchover to the
2. Providing the benefit of 'single window cultivation of commercial crops, the area
clearance' under the cultivation of food grains has
3. Establishing the Technology Acquisition steadily decreased in the last five years
and Development Fund by about 30%
Select the correct answer using the codes 2. As a consequence of increasing incomes,
given below : the consumption patterns of the people
have undergone a significant change.
(a) 1 only
(b) 2 and 3 only 3. The food supply chain has structural
constraints.
(c) 1 and 3 only
Which of the statements given above are
(d) 1, 2 and 3
correct?
(a) 1 and 2 only
13. Consider the following specific stages of
(b) 2 and 3 only
demographic transition associated with
economic development : (c) 1 and 3 only

1. Low birthrate with low death rate (d) 1, 2 and 3

2. High birthrate with high death rate


3. High birthrate with low death rate 2. In terms of economy, the visit by foreign
nationals to witness the XIX Common Wealth
Select the correct order of the above stages
Games in India amounted to
using the codes given below:
(a) Export
(a) 1, 2, 3
(b) Import
(b) 2, 1, 3
(c) Production
(c) 2, 3, 1
(d) Consumption
(d) 3, 2, 1

22

https://telegram.me/CivilServices_UPSC
https://t.me/UPSC_PDF

INDIAN ECONOMY (ECONOMIC AND SOCIAL DEVELOPMENT)

3. Which one of the following statements (b) It is a tax levied on value additoin at each
appropriately describes the "fiscal stimulus"? stage of transaction in production-
(a) It is a massive investment by the distribution chain
Government in manufacturing sector to (c) It is a tax on the final consumption of
ensure the supply of goods to meet the goods or services and must ultimately be
demand surge caused by rapid economic borne by the consumer
growth (d) It is basically a subject of the Central
(b) It is an intense affirmative action of the Government and the State Governments
Government to boost economic activity in are only a facilitator for its successful
the country implementation
(c) It is Government's intensive action on
financial institutions to ensure 7. A "closed economy" is an economy is which
disbursement of loans to agriculture and (a) the money supply is fully controlled
allied sectors to promote greater food
(b) deficit financing takes place
production and contain food inflation
(c) only exports take place
(d) It is an extreme affirmative action by the
(d) neither exports nor imports take place.
Government to pursue its policy of
financial inclusion
8. Both Foreign Direct Investment (FDI) and
Foreign Institutional Investor (FII) are related
4. Consider the following actions which the
to investment in a country. Which one of the
Government can take:
following statements best represents an
1. Devaluing the domestic currency.
important difference between the two ?
2. Reduction in the export subsidy.
(a) FII helps bring better management skills
3. Adopting suitable policies which attract and technology, while FDI only brings in
greater FDI and more funds from FIIs. capital
Which of the above action/actions can help in (b) FII helps in increasing capital availability
reducing the current account deficit? in general, while FDI only targets specific
(a) 1 and 2 sectors
(b) 2 and 3 (c) FDI flows only into the secondary market,
(c) 3 only while FII targets primary market
(d) 1 and 3 (d) FII is considered to be more stable than
FDI
5. A rapid increase in the rate of inflation is
sometimes attribute to the "bass effect". What 9. With reference to "Aam Admi Bima Yojana",
is "base effect"? consider the following statements:
(a) It is the impact of drastic deficiency in 1. The member insured under the scheme
supply due to failure of crops must be the head of the family or an
(b) It is the impact of the surge in demand earning member of the family in a rural
due to rapid economic growth landless household.
(c) It is the impact of the price levels of 2. The member insured must be in the age
previous year on the calculation of group of 30 to 65 years.
inflation rate 3. There is a provision for free scholarship
(d) None of the statements (a), (b) and (c) for up to two children of the insurred who
given above is correct in this context are studying between classes 9 and 12.
Which of the statements given above is/ are
6. Which one of the following is not a feature of correct?
"Value Added Tax"? (a) 1 only
(a) It is a mutli-point destination-based (b) 2 and 3 only
system of taxation (c) 1 and 3 only
(d) 1, 2 and 3

23

https://telegram.me/CivilServices_UPSC
https://t.me/UPSC_PDF

INDIAN ECONOMY (ECONOMIC AND SOCIAL DEVELOPMENT)

(a) The President of India


10. In the context of global oil prices, "Brent (b) The Parliament of India
crude oil" is frequently referred to in the (c) The Prime Minister of India
news. What does this term imply ? (d) The Union Finance Minister
1. It is a major classification of crude oil.
2. It is sourced from North Sea. 14. All revenues received by the Union
3. It does not contain sulphur. Government by way of taxes and other
Which of the statements given above is/ are receipts for the conduct of Government
correct? business are credited to the
(a) 2 only (a) Contingency Fund of India
(b) 1 and 2 only (b) Public Account
(c) 1 and 3 only (c) Consolidated Fund of India
(d) 1, 2 and 3 (d) Deposits and Advances Fund

11. India is home to lakhs of persons with 15. Microfinance is the provision of financial
disabilities. What are the benefits available to services to people of low-income groups.
them under the law ? This includes both the consumers and the
1. Free schooling till the age of 18 years in self-employed. The service/services rendered
government-run schools. under microfinance is/are :
2. Preferential allotment of land for setting 1. Credit facilities
up business. 2. Savings facilities
3. Ramps in public buildings. 3. Insurance facilities
Which of the statements given above is/ are 4. Fund Transfer facilities
correct? Select the correct answer using the codes
(a) 1 only gives below the lists:
(b) 2 and 3 only (a) 1 only
(c) 1 and 3 only (b) 1 and 4 only
(d) 1, 2 and 3 (c) 2 and 3 only
(d) 1, 2, 3 and 4
12. With what purpose is the Government of
India promoting the concept of "Mega Food 16. Among the following who are eligible to
Parks"? benefit from the "Mahatma Gandhi National
1. To provide good infrastructure facilities Rural Employment Guarantee Act"?
for the food processing industry. (a) Adult members of only the scheduled
2. To increase the processing of perishable caste and scheduled tribe households.
items and reduce wastage. (b) Adult members of below poverty line
3. To provide emerging and eco-friendly (BPL) households.
food processing technologies to (c) Adult members of households of all
entrepreneurs. backward communities.
Select the correct answer using the codes (d) Adult members of any household
given below :
(a) 1 only 17. With reference to the Finance Commission of
(b) 1 and 2 only India, which of the following statements is
(c) 2 and 3 only correct?
(d) 1, 2 and 3 (a) It encourages the inflow of foreign capital
for infrastructure development.
13. The authorization for the withdrawal of funds (b) It facilitates the proper distribution of
from the Consolidated Fund of India must finances among the Public Sector
come from Undertakings.

24

https://telegram.me/CivilServices_UPSC
https://t.me/UPSC_PDF

INDIAN ECONOMY (ECONOMIC AND SOCIAL DEVELOPMENT)

(c) It ensures transparency in financial 21. In the context of Indian economy, consider
administration. the following statements :
(d) None of the statements (a), (b) and (c) 1. The growth rate of GDP has steadily
given above is correct in this context. increased in the last five years.
2. The growth rate in per capita income has
18. What is the difference between "vote on steadily increased in the last five years.
account" and "interim budget"? Which of the statements given above is/ are
1. The provision of a "vote-on-account" is correct?
used by a regular Government, while an (a) 1 only
"interim budget" is a provision used by a (b) 2 only
caretaker Government. (c) Both 1 and 2
2. A "vote-on-account" only deals with the (d) Neither 1 nor 2
expenditure in Government's budget,
while an "interim budget" includes both
22. In India, which of the following have the
expenditure and receipts.
highest share in the disbursement of credit to
Which of the statements given above is/ are
agriculture and allied activities?
correct?
(a) Commercial Banks
(a) 1 only
(b) Cooperative Banks
(b) 2 only
(c) Regional Rural Banks
(c) Both 1 and 2
(d) Microfinance Institutions
(d) Neither 1 nor 2

23. Which of the following can aid in furthering


19. Regarding the International Monetary Fund,
the Government's objective of inclusive
which one of the following statements is
growth?
correct ?
1. Promoting Self-Help Groups.
(a) It can grant loans to any country.
2. Promoting Micro, Small and Medium
(b) It can grant loans to only developed
Enterprises.
countries.
3. Implementing the Right to Education Act
(c) It grants loans to only member countries.
Select the correct answer using the codes
(d) It can grant loans to the central bank of a
given below :
country.
(a) 1 only
(b) 1 and 2 only
20. Why is the offering of "teaser loans" by
(c) 2 and 3 only
commercial banks a cause of economic
concern ? (d) 1, 2 and 3
1. The teaser loans are considered to be an
aspect of sub-prime lending and banks 24. Why is the Government of India disinvesting
may be exposed to the risk of defaulters its equity in the Central Public Sector
in future. Enterprises (CPSEs) ?
2. In India, the teaser loans are mostly 1. The Government intends to use the
given to inexperienced entrepreneurs to revenue earned from the disinvestment
set up manufacturing or export units? mainly to pay back the external debt.
Which of the statements given above is/ are 2. The Government no longer intends to
correct? retain the management control of the
(a) 1 only CPSEs.
(b) 2 only Which of the statements given above is/ are
correct?
(c) Both 1 and 2
(a) 1 only
(d) Neither 1 nor 2
(b) 2 only
(c) Both 1 and 2

25

https://telegram.me/CivilServices_UPSC
https://t.me/UPSC_PDF

INDIAN ECONOMY (ECONOMIC AND SOCIAL DEVELOPMENT)

(d) Neither 1 nor 2 Geographical Indication is assigned to


the agricultural goods/products and
25. Economic growth is usually coupled with handicrafts only.
(a) Deflation Which of the statements given above is/ are
correct?
(b) Inflation
(a) 1 only
(c) Stagflation
(b) 1 and 2 only
(d) Hyperinflation
(c) 2 and 3 only
(d) 1, 2 and 3
26. The lowering of Bank Rate by the Reserve
Bank of India leads to
(a) More liquidity in the market 4. The SEZ Act, 2005 which came into effect in
February 2006 has certain objectives. In this
(b) Less liquidity in the market
context, consider the following :
(c) No change in the liquidity in the market
1. Development of infrastructure facilities.
(d) Mobilization of more deposits by
2. Promotion of investment from foreign
commercial banks
sources.
3. Promotion of exports of services only.
2010 Which of the above are the objectives of this
1. In the context of the affairs of which of the Act?
following is the phrase "Special Safeguard (a) 1 and 2 only
Mechanisms" mentioned in the news (b) 3 only
frequently? (c) 2 and 3 only
(a) United Nations Environment (d) 1, 2 and 3
Programme
(b) World Trade Organisation
5. Which one of the following statements is an
(c) ASEAN-India Free Trade Agreement appropriate description of deflation?
(d) G-20 Summits (a) It is a sudden fall in the value of a
currency against other currencies
2. Which of the following terms indicates a (b) It is a persistent recession in both the
mechanism used by commercial banks for financial and real sectors of economy
providing credit to the government? (c) It is a persistent fall in the general price
(a) Cash Credit Ratio level of goods and services
(b) Debt Service Obligation (d) It is a fall in the rate of inflation over a
(c) Liquidity Adjustment Facility period of time
(d) Statutory Liquidity Ratio
6. With reference to the Non-banking Financial
Companies (NBFCs) in India, consider the
3. In order to comply with TRI PS Agreement, following statements :
India enacted the Geographical Indications of
1. They cannot engage in the acquisition of
Goods (Registration & Protection) Act, 1999.
securities issued by the government.
The difference/differences between a "Trade
Mark" and a Geographical Indication is/ are: 2. They cannot accept demand deposits like
Savings Account.
1. A Trade Mark is an individual or a
company's right whereas a Geographical Which of the statements given above is/ are
Indication is a community's right. correct ?
2. A Trade Mark can be licensed whereas a (a) 1 only
Geographical Indication cannot be (b) 2 only
licensed. (c) Both 1 and 2
3. A Trade Mark is assigned to the (d) Neither 1 nor 2
manufactured goods whereas the

26

https://telegram.me/CivilServices_UPSC
https://t.me/UPSC_PDF

INDIAN ECONOMY (ECONOMIC AND SOCIAL DEVELOPMENT)

7. Which one of the following was not stipulated (a) 1 only


in the Fiscal Responsibility and Budget (b) 2 and 3 only
Management Act, 2003 ? (c) 1 and 3 only
(a) Elimination of revenue deficit by the end (d) 1, 2 and 3
of the fiscal year 2007-08
(b) Non-borrowing by the central government
11. With reference to BRIC countries, consider
from Reserve Bank of India except under
the following statements :
certain circumstances
1. At present, China's GDP is more than the
(c) Elimination of primary deficit by the end
combined GDP of all the three other
of the fiscal year 2008-09
countries.
(d) Fixing government guarantees in any
2. China's population is more than the
financial year as a percentage of GDP
combined population of any two other
countries.
8. In the parlance of financial investments, the Which of the statements given above is/ are
term 'bear' denotes correct ?
(a) An investor who feels that the price of a (a) 1 only
particular security is going to fall
(b) 2 only
(b) An investor who expects the price of
(c) Both 1 and 2
particular shares to rise
(d) Neither 1 nor 2
(c) A shareholder or a bondholder who has
an interest in a company, financial or
otherwise 12. Which of the following is/are treated as
artificial currency ?
(d) Any lender whether by making a loan or
buying a bond (a) ADR
(b) GDR
9. A great deal of Foreign Direct Investment (c) SDR
(FDI) to India comes from Mauritius than from (d) Both ADR and SDR
many major and mature economies like UK
and France. Why? 13. In India, the interest rate on savings accounts
(a) India has preference for certain countries in all the nationalized commercial banks is
as regards receiving FDI fixed by
(b) India has double taxation avoidance (a) Union Ministry of Finance
agreement with Mauritius (b) Union Finance Commission
(c) Most citizens of Mauritius have ethnic (c) Indian Banks' Association
identity with India and so they feel secure (d) None of the above
to invest in India
(d) Impending dangers of global climatic
14. Inclusive growth as enunciated in the
change prompt Mauritius to make huge
Eleventh Five Year Plan does not include one
investments in India
of the following :
(a) Reduction of poverty
10. In the context of Indian economy, consider
(b) Extension of employment opportunities
the following pairs :
(c) Strengthening of capital market
Term Most appropriate
(d) Reduction of gender inequality
description
1. Melt down : Fall in stock prices
15. Consider the following actions by the
2. Recession : Fall in growth rate Government:
3. Slow down : Fall in GDP 1. Cutting the tax rates
Which of the pairs given above is/are 2. Increasing the government spending
correctly matched ? 3. Abolishing the subsidies

27

https://telegram.me/CivilServices_UPSC
https://t.me/UPSC_PDF

INDIAN ECONOMY (ECONOMIC AND SOCIAL DEVELOPMENT)

In the context of economic recession, which (c) Internal governance may be decided by
of the above actions can be considered a part mutual agreement among partners
of the "fiscal stimulus" package ? (d) It is corporate body with perpetual
(a) 1 and 2 only succession
(b) 2 only
(c) 1 and 3 only 20. With reference to the institution of Banking
(d) 1, 2 and 3 Ombudsman in India, which one of the
statements is not correct ?
16. When the Reserve Bank of India announces (a) The Banking Ombudsman is appointed
an increase of the Cash Reserve Ratio, what by the Reserve Bank of India
does it mean ? (b) The Banking Ombudsman can consider
(a) The commercial banks will have less complaints from NonResident Indians
money to lend having accounts in India
(b) The Reserve Bank of India will have less (c) The orders passed by the Banking
money to lend Ombudsman are final and binding on the
parties concerned
(c) The Union Government will have less
money to lend (d) The service provided by the Banking
Ombudsman is free of any fee
(d) The commercial banks will have more
money to lend
21. Consider the following statements :
17. With reference to India, consider the following The functions of commercial banks in India
statements : include
1. The Wholesale Price Index (WPI) in India 1. Purchase and sale of shares and
is available on a monthly basis only. securities on behalf of customers.
2. As compared to Consumer Price Index 2. Acting as executors and trustees of wills.
for Industrial Workers (CPI(IW)), the WPI Which of the statements given above is/ are
gives less weight to food articles. correct ?
Which of the statements given above is/are (a) 1 only
correct ? (b) 2 only
(a) 1 only (c) Both 1 and 2
(b) 2 only (d) Neither 1 nor 2
(c) Both 1 and 2
(d) Neither, 1 nor 2 22. In India, the tax proceeds of which one of the
following as a percentage of gross tax
18. In the context of India's Five Year Plans, a revenue has significantly declined in the last
shift in the pattern of industrialization, with five years ?
lower emphasis on heavy industries and (a) Service tax
more on infrastructure begins in (b) Personal income tax
(a) Fourth Plan (c) Excise duty
(b) Sixth Plan (d) Corporation tax
(c) Eighth Plan
(d) Tenth Plan 23. Consider the following statements :
In India, taxes on transactions in Stock
19. Which one of the following is not a feature of Exchanges and Futures Markets are
Limited Liability Partnership firm? 1. levied by the Union
(a) Partners should be less than 20 2. collected by the State
(b) Partnership and. management need not Which of the statements given above is /are
be separate correct ?
(a) 1 only

28

https://telegram.me/CivilServices_UPSC
https://t.me/UPSC_PDF

INDIAN ECONOMY (ECONOMIC AND SOCIAL DEVELOPMENT)

(b) 2 only (c) Both 1 and 2


(c) Both 1 and 2 (d) Neither 1 nor 2
(d) Neither 1 nor 2
25. Sustainable development is described as the
24. With reference to Indian economy, consider development that meets the needs of the
the following statements: present without compromising the ability of
1. The Gross Domestic Product (GDP) has future generations to meet their own needs.
increased by four times in the last 10 In this perspective, inherently the concept of
years. sustainable development is intertwined with
which of the following concepts?
2. The percentage share of Public Sector in
GDP has declined in the last 10 years. (a) Social justice and empowerment
Which of the statements given above is/ are (b) Inclusive Growth
correct? (c) Globalization
(a) 1 only (d) Carrying capacity
(b) 2 only

29

https://telegram.me/CivilServices_UPSC
https://t.me/UPSC_PDF

ANSWERS - INDIAN ECONOMY (ECONOMIC AND SOCIAL DEVELOPMENT)

ANSWERS OF INDIAN ECONOMY

2019 8. (b) 2, 4, 5 and 6 only


9. (c) Both 1 and 2
10. (a) 1 only
1. (c) NITI Aayog
11. (c) 1 and 3 only
2. (c) World Economic Forum
12. (c) The charge to a merchant by a bank
3. (b) price levels vary from State to State
for accepting payments from his
4. (a) 1 only customers through the bank’s debit
5. (d) Vegetable oils cards.
6. (d) Following an expansionary monetary 13. (a) 1 only
policy 14. (c) 1 and 3 only
7. (a) 1 only 15. (c) National Payments Corporation of
8. (b) Increase in the banking habit of the India
population 16. (b) 3 only
9. (b) Lead Bank Scheme 17. (a) 1 only
10. (d) Neither 1 nor 2 18. (a) 1 only
11. (b) Deposits 19. (a) 1 only
12. (b) 1 and 3 only
13. (d) Participatory Note
14. (a) 1 and 2 2017
15. (a) 1 and 2 only
16. (c) Both 1 and 2 1. (a) 1 only
17. (a) 1 only 2. (c) All State Government employees
18. (d) To aim at faster resolution of joining the services after the date of
stressed assets of Rs 50 crore or notification by the respective State
more which are under consortium Governments
19. (a) Banks Board Bureau 3. (a) 1 only
20. (a) Maintenance of law and order 4. (c) Both 1 and 2
21. (c) procurement incidentals and 5. (c) Both 1 and 2
distribution cost 6. (a) 1 and 2 only
22. (d) The level of mutual trust and 7. (a) Mobile wallets will not be necessary
harmony in the society for online payments.
8. (d) Neither 1 nor 2
2018 9. (b) World Bank collaboration that
facilitates the preparation and
structuring of complex infrastructure
1. (c) 2 and 3 only Public-Private Partnerships (PPPs)
2. (b) The money which a creditor is under to enable mobilization of private
compulsion to accept in settlement of sector and institutional investor
his claims capital.
3. (b/d) the opportunity cost is ignored. 10. (b) 2 only
the opportunity cost is transferred 11. (b) 3 only
from the consumers of the product to 12. (b) 2, 3 and 4 only
the Government.
13. (c) Both 1 and 2
4. (c) poverty and unemployment increase.
14. (a) 1 only
5. (c) 2 and 4
15. (d) Neither 1 nor 2
6. (d) high capital-output ratio
7. (c) 2 and 3 only

30

https://telegram.me/CivilServices_UPSC
https://t.me/UPSC_PDF

ANSWERS - INDIAN ECONOMY (ECONOMIC AND SOCIAL DEVELOPMENT)

2016 6. (b) Involving the farmers for training in


quality seed production and thereby
to make available quality seeds, to
1. (c) Both 1 and 2 others at appropriate time and
2. (a) 1 only affordable cost
3. (c) World Bank 7. (a) 1 and 3 only
4. (c) Both 1 and 2 8. (c) Both 1 and 2
5. (a) 1 only 9. (c) promoting financial inclusion in the
6. (c) 2 and 3 only country
7. (a) 1 only 10. (a) 1 only
8. (b) curbing of the tax evasion by 11. (b) 2 only
multinational companies 12. (b) Electricity generation
9. (a) Andhra Pradesh 13. (a) Cabinet Committee on Economic
10. (d) providing affordable and quality Affairs.
education to the citizens for free 14. (d) The World Bank
11. (c) 1 and 3 only 15. (c) Scheduled Commercial Banks may
12. (a) bringing the small entrepreneurs into cut their lending rates
formal financial system 16. (b) Agricultural Produce Market
13. (b) International Monetary Fund Committee Act enacted by States
14. (c) 1 and 3 only 17. (c) The Labour Bureau
15. (b) ASEAN 18. (c) World Trade Organization
16. (b) 2 only 19. (c) freely permitting the conversion of
17. (c) 1, 2 and 3 rupee to other currencies and vice
versa
18. (c) 1 and 3 only
20. (d) Planning Commission
19. (b) 1 and 3 only
20. (d) Providing for financial turnaround
and revival of power distribution 2014
companies
21. (a) 1 only 1 (d) tax imposed and collected by the
22. (a) WTO affairs State Government
23. (d) 1, 2 and 3 2 (b) A long-term start-up capital provided
to new entrepreneurs
24. (d) Renminbi
3 (d) faster, sustainable and more
25. (c) Both 1 and 2
inclusive growth
26. (c) eradicate the practice of manual
4 (c) 1 and 3
scavenging and rehabilitate the
manual scavengers 5 (a) banking operations
27. (d) It is the number of months of imports 6 (c) Both 1 and 2
that could be paid for by a country’s 7 (b) 1 and 2 only
international reserves 8 (a) The International Monetary Fund
28. (b) 2 only 9 (c) 1, 2, 3 and 4
10 (c) increase the investment expenditure
2015 in the economy

1 (c) Both 1 and 2 2013


2. (c) 1 and 2
3. (c) Decreased money circulation helps 1 (b) the Preamble and the Directive
in controlling the inflation Principles of State Policy
4. (b) capital-augmenting technological 2 (a) all import and export transactions of
progress a country during a given period of
5. (c) dollars and other hard currencies time, normally a year

31

https://telegram.me/CivilServices_UPSC
https://t.me/UPSC_PDF

ANSWERS - INDIAN ECONOMY (ECONOMIC AND SOCIAL DEVELOPMENT)

3 (d) 1, 2, 3 and 4 2. (a) Export


4 (d) Central Bank is following a tight 3. (b) It is an intense affirmative action of
money policy the Government to boost economic
5 (a) economic development activity in the country.
6 (c) 1 and 3 only 4. (d) 1 and 3
5. (c) It is the impact of the price levels of
7 (c) To create autonomous regions in
previous year on the calculation of
tribal areas
inflation rate.
8 (d) All of the above 6. (d) It is basically a subject of the Central
9 (b) Paper and pulp Government and the State
10 (a) Promoting skill development Governments are only a facilitator for
11 (d) 1, 2 and 3 its successful implementation
12 (b) Foreign-currency essets, gold 7. (d) neither exports nor imports take
holdings of the RBI and SDRs place
13 (d) Creating new money to finance a 8. (b) FII helps in increasing capital
budget deficit availability in general, while FDI only
14 (d) an increase in the level of income targets specific sectors.
and employment 9. (c) 1 and 3 only.
15 (c) there is capital formation in X 10. (b) 1 and 2 only
11. (d) 1, 2 and 3
16 (b) 1, 3 and 4 only
12. (d) 1, 2 and 3
17 (a) total value of goods and services 13. (b) The Parliament of India.
produced by the nationals 14. (c) Consolidated Fund of India.
18 (c) 1 and 3 only 15. (d) 1, 2, 3 and 4
19 (b) 1, 2 and 4 16. (d) Adult members of any household.
20 (a) 1 only 17. (d) None of the statements (a), (b) and
21 (c) marginal productivity of labour is zero (c) given above is correct in this
22 (d) 4-1-3-2 context.
23 (c) purchase and sale of government 18. (b) 2 only
securities by the RBI . 19. (c) It grants loans to only member
countries.
20. (a) 1 only
2012 21. (b) 2 only
22. (a) Commercial Banks
1 (a) 1 and 2 only 23. (d) 1, 2 and 3
2 (d) 1, 2, 3 and 4 24. (b) 2 only
3 (b) 3 and 4 only 25. (b) Inflation
26. (a) More liquidity in the market
4 (d) 1, 2 and 3
5 (c) 2 only
2010
6 (c) 1 and 3
7 (d) 1, 2 and 3 only 1. (b) World Trade Organisation
8 (b) 2 and 3 only 2. (d) Statutory Liquidity Ratio
9 (c) individual banks should adopt particular 3. (c) 2 and 3 only
districts for intensive development 4. (a) 1 and 2 only
10 (a) 1 only 5. (c) It is a persistent fall in the general
11 (c) 1 and 3 only price level of goods and services
6. (d) Neither 1 nor 2
12 (c) 1 and 3 only
7. (c) Elimination of primary deficit by the
13 (c) 2, 3, 1 end of the fiscal year 2008-09
14 (c) 1, 2, 3 and 4 only 8. (a) An investor who feels that the price
of a particular security is going to
2011 fall
9. (b) India has double taxation avoidance
1. (b) 2 and 3 only agreement with Mauritius

32

https://telegram.me/CivilServices_UPSC
https://t.me/UPSC_PDF

ANSWERS - INDIAN ECONOMY (ECONOMIC AND SOCIAL DEVELOPMENT)

10. (a) 1 only


11. (c) Both 1 and 2
12. (c) SDR
13. (d) None of the above
15. (a) 1 and 2 only
16. (a) The commercial banks will have
less money to lend
17. (c) Both 1 and 2
18. (c) Eighth Plan
19. (a) Partners should be less than 20
20. (c) The orders passed by the Banking
Ombudsman are final and binding
on the parties concerned
21. (c) Both 1 and 2
22. (c) Excise duty
23. (a) 1 only
24. (b) 2 only
25. (b) Inclusive Growth

33

https://telegram.me/CivilServices_UPSC
https://t.me/UPSC_PDF

UPSC
PRELIMINARY EXAMINATION
(PAPER-I)
GENERAL STUDIES

PREVIOUS YEARS QUESTIONS

(YEAR 2011 TO 2019)

THEME  GENERAL ISSUES ON


ENVIRONMENTAL ECOLOGY, BIO-
DIVERSITY AND CLIMATE CHANGE

https://telegram.me/CivilServices_UPSC
https://t.me/UPSC_PDF

GENERAL ISSUES ON ENVIRONMENTAL ECOLOGY, BIO-DIVERSITY AND CLIMATE CHANGE

1. Carbon monoxide
2019
2. Methane
3. Ozone
1. In the context of proposals to the use of 4. Sulphur dioxide
hydrogen-enriched CNG (H-CNG) as fuel Which of the above are released into
for buses in public transport, consider the atmosphere due to the burning of
following statements: crop/biomass residue?
1. The main advantage of the use of H- (a) 1 and 2 only
CNG is the elimination of carbon (b) 2, 3 and 4 only
monoxide emissions. (c) 1 and 4 only
2. H-CNG as fuel reduces carbon (d) 1, 2, 3 and 4
dioxide and hydrocarbon emissions.
3. Hydrogen up to one-fifth by volume 5. In India, the use of carbofuran, methyl
can be blended with CNG as fuel for parathion, phorate and triazophos is viewed
buses. with apprehension. These chemicals are
4. H-CNG makes the fuel less used as
expensive than CNG. (a) pesticides in agriculture
Which of the statements given above is/are (b) preservatives in processed foods
correct? (c) fruit-ripening agents
(a) 1 only (d) moisturising agents in cosmetics
(b) 2 and 3 only
(c) 4 only 6. Consider the following statements:
(d) 1, 2, 3 and 4 1. Under Ramsar Convention, it is
mandatory on the part of the
2. Recently, there was a growing awareness Government of India to protect and
in our country about the importance of conserve all the wetlands in the
Himalayan nettle (Girardinia diversifolia) territory of India.
because it is found to be a sustainable 2. The Wetlands (Conservation and
source of Management) Rules, 2010 were
(a) anti-malarial drug framed by the Government of India
(b) biodiesel based on the recommendations of
(c) pulp for paper industry Ramsar Convention.
(d) textile fibre 3. The Wetlands (Conservation and
Management) Rules, 2010 also
3. Which of the following statements are encompass the drainage area or
correct about the deposits of 'methane catchment regions of the wetlands as
hydrate? determined by the authority.
1. Global warming might trigger the Which of the statements given above is/are
release of methane gas from these correct?
deposits. (a) 1 and 2 only
2. Large deposits of 'methane hydrate' (b) 2 and 3 only
are found in Arctic Tundra and under (c) 3 only
the seafloor. (d) 1, 2 and 3
3. Methane in atmosphere oxidizes to
carbon dioxide after a decade or two. 7. Consider the following statements:
Select the correct answer using the code 1. Asiatic lion is naturally found in India
given below. only.
(a) 1 and 2 only 2. Double-humped camel is naturally
(b) 2 and 3 only found in India only.
(c) 1 and 3 only 3. One-horned rhinoceros is naturally
(d) 1, 2 and 3 found in India only.
Which of the statements given above is/are
4. Consider the following: correct?

https://telegram.me/CivilServices_UPSC
https://t.me/UPSC_PDF

GENERAL ISSUES ON ENVIRONMENTAL ECOLOGY, BIO-DIVERSITY AND CLIMATE CHANGE

(a) 1 only Dolphin


(b) 2 only 3. Rusty- : Eastern Ghats
(c) 1 and 3 only spotted Cat
(d) 1, 2 and 3
Which of the pairs given above are
correctly matched?
8. In the context of which of the following do
(a) 1 and 2 only
some scientists suggest the use of cirrus
(b) 2 and 3 only
cloud thinning technique and the injection
(c) 1 and 3 only
of sulphate aerosol into stratosphere?
(d) 1, 2 and 3
(a) Creating the artificial rains in some
regions
12. Why is there a great concern about the
(b) Reducing the frequency and intensity
'microbeads' that are released into
of tropical cyclones
environment?
(c) Reducing the adverse effects of solar
(a) They are considered harmful to
wind on the Earth
marine ecosystems.
(d) Reducing the global warming
(b) They are considered to cause skin
cancer in children.
9. Which of the following are in Agasthyamala
(c) They are small enough to be
Biosphere Reserve?
absorbed by crop plants in irrigated
(a) Neyyar, Peppara and Shendurney
fields.
Wildlife Sanctuaries; and Kalakad
(d) They are often found to be used as
Mundanthurai Tiger Reserve
food adulterants.
(b) Mudumalai, Sathyamangalam and
Wayanad Wildlife Sanctuaries; and
13. The word 'Denisovan' is sometimes
Silent Valley National Park
mentioned in media in reference to
(c) Kaundinya, Gundla Brahme-swaram
(a) fossils of a kind of dinosaurs
and Papikonda Wildlife Sanctuaries;
(b) an early human species
and Mukurthi National Park
(c) a cave system found in North-East
(d) Kawal and Sri Venkateswara Wildlife
India
Sanctuaries; and Nagarjunasagar-
(d) a geological period in the history of
Srisailam Tiger Reserve
Indian subcontinent

10. Consider the following statements:


14. Consider the following statements :
1. Some species of turtles are
1. As per law, the Compensatory
herbivores.
Afforestation Fund Management and
2. Some species of fish are herbivores.
Planning Authority exists at both
3. Some species of marine mammals
National and State levels.
are herbivores.
2. People's participation is mandatory in
4. Some species of snakes are
the compensatory afforestation
viviparous.
programmes carried out under the
Which of the statements given above are
Compensatory Afforestation Fund
correct?
Act, 2016.
(a) 1 and 3 only
Which of the statements given above is/are
(b) 2, 3 and 4 only
correct?
(c) 2 and 4 only
(a) 1 only
(d) 1, 2, 3 and 4
(b) 2 only
(c) Both 1 and 2
11. Consider the following pairs:
(d) Neither 1 nor 2
Wildlife : Naturally found
in 15. Consider the following statements :
1. Blue-finned : Cauvery River The Environment Protection Act, 1986
Mahseer empowers the Government of India to
2. Irrawaddy : Chambal River

https://telegram.me/CivilServices_UPSC
https://t.me/UPSC_PDF

GENERAL ISSUES ON ENVIRONMENTAL ECOLOGY, BIO-DIVERSITY AND CLIMATE CHANGE

1. state the requirement of public Select the correct answer using the code
participation in the process of given below :
environmental protection, and the (a) 1 only
procedure and manner in which it is (b) 2 and 3 only
sought
(c) 1 and 3 only
2. lay down the standards for emission
(d) 1, 2 and 3
or discharge of environmental
pollutants from various sources
Which of the statements given above is/are 2. The Partnership for Action on Green
correct? Economy (PAGE), a UN mechanism to assist
(a) 1 only countries transition towards greener and
(b) 2 only more inclusive economies, emerged at
(c) Both 1 and 2 (a) The Earth Summit on Sustainable
(d) Neither 1 nor 2 Development 2002, Johannesburg
(b) The United Nations Conference on
16. As per the Solid Waste Management Sustainable Development 2012, Rio de
Rules, 2016 in India, which one of the Janeiro
following statements is correct? (c) The United Nations Framework
(a) Waste generator has to segregate Convention on Climate Change 2015,
waste into five categories. Paris
(b) The Rules are applicable to notified
(d) The World Sustainable Development
urban local bodies, notified towns
Summit 2016, New Delhi
and all industrial townships only.
(c) The Rules provide for exact and
elaborate criteria for the identification 3. Why is a plant called Prosopis juliflora often
of sites for landfills and waste mentioned in news?
processing facilities. (a) Its extract is widely used in cosmetics.
(d) It is mandatory on the part of waste (b) It tends to reduce the biodiversity in the
generator that the waste generated in area in which it grows.
one district cannot be moved to (c) Its extract is used in the synthesis of
another district. pesticides.
17. In India, 'extended producer responsibility'
(d) None of the above
was introduced as an important feature in
which of the following?
(a) The Bio-medical Waste 4. Consider the following statements :
(Management and Handling) Rules, 1. Most of the world's coral reefs are in
1998 tropical waters.
(b) The Recycled (Manufacturing and 2. More than one-third of the world's coral
Usage) Pules, 1999 reefs are located in the territories of
(c) The (e-Waste) (Management and Australia, Indonesia and Philippines.
Handling) Rules, 2011 3. Coral reefs host far more number of
(d) The Food Safety and Standard animal phyla than those hosted by
Regulations, 2011 tropical rainforests.
Which of the statements given above is/ are
2018 correct ?
(a) 1 and 2 only
(b) 3 only
1. Which of the following is/are the possible
(c) 1 and 3 only
consequence/s of heavy sand mining in
riverbeds? (d) 1, 2 and 3
1. Decreased salinity in the river
2. Pollution of groundwater 5. "Momentum for Change : Climate Neutral
Now" is an initiative launched by
3. Lowering of the water-table

https://telegram.me/CivilServices_UPSC
https://t.me/UPSC_PDF

GENERAL ISSUES ON ENVIRONMENTAL ECOLOGY, BIO-DIVERSITY AND CLIMATE CHANGE

(a) The Intergovernmental Panel on ecosystems, pollution and global climate


Climate Change change.
(b) The UNEP Secretariat
(c) The UNFCCC Secretariat 8. In which one of the following States is Pakhui
(d) The World Meteorological Organization Wildlife Sanctuary located ?
(a) Arunachal Pradesh
6. With reference to the circumstances in I (b) Manipur
ndian agriculture, the concept of (c) Meghalaya
"Conservation Agriculture" assumes (d) Nagaland
significance. Which of the following fall under
the Conservation Agriculture ?
9. Consider the following statements :
1. Avoiding the monoculture practices
1. The definition of "Critical Wildlife Habitat"
2. Adopting minimum tillage is incorporated in the Forest Rights Act,
3. Avoiding the cultivation of plantation 2006.
crops 2. For the first time in India, Baigas have
4. Using crop residues to cover soil surface been given Habitat Rights.
5. Adopting spatial and temporal crop 3. Union Ministry of Environment, Forest
sequencing/crop rotations and Climate Change officially decides
Select the correct answer using the code and declares Habitat Rights for Primitive
given below : and Vulnerable Tribal Groups in any part
(a) 1, 3 and 4 of India.
(b) 2, 3, 4 and 5 Which of the statements given above is/ are
(c) 2, 4 and 5 correct ?
(d) 1, 2, 3 and 5 (a) 1 and 2 only
(b) 2 and 3 only
7. The term "sixth mass extinction/sixth (c) 3 only
extinction" is often mentioned in the news in (d) 1, 2 and 3
the context of the discussion of
(a) Widespread monoculture practices in 10. With reference to organic farming in India,
agriculture and large-scale commercial consider the following statements:
farming with indiscriminate use of 1. "The National Programme for Organic
chemicals in many parts of the world that Production' (NPOP) is operated under
may result in the loss of good native the guidelines and directions of the Union
ecosystems. Ministry of Rural Development.
(b) Fears of a possible collision of a 2. The Agricultural and Processed Food
meteorite with the Earth in the near future Products Export Development Authority'
in the manner it happened 65 million (APEDA) functions as the Secretariat for
years ago that caused the mass the implementation of NPOP.
extinction of many species including 3. Sikkim has become India's first fully
those of dinosaurs. organic State.
(c) Large scale cultivation of genetically Which of the statements given above is/ are
modified crops in many parts of the world correct ?
and promoting their cultivation in other
(a) 1 and 2 only
parts of the world which may cause the
(b) 2 and 3 only
disappearance of good native crop plants
and the loss of food biodiversity. (c) 3 only
(d) Mankind's over-exploitation/misuse of (d) 1, 2 and 3
natural resources, fragmentation/ loss of
natural habitats, destruction of

https://telegram.me/CivilServices_UPSC
https://t.me/UPSC_PDF

GENERAL ISSUES ON ENVIRONMENTAL ECOLOGY, BIO-DIVERSITY AND CLIMATE CHANGE

11. How is the National Green Tribunal (NGT) 14. Which of the following statements best
different from the Central Pollution Control describes "carbon fertilization" ?
Board (CPCB) ? (a) Increased plant growth due to increased
1. The NGT has been established by an Act concentration of carbon dioxide in the
whereas the CPCB has been created by atmosphere
an executive order of the Government. (b) Increased temperature of Earth due to
2. The NGT provides environmental justice increased concentration of carbon
and helps reduce the burden of litigation dioxide in the atmosphere
in the higher courts whereas the CPCB (c) Increased acidity of oceans as a result of
promotes cleanliness of streams and increased concentration of carbon
wells, and aims to improve the quality of dioxide in the atmosphere
air in the country. (d) Adaptation of all living beings on Earth to
Which of the statements given above is/ are the climate change brought about by the
correct ? increased concentration of carbon
(a) 1 only dioxide in the atmosphere
(b) 2 only
(c) Both 1 and 2 15. With reference to solar power production in
(d) Neither 1 nor 2 India, consider the following statements:
1. India is the third largest in the world in the
12. Which of the following has/have shrunk manufacture of silicon wafers used in
immensely/dried up in the recent past due to photovoltaic units.
human activities ? 2. The solar power tariffs are determined by
1. Aral Sea the Solar Energy Corporation of India.
2. Black Sea Which of the statements given above is/ are
correct ?
3. Lake Baikal
(a) 1 only (b) 2 only
Select the correct answer using the code
given below : (c) Both 1 and 2 (d) Neither 1 nor 2
(a) 1 only
(b) 2 and 3 2017
(c) 2 only
(d) 1 and 3 1. From the ecological point of view, which one
of the following assumes importance in being
13. With reference to the 'Global Alliance for a good link between the Eastern Ghats and
Climate-Smart Agriculture (GACSA)', which the Western Ghats ?
of the following statements is/are correct (a) Sathyamangalam Tiger Reserve
1. GACSA is an outcome of the Climate (b) Nallamala Forest
Summit held in Paris in 2015. (c) Nagarhole National Park
2. Membership of GACSA does not create (d) Seshachalam Biosphere Reserve
any binding obligations.
3. India was instrumental in the creation of
2. Consider the following statements in respect
GACSA.
of Trade Related Analysis of Fauna and
Select the correct answer using the code Flora in Commerce (TRAFFIC) :
given below :
1. TRAFFIC is a bureau under United
(a) 1 and 3 only Nations Environment Programme
(b) 2 only (UNEP).
(c) 2 and 3 only 2. The mission of TRAFFIC is to ensure that
(d) 1, 2 and 3 trade in wild plants and animals is not a
threat to the conservation of nature.

https://telegram.me/CivilServices_UPSC
https://t.me/UPSC_PDF

GENERAL ISSUES ON ENVIRONMENTAL ECOLOGY, BIO-DIVERSITY AND CLIMATE CHANGE

Which of the above statements is/are correct following can be the potential sites for carbon
? sequestration ?
(a) 1 only 1. Abandoned and uneconomic coal seams.
(b) 2 only 2. Depleted oil and gas reservoirs.
(c) Both 1 and 2 3. Subterranean deep saline formations
(d) Neither 1 nor 2 Select the correct answer using the code
given below :
3. In the context of solving pollution problems, (a) 1 and 2 only
what is/are the advantage/ advantages of (b) 3 only
bioremediation technique? (c) 1 and 3 only
1. It is a technique for cleaning up pollution (d) 1, 2 and 3
by enhancing the same biodegradation
process that occurs in nature.
6. According to the Wildlife (Protection) Act,
2. Any contaminant with heavy metals such 1972, which of the following animals cannot
as cadmium and lead can be readily and be hunted by any person except under some
completely treated by bioremediation provisions provided by law?
using microorganisms.
1. Gharial
3. Genetic engineering can be used to
2. Indian wild ass
create microorganisms specifically
3. Wild buffalo
designed for bioremediation.
Select the correct answer using the code
Select the correct answer using the code
given below :
given below :
(a) 1 only
(a) 1 only
(b) 2 and 3 only
(b) 2 and 3 only
(c) 1 and 3 only
(c) 1 and 3 only
(d) 1, 2 and 3
(d) 1, 2 and 3

7. With reference to 'Global Climate Change


4. Due to some reasons, if there is a huge fall in
Alliance', which of the following statements
the population of species of butterflies, what
is/are correct?
could be its likely consequence/
consequences ? 1. It is an initiative of the European Union.
1. Pollination of some plants could be 2. It provides technical and financial support
adversely affected. to targeted developing countries to
integrate climate change into their
2. There could be a drastic increase in the
development policies and budgets.
fungal infections of some cultivated
plants. 3. It is coordinated by World Resources
Institute (WRI) and World Business
3. It could lead to a fall in the population of
Council for Sustainable Development
some species of wasps, spiders and
(WBCSD).
birds.
Select the correct answer using the code
Select the correct answer using the code
given below :
given below :
(a) 1 and 2 only
(a) 1 only
(b) 3 only
(b) 2 and 3 only
(c) 2 and 3 only
(c) 1 and 3 only
(d) 1, 2 and 3
(d) 1, 2 and 3

8. Biological Oxygen Demand (BOD) is a


5. In the context of mitigating the impending
standard criterion for
global warming due to anthropogenic
emissions of carbon dioxide, which of the (a) Measuring oxygen levels in blood

https://telegram.me/CivilServices_UPSC
https://t.me/UPSC_PDF

GENERAL ISSUES ON ENVIRONMENTAL ECOLOGY, BIO-DIVERSITY AND CLIMATE CHANGE

(b) Computing oxygen levels in forest associated landscapes, agricultural


ecosystems. biodiversity and knowledge systems of
(c) Pollution assay in aquatic ecosystems the local communities
(d) Assessing oxygen levels in high altitude 3. To provide Geographical Indication status
regions. to all the varieties of agricultural produce
in such identified GIAHS
9. If you want to see gharials in their natural Select the correct answer using the code
habitat, which one of the following is the best given below.
place to visit ? (a) 1 and 3 only
(a) Bhitarkanika Mangroves (b) 2 only
(b) Chambal River (c) 2 and 3 only
(c) Pulicat Lake (d) 1, 2 and 3
(d) Deepor Beel
2. Consider the following pairs
10. In India, if a species of tortoise is declared Terms sometimes seen Their origin
protected under Schedule I of the Wildlife in the news
(Protection) Act, 1972, what does it imply?
1. Annex-I Countries : Cartagena
(a) It enjoys the same level of protection as
Protocol
the tiger.
(b) It no longer exists in the wild, a few 2. Certified : Nagoya
individuals are under captive protection; Emissions Protocol
and now it is impossible to prevent its Reductions
extinction,
(c) It is endemic to a particular region of 3. Clean : Kyoto
India. Development Protocol
(d) Both (b) and (c) stated above are correct Mechanism
in this context.
Which of the pairs given above is/are
correctly matched?
11. Recently there was a proposal to translocate
(a) 1 and 2 only
some of the lions from their natural habitat in
(b) 2 and 3 only
Gujarat to which one of the following sites ?
(c) 3 only
(a) Corbett National Park
(d) 1, 2 and 3
(b) Kuno Palpur Wildlife Sanctuary
(c) Mudumalai Wildlife Sanctuary
3. Which of the following best describes/
(d) Sariska National Park
describe the aim of 'Green India Mission' of
the Government of India?
2016 1. Incorporating environmental benefits and
costs into the Union and State Budgets
thereby implementing the 'green
1. The FAO accords the status of 'Globally
accounting'
Important Agricultural Heritage System
(GIAHS)' to traditional agricultural systems. 2. Launching the second green revolution to
What is the overall goal of this initiative? enhance agricultural output so as to
ensure food security to one and all in the
1. To provide modern technology, training in
future
modern farming methods and financial
support to local communities of identified 3. Restoring and enhancing forest cover
GIAHS so as to greatly enhance their and responding to climate change by a
agricultural productivity combination of adaptation and mitigation
measures
2. To identify and safeguard eco-friendly
traditional farm practices and their

https://telegram.me/CivilServices_UPSC
https://t.me/UPSC_PDF

GENERAL ISSUES ON ENVIRONMENTAL ECOLOGY, BIO-DIVERSITY AND CLIMATE CHANGE

Select the correct answer using the code 2. It is a global initiative that focuses on
given below: drawing attention to the economic
(a) 1 only benefits of biodiversity.
(b) 2 and 3 only 3. It presents an approach that can help
(c) 3 only decision-makers recognize, demonstrate
and capture the value of ecosystems and
(d) 1, 2 and 3
biodiversity.
Select the correct answer using the code
4. 'Net metering' is sometimes seen in the news
given below.
in the context of promoting the
(a) 1 and 2 only
(a) production and use of solar energy by the
(b) 3 only
households/consumers
(c) 2 and 3 only
(b) use of piped natural gas in the kitchens of
households (d) 2 and 3
(c) installation of CN G kits in motor-cars
(d) installation of water meters in urban 8. Which of the following statements is/are
households correct?
Proper design and effective implemen-tation
of UN-REDD+ Programme can significantly
5. What is/are unique about 'Kharai camel', a
contribute to
breed found in India?
1. protection of biodiversity
1. It is capable of swimming up to three
kilometres in seawater. 2. resilience of forest ecosystems
2. It survives by grazing on mangroves. 3. poverty reduction
3. It lives in the wild and cannot be Select the correct answer using the code
domesticated. given below.
Select the correct answer using the code (a) 1 and 2 only
given below. (b) 3 only
(a) 1 and 2 only (c) 2 and 3 only
(b) 3 only (d) 1, 2 and 3
(c) 1 and 3 only
(d) 1, 2 and 3 9. What is 'Greenhouse Gas Protocol?
(a) It is an international accounting tool for
6. Recently, our scientists have discovered a government and business leaders to
new and distinct species of banana plant understand, quantify and manage
which attains a height of about 11 metres and greenhouse gas emissions
has orange-coloured fruit pulp. In which part (b) It is an initiative of the United Nations to
of India has it been discovered? offer financial incentives to developing
(a) Andaman Islands countries to reduce greenhouse gas
emissions and to adopt eco-friendly
(b) Anaimalai Forests
technologies
(c) Maikala Hills
(c) It is an inter-governmental agreement
(d) Tropical rain forests of northeast
ratified by all the member countries of the
United Nations to reduce greenhouse gas
7. With reference to an initiative called The emissions to specified levels by the year
Economics of Ecosystems and Biodiversity 2022
(TEEB)', which of the following statements (d) It is one of the multilateral REDD+
is/are correct? initiatives hosted by the World Bank
1. It is an initiative hosted by UNEP, IMF
and World Economic Forum.

https://telegram.me/CivilServices_UPSC
https://t.me/UPSC_PDF

GENERAL ISSUES ON ENVIRONMENTAL ECOLOGY, BIO-DIVERSITY AND CLIMATE CHANGE

10. 'Gadgil Committee Report' and


'Kasturirangan Committee Report', 14. Recently, for the first time in our country,
sometimes seen in the news, are related to which of the following States has declared a
(a) constitutional reforms particular butterfly as 'State Butterfly?
(b) Ganga Action Plan (a) Arunachal Pradesh
(c) linking of rivers (b) Himachal Pradesh
(d) protection of Western Ghats (c) Karnataka
(d) Maharashtra
11. In which of the following can you find the
Bureau of Energy Efficiency Star Label? 15. With reference to the Agreement at the
1. Ceiling fans UNFCCC Meeting in Paris in 2015, which of
2. Electric geysers the following statements is/are correct?
3. Tubular fluorescent lamps 1. The Agreement was signed by all the
Select the correct answer using the code member countries of the UN and it will go
given below. into effect in 2017.
(a) 1 and 2 only 2. The Agreement aims to limit the
greenhouse gas emissions so that the
(b) 3 only
rise in average global temperature by the
(c) 2 and 3 only
end of this century does not exceed 2°C
(d) 1, 2 and 3 or even 1.5°C above pre-industrial levels.
3. Developed countries acknowledged their
12. Consider the following statements : historical responsibility in global warming
1. The International Solar Alliance was and committed to donate $ 1000 billion a
launched at the United Nations Climate year from 2020 to help developing
Change Conference in 2015. countries to cope with climate change.
2. The Alliance includes all the member Select the correct answer using the code
countries of the United Nations. given below.
Which of the statements given above is/ are (a) 1 and 3 only
correct? (b) 2 only
(a) 1 only (c) 2 and 3 only
(b) 2 only (d) 1, 2 and 3
(c) Both 1 and 2
(d) Neither 1 nor 2 16. The term 'Intended Nationally Determined
Contributions' is sometimes seen in the news
13. In the cities of our country, which among the in the context of
following atmospheric gases are normally (a) pledges made by the European countries
considered in calculating the value of Air to rehabilitate refugees from the war-
Quality Index? affected Middle East
1. Carbon dioxide x (b) plan of action outlined by the countries of
2. Carbon monoxide the world to combat climate change
3. Nitrogen dioxide (c) capital contributed by the member
4. Sulfur dioxide countries in the establishment of Asian
Infrastructure Investment Bank
5. Methane
(d) plan of action outlined by the countries of
Select the correct answer using the code
the world regarding Sustainable
given below.
Development Goals
(a) 1, 2 and 3 only
(b) 2, 3 and 4 only
17. What is/are the importance/importances of
(c) 1, 4 and 5 only the 'United Nations Convention to Combat
(d) 1, 2, 3, 4 and 5 Desertification'?

10

https://telegram.me/CivilServices_UPSC
https://t.me/UPSC_PDF

GENERAL ISSUES ON ENVIRONMENTAL ECOLOGY, BIO-DIVERSITY AND CLIMATE CHANGE

1. It aims to promote effective action 1. With reference to an organization known as


through innovative national programmes 'BirdLife International', which of the following
and supportive inter-national statements is/are correct ?
partnerships. 1. It is a Global Partnership of Conservation
2. It has a special/particular focus on South Organizations.
Asia and North Africa regions, and its 2. The concept of 'biodiversity hotspots'
Secretariat facilitates the allocation of originated from this organization.
major portion of financial resources to 3. It identifies the sites known/referred to as
these regions. 'Important Bird and Biodiversity Areas'.
3. It is committed to bottom-up approach, Select the correct answer using the code
encouraging the partici-pation of local given below.
people in combating the desertification.
(a) 1 only
Select the correct answer using the code
(b) 2 and 3 only
given below.
(c) 1 and 3 only
(a) 1 only
(d) 1, 2 and 3
(b) 2 and 3 only
(c) 1 and 3 only
2. In India, in which one of the following types of
(d) 1, 2 and 3
forests is teak a dominant tree species?
(a) Tropical moist deciduous forest
18. In which of the following regions of India are
(b) Tropical rain forest
you most likely to come across the 'Great
(c) Tropical thorn scrub forest
Indian Hornbill' in its natural habitat?
(d) Temperate forest with grasslands
(a) Sand deserts of northwest India
(b) Higher Himalayas of Jammu and Kashmir
3. Which one of the following is the best
(c) Salt marshes of western Gujarat
description of the term 'ecosystem'?
(d) Western Ghats
(a) A community of' organisms interacting
with one another
19. Which of the following are the key features of
(b) That part of' the Earth which is inhabited
'National Ganga River Basin Authority
by living organisms
(NGRBA) ?
(c) community of organisms together with
1. River basin is the unit of planning and
the environment in which they live
management.
(d) The flora and fauna of a geographical
2. It spearheads the river conser-vation
area
efforts at the national level.
3. One of the Chief Ministers of the States
4. Which of the following National Parks is
through which the Ganga flows becomes
unique in being a swamp with floating
the Chairman of NGRBA on rotation
vegetation that supports a rich biodiversity?
basis.
(a) Bhitarkanika National Park
Select the correct answer using the code
given below. (b) Keibul Lamjao National Park
(a) 1 and 2 only (c) Keoladeo Ghana National Park
(b) 2 and 3 only (d) Sultanpur National Park
(c) 1 and 3 only
(d) 1, 2 and 3 5. With reference to the International Union for
Conservation of Nature and Natural
Resources (IUCN) and the Convention on
2015 International Trade in Endangered Species of
Wild Fauna and Flora (CITES), which of the
following statements is/are correct?

11

https://telegram.me/CivilServices_UPSC
https://t.me/UPSC_PDF

GENERAL ISSUES ON ENVIRONMENTAL ECOLOGY, BIO-DIVERSITY AND CLIMATE CHANGE

1. IUCN is an organ of the United Nations (a) algae and bacteria


and CI TES is an international agreement (b) algae and fungi
between governments. (c) bacteria and fungi
2. IUCN runs thousands of field projects (d) fungi and mosses
around the world to better manage
natural environments.
2. If you travel through the Himalayas, you are
3. CITES is legally binding on the States
likely to see which of the following plants
that have joined it, but this Convention
naturally growing there ?
does not take the place of national laws.
1. Oak
Select the correct answer thing the code
2. Rhododendron
given below.
3. Sandalwood
(a) 1 only
Select the correct answer using the code
(b) 2 and 3 only
given below.
(c) 1 and 3 only
(a) 1 and 2 only
(d) 1, 2 and 3
(b) 3 only
(c) 1 and 3 only
6. Which one, of the following regions of India
(d) 1, 2 and 3
has , a combination of mangrove forest,
evergreen forest and deciduous forest?
(a) North Coastal Andhra Pradesh 3. Which of the following are some important
pollutants released by steel industry in India?
(b) South-West Bengal
1. Oxides of sulphur
(c) Southern Saurashtra
2. Oxides of nitrogen
(d) Andaman and Nicobar Islands
3. Carbon monoxide
4. Carbon dioxide
7. Which one of the following is associated with
the issue of control and phasing out of the Select the correct answer using the code
use of ozone-depleting substances? given below
(a) Bretton Woods Conference (a) 1, 3 and 4 only
(b) Montreal Protocol (b) 2 and 3 only
(c) Kyoto Protocol (c) 1 and 4 only
(d) Nagoya Protocol (d) 1, 2, 3 and 4

8. Consider the following States: 4. With reference to 'Eco-Sensitive Zones',


which of the following statements is/are
1. Arunachal Pradesh
correct?
2. Himachal Pradesh
1. Eco-Sensitive Zones are the areas that
3. Mizoram
are declared under the Wildlife
In which of the above States do 'Tropical Wet (Protection) Act, 1972.
Evergreen Forests' occur?
2. The purpose of the declaration of Eco-
(a) 1 only Sensitive Zones is to prohibit all kinds of
(b) 2 and 3 only human activities in those zones except
(c) 1 and 3 only agriculture.
(d) 1, 2 and 3 Select the correct answer using the code
given below.
(a) 1 only
2014
(b) 2 only
(c) Both 1 and 2
1. Lichens, which are capable of initiating (d) Neither 1 nor 2
ecological succession even on a bare rock,
are actually a symbiotic association of

12

https://telegram.me/CivilServices_UPSC
https://t.me/UPSC_PDF

GENERAL ISSUES ON ENVIRONMENTAL ECOLOGY, BIO-DIVERSITY AND CLIMATE CHANGE

5. Consider the following statements: 2. It is a movement in which the participants


1. Animal Welfare Board of India is switch off the lights for one hour on a
established under the Environment certain day every year.
(Protection) Act, 1986. 3. It is a movement to raise the awareness
2. National Tiger Conservation Authority is a about the climate change and the need to
statutory body. save the planet.
3. National Ganga River Basin Authority is Which of the statements given above is/ are
chaired by the Prime Minister. correct?
Which of the statements given above is/ are (a) 1 and 3 only
correct? (b) 2 only
(a) 1 only (c) 2 and 3 only
(b) 2 and 3 only (d) 1, 2 and 3
(c) 2 only
(d) 1, 2 and 3 9. If a wetland of international importance is
brought under the 'Montreux Record', what
6. There is some concern regarding the does it imply?
nanoparticles of some chemical elements (a) Changes in ecological character have
that are used by the industry in the occurred, are occurring or are likely to
manufacture of various products. Why? occur in the wetland as a result of human
1. They can accumulate in the environment, interference
and contaminate water and soil. (b) The country in which the wetland is
2. They can enter the food chains. located should enact a law to prohibit any
human activity within five kilometres from
3. They can trigger the production of free
the edge of the wetland
radicals.
(c) The survival of the wetland depends on
Select the correct answer using the code
the cultural practices and traditions of
given below.
certain communities living in its vicinity
(a) 1 and 2 only
and therefore the cultural diversity therein
(b) 3 only should not be destroyed
(c) 1 and 3 only (d) It is given the status of 'World Heritage
(d) 1, 2 and 3 Site'

7. Consider the following international 10. With reference to Bombay Natural History
agreements : Society (BNHS), consider the following
1. The International Treaty on Plant Genetic statements :
Resources for Food and Agriculture 1. It is an autonomous organization under
2. The United Nations Convention to the Ministry of Environment and Forests.
Combat Desertification 2. It strives to conserve nature through
3. The World Heritage Convention action-based research, education and
Which of the above has/have a bearing on public awareness.
the biodiversity? 3. It organizes and conducts nature trails
(a) 1 and 2 only and camps for the general public.
(b) 3 only Which of the statements given above is/ are
(c) 1 and 3 only correct?
(d) 1, 2 and 3 (a) 1 and 3 only
(b) 2 only

8. Consider the following statements regarding (c) 2 and 3 only


'Earth Hour' : (d) 1, 2 and 3
1. It is an initiative of UNEP and UNESCO.

13

https://telegram.me/CivilServices_UPSC
https://t.me/UPSC_PDF

GENERAL ISSUES ON ENVIRONMENTAL ECOLOGY, BIO-DIVERSITY AND CLIMATE CHANGE

11. With reference to 'Global Environment


Facility', which of the following statements 14. Brominated flame retardants are used in
is/are correct? many household products like mattresses
(a) It serves as financial mechanism for and upholstery.
'Convention on Biological Diversity' and Why is there some concern about their use?
'United Nations Framework Convention 1. They are highly resistant to degradation
on Climate Change' in the environment.
(b) It undertakes scientific research on 2. They are able to accumulate in humans
environmental issues at global level and animals.
(c) It is an agency under OECD to facilitate Select the correct answer using the code
the transfer of technology and funds to given below.
underdeveloped countries with specific
(a) 1 only
aim to protect their environment
(b) 2 only
(d) Both (a) and (b)
(c) Both 1 and 2
(d) Neither 1 nor 2
12. With reference to a conservation organization
called 'Wetlands International', which of the
following statements is/are correct? 15. Consider the following :
1. It is an intergovernmental organization 1. Bats
formed by the countries which are 2. Bears
signatories to Ramsar Convention. 3. Rodents
2. It works at the field level to develop and The phenomenon of hibernation can be
mobilize knowledge, and use the observed in which of the above kinds of
practical experience to advocate for animals?
better policies. (a) 1 and 2 only
Select the correct answer using the code (b) 2 only
given below. (c) 1, 2 and 3
(a) 1 only (d) Hibernation cannot be observed in any of
(b) 2 only the above
(c) Both 1 and 2
(d) Neither 1 nor 2 16. Which of the following adds/add carbon
dioxide to the carbon cycle on the planet
13. Other than poaching, what are the possible Earth?
reasons for the decline in the population of 1. Volcanic action
Ganges River Dolphins? 2. Respiration
1. Construction of dams and barrages on 3. Photosynthesis
rivers. 4. Decay of organic matter
2. Increase in the population of crocodiles in Select the correct answer using the code
rivers. given below.
3. Getting trapped in fishing nets (a) 1 and 3 only
accidentally.
(b) 2 only
4. Use of synthetic fertilizers and other
(c) 1, 2 and 4 only
agricultural chemicals in crop-fields in the
vicinity of rivers. (d) 1, 2, 3 and 4
Select the correct answer using the code
given below. 17. If you walk through countryside, you are likely
(a) 1 and 2 only to see some birds stalking alongside the
cattle to seize the insects disturbed by their
(b) 2 and 3 only
movement through grasses.
(c) 1, 3 and 4 only
Which of the following is/are such bird/ birds?
(d) 1, 2, 3 and 4

14

https://telegram.me/CivilServices_UPSC
https://t.me/UPSC_PDF

GENERAL ISSUES ON ENVIRONMENTAL ECOLOGY, BIO-DIVERSITY AND CLIMATE CHANGE

1. Painted Stork
2. Common Myna 21. The scientific view is that the increase in
3. Black-necked Crane global temperature should not exceed 2°C
Select the correct answer using the code above pre-industrial level. If the global
o
given below. temperature increases beyond 3 C above the
pre-industrial level, what can be its possible
(a) 1 and 2
impact/impacts on the world?
(b) 2 only
1. Terrestrial biosphere tends toward a net
(c) 2 and 3
carbon source.
(d) 3 only
2. Widespread coral mortality will occur.
3. All the global wetlands will permanently
18. Which of the following have coral reefs? disappear.
1. Andaman and Nicobar Islands 4. Cultivation of cereals will not be possible
2. Gulf of Kachchh anywhere in the world.
3. Gulf of Mannar Select the correct answer using the code
4. Sunderbans given below.
Select the correct answer using the code (a) 1 only
given below. (b) 1 and 2 only
(a) 1, 2 and 3 only (c) 2, 3 and 4 only
(b) 2 and 4 only (d) 1, 2, 3 and 4
(c) 1 and 3 only
(d) 1, 2, 3 and 4
2013
19. Consider the following pairs:
1. Due to improper/indiscriminate disposal of old
Wetlands Confluence of rivers
and used computers or their parts, which of
1. Harike Wetlands : Confluence of Beas the following are released into the
and Satluj / Sutlej environment as e-waste?
1. Beryllium
2. Keoladeo : Confluence of 2. Cadmium
Ghana National Banas and Chambal
3. Chromium
Park
4. Heptachlor
3. Kolleru Lake : Confluence of Musi 5. Mercury
and Krishna 6. Lead
7. Plutonium
Which of the above pairs is/are correctly
Select the correct 'answer using the codes
matched ?
given below.
(a) 1 only
(a) 1, 3, 4, 6 and 7 only
(b) 2 and 3 only
(b) 1, 2, 3, 5 and 6 only
(c) 1 and 3 only
(c) 2, 4, 5 and 7 only
(d) 1, 2 and 3
(d) 1, 2, 3, 4, 5, 6 and 7

20. The most important strategy for the


2. Acid rain is caused by the pollution of
conservation of biodiversity together with
environment by
traditional human life is the establishment of
(a) carbon dioxide and nitrogen
(a) biosphere reserves
(b) carbon monoxide and carbon dioxide
(b) botanical gardens
(c) ozone and carbon dioxide
(c) national parks
(d) nitrous oxide and sulphur dioxide
(d) wildlife sanctuaries

15

https://telegram.me/CivilServices_UPSC
https://t.me/UPSC_PDF

GENERAL ISSUES ON ENVIRONMENTAL ECOLOGY, BIO-DIVERSITY AND CLIMATE CHANGE

3. With reference to food chains in ecosystems) (d) Oceans, mangroves, takes,


consider the following statements : grasslands
1. A food chain illustrates the order in which
a chain of organisms feed upon each 7. With reference to the food chains in
other. ecosystems, which of the following kinds of
2. Food chains are found within the organism is/are known as decomposer
populations of a species. organism/organisms?
3. A food chain illustrates the numbers of 1. Virus
each organism which are eaten by 2. Fungi
others. 3. Bacteria
Which of the statements given above is/ are Select the correct answer using the codes
correct? given below.
(a) 1 only (a) 1 only
(b) 1 and 2 only (b) 2 and 3 only
(c) 1, 2 and 3 (c) 1 and 3 only
(d) None (d) 1, 2 and 3

4. Which of the following can be found as


pollutants in the drinking water in some parts 2012
of India?
1. Arsenic 1. Consider the following protected areas:
2. Sorbitol 1. Bandipur
3. Fluoride 2. Bhitarkanjka
4. Formaldehyde 3. Manas
5. Uranium 4. Sunderbans
Select the correct answer using the codes Which of the above are declared Tiger
given below. Reserves?
(a) 1 and 3 only (a) 1 and 2 only
(b) 2, 4 and 5 only (b) 1, 3 and 4 only
(c) 1, 3 and 5 only (c) 2, 3 and 4 only
(d) 1, 2, 3, 4 and 5 (d) 1, 2, 3 and 4

5. In the grasslands, trees do not replace the 2. Which one of the following is the
grasses as a part of an ecological succession characteristic climate of the Tropical
because of Savannah Region ?
(a) insects and fungi (a) Rainfall throughout the year
(b) limited sunlight and paucity of nutrients (b) Rainfall in winter only
(c) water limits and fire (c) An extremely short dry season
(d) None of the above (d) A definite dry and wet season

6. Which one of the following is the correct 3. In which one among the following categories
sequence of ecosystems in the order of of protected areas in India are local people
decreasing productivity? not allowed to collect and use the biomass?
(a) Oceans, lakes, grasslands, (a) Biosphere Reserves
mangroves (b) National Parks
(b) Mangroves, oceans, grasslands, lakes (c) Wetlands declared under Ramsar
(c) Mangroves grasslands, lakes, oceans Convention
(d) Wildlife Sanctuaries

16

https://telegram.me/CivilServices_UPSC
https://t.me/UPSC_PDF

GENERAL ISSUES ON ENVIRONMENTAL ECOLOGY, BIO-DIVERSITY AND CLIMATE CHANGE

(d) None of them


4. The Millennium Ecosystem Assessment
describes the following major categories of 8. What would happen if phytoplankton of an
ecosystem services—provisioning, ocean is completely destroyed for some
supporting, regulating, preserving and reason?
cultural. 1. The ocean as a carbon sink would be
Which one of the following is supporting adversely affected.
service? 2. The food chains in the ocean would be
(a) Production of food and water adversely affected.
(b) Control of climate and disease 3. The density of ocean water would
(c) Nutrient cycling and crop pollination drastically decrease.
(d) Maintenance of diversity Select the correct answer using the codes
given below :
5. Which of the following can be threats to the (a) 1 and 2 only
biodiversity of a geographical area ? (b) 2 only
1. Global warming (c) 3 only
2. Fragmentation of habitat (d) 1, 2 and 3
3. Invasion of alien species
4. Promotion of vegetarianism 9. Vultures which used to be very common in
Select the correct answer using the codes Indian countryside some years ago are rarely
given below: seen nowadays. This is attributed to:
(a) 1, 2 and 3 only (a) the destruction of their nesting sites by
(b) 2 and 3 only new invasive species
(c) 1 and 4 only (b) a drug used by cattle owners for treating
their diseased cattle
(d) 1, 2, 3 and 4
(c) scarcity of food available to them
(d) a widespread, persistent and fatal
6. Consider the following :
disease among them
1. Black-necked crane
2. Cheetah
10. How does National Biodiversity Authority
3. Flying squirrel
(NBA) help in protecting the Indian
4. Snow leopard agriculture?
Which of the above are naturally found in 1. NBA checks the biopiracy and protects
India ? the indigenous and traditional genetic
(a) 1, 2 and 3 only resources.
(b) 1, 3 and 4 only 2. NBA directly monitors and supervises the
(c) 2 and 4 only scientific research on genetic
(d) 1, 2, 3 and 4 modification of crop plants.
3. Application for Intellectual Property
7. Consider the following agricultural practices - Rights related to genetic/biological
resources cannot be made without the
1. Contour bunding
approval of NBA.
2. Relay cropping
Which of the statements given above is/ are
3. Zero tillage correct?
In the context of global climate change, which (a) 1 only
of the above helps/help in carbon
(b) 2 and 3 only
sequestration/storage in the soil?
(c) 1 and 3 only
(a) 1 and 2 only
(d) 1, 2 and 3
(b) 3 only
(c) 1, 2 and 3

17

https://telegram.me/CivilServices_UPSC
https://t.me/UPSC_PDF

GENERAL ISSUES ON ENVIRONMENTAL ECOLOGY, BIO-DIVERSITY AND CLIMATE CHANGE

11. Biomass gasification is considered to be one a protected area. Which one of the following
of the sustainable solutions to the power could be this ?
crisis in India. In this context, which of the (a) Bhitarkanika
following statements is/are correct? (b) Chandipur - on-sea
1. Coconut shells, groundnut shells and rice (c) Gopalpur-on-sea
husk can be used in biomass gasification.
(d) Simlipal
2. The combustible gases generated from
biomass gasification consist of hydrogen
3. Regarding "carbon credits", which one of the
and carbon dioxide only.
following statements is not correct ?
3. The combustible gases generated from
(a) The carbon credit system was ratified in
biomass gasification can be used for
conjuction with the Kyoto Protocol
direct heat generation but not in internal
combustion engines. (b) Carbon credits are awarded to countries
or groups that have reduced greenhouse
Select the correct answer using the codes
gases below their emission quota
given below :
(c) The goal of the carbon credit system is to
(a) 1 only
limit the increase of carbon dioxide
(b) 2 and 3 only
emission
(c) 1 and 3 only
(d) Carbon credits are traded at a price fixed
(d) 1, 2 and 3 from time to time by the United Nations
Environment Programme

2011 4. Three of the following criteria have


contributed to the recognition of Western
1. In the Union Budget 2011-12, a full Ghats- Sri Lanka and Indo-Burma regions as
exemption from the basic customs duty was hotspots of biodiversity :
extended to the bio-based asphalt 1. Species richness
(bioasphalt). What is the importance of this 2. Vegetation density
material ?
3. Endemism
1. Unlike traditional asphalt, bioasphalt is
4. Ethno-botanical importance
not based on fossil fuels.
5. Threat perception
2. Bioasphalt can be made from non-
6. Adaptation of flora and fauna to warm
renewable resources.
and humid conditions.
3. Bioasphalt can be made from organic
Which three of the above are correct criteria
waste materials.
in this context ?
4. It is eco-friendly to use bioasphalt for
(a) 1, 2 and 6 (b) 2, 4 and 6
surfacting of the roads.
(c) 1, 3 and 5 (d) 3, 4 and 6
Which of the statements given above are
correct?
(a) 1, 2 and 3 only 5. With reference to India, consider the following
Central Acts:
(b) 1, 3 and 4 only
1. Import and Export (Control) Act, 1947.
(c) 2 and 4 only
2. Mining and Mineral Development
(d) 1, 2, 3 and 4
(Regulation) Act, 1957
3. Customs Act, 1962
2. Two important rivers - one with its source in
4. Indian Forest Act, 1927
Jharkhand (and known by a different name in
Odisha), and another, with its source in Which of the above Acts have relevance to/
Odisha - merge at a place only a short bearing on the biodiversity conservation in
distance from the coast of Bay of Bengal the country ?
before flowing into the sea. This is an (a) 1 and 3 only
important site of wildlife and biodiversity and (b) 2, 3 and 4 only

18

https://telegram.me/CivilServices_UPSC
https://t.me/UPSC_PDF

GENERAL ISSUES ON ENVIRONMENTAL ECOLOGY, BIO-DIVERSITY AND CLIMATE CHANGE

(c) 1, 2, 3 and 4 8. Biodiversity forms the basis for human


(d) None of the above Acts existence in the following ways :
1. Soil formation
6. There is a concern over the increase in harmful 2. Prevention of soil erosion.
algal blooms in the seawaters of India. What 3. Recycling of waste
could be the causative factors for this 4. Pollination of crops
phenomenon ? Select the correct answer using the codes
1. Discharge of nutrients from the estuaries. given below:
2. Run-off from the land during the (a) 1, 2 and 3 only
monsoon. (b) 2, 3 and 4 only
3. Upwelling in the seas. (c) 1 and 4 only
Select the correct answer from the codes (d) 1, 2, 3 and 4
given below:
(a) 1 only
9. Consider the following statements:
(b) 1 and 2 only
1. Biodiversity is normally greater in the
(c) 2 and 3 only lower latitudes as compared to the higher
(d) 1, 2 and 3 latitudes.
2. Along the mountain gradients,
7. A sandy and saline area is the natural habitat biodiversity is normally greater in the
of an Indian animal species. The animal has lower altitudes as compared to the higher
no predators in that area but its existence is altitudes.
threatened due to the destruction of its Which of the statements given above is/ are
habitat. Which one of the following could be correct?
that animal? (a) 1 only
(a) Indian wild buffalo (b) 2 only
(b) Indian wild ass (c) Both 1 and 2
(c) Indian wild boar (d) Neither 1 nor 2
(d) Indian gazelle

19

https://telegram.me/CivilServices_UPSC
https://t.me/UPSC_PDF

GENERAL ISSUES ON ENVIRONMENTAL ECOLOGY, BIO-DIVERSITY AND CLIMATE CHANGE

GENERAL ISSUES ON ENVIRONMENTAL ECOLOGY, BIO-


DIVERSITY AND CLIMATE CHANGE

2019 9. (a) 1 and 2 only


10. (b) 2 and 3 only
1. (b) 2 and 3 only 11. (b) 2 only
2. (d) textile fibre 12. (d) 1 and 3
3. (d) 1, 2 and 3 13. (b) 2 only
4. (d) 1, 2, 3 and 4 14. (a) Increased plant growth due to
increased concentration of carbon
5. (a) pesticides in agriculture
dioxide in the atmosphere
6. *
15. (b) 2 only
7. (a) 1 only
8. (d) Reducing the global warming
9. (a) Neyyar, Peppara and Shendurney 2017
Wildlife Sanctuaries; and Kalakad
Mundanthurai Tiger Reserve 1. (a) Sathyamangalam Tiger Reserve
10. (d) 1, 2, 3 and 4 2. (b) 2 only
11. (c) 1 and 3 only 3. (d) 1, 2 and 3
12. (a) They are considered harmful to 4. (d) 1, 2 and 3
marine ecosystems. 5. (d) 1, 2 and 3
13. (b) an early human species 6. (d) 1, 2 and 3
14. (a) 1 only 7. (a) 1 and 2 only
15. (b) 2 only 8. (c) Pollution assay in aquatic
16. (c) The Rules provide for exact and ecosystems
elaborate criteria for the identification 9. (b) Chambal River
of sites for landfills and waste 10. (a) It enjoys the same level of protection
processing facilities. as the tiger.
17. (c) The (e-Waste) (Management and 11. (b) Kuno Palpur Wildlife Sanctuary
Handling) Rules, 2011

2016
2018
1. (b) 2 only
1. (b) 2 and 3 only
2. (c) 3 only
2. (b) The United Nations Conference on
3. (c) 3 only
Sustainable Development 2012, Rio
de Janeiro 4. (a) production and use of solar energy
by the households/consumers
3. (b) It tends to reduce the biodiversity in
the area in which it grows. 5. (a) 1 and 2 only
4. (d) 1, 2 and 3 6. (a) Andaman Islands
5. (c) The UNFCCC Secretariat 7. (c) 2 and 3 only
6. (c) 2, 4 and 5 8. (d) 1, 2 and 3
7. (d) Mankind’s over-exploitation/misuse 9. (a) It is an international accounting tool
of natural resources, for government and business leaders
fragmentation/loss of natural to understand, quantify and manage
habitats, destruction of ecosystems, greenhouse gas emissions
pollution and global climate change. 10. (d) protection of Western Ghats
8. (a) Arunachal Pradesh 11. (d) 1, 2 and 3

20

https://telegram.me/CivilServices_UPSC
https://t.me/UPSC_PDF

GENERAL ISSUES ON ENVIRONMENTAL ECOLOGY, BIO-DIVERSITY AND CLIMATE CHANGE

12. (a) 1 only 17 (b) 2 only


13. (b) 2, 3 and 4 only 18 (a) 1, 2 and 3 only
14. (d) Maharashtra 19 (a) 1 only
15. (b) 2 only 20 (a) biosphere reserves
16. (b) plan of action outlined by the 21 (b) 1 and 2 only
countries of the world to combat
climate change
2013
17. (c) 1 and 3 only
18. (d) Western Ghats
19. (a) 1 and 2 only 1 (b) 1, 2, 3, 5 and 6 only
2 (d) nitrous oxide and sulphur dioxide
3 (a) 1 only
2015
4 (c) 1, 3 and 5 only
5 (c) water limits and fire
1. (c) 1 and 3 only 6 (d) Oceans, mangroves, takes,
2. (a) Tropical moist deciduous forest grasslands
3. (c) community of organisms together 7 (b) 2 and 3 only
with the environment in which they
live
2012
4. (b) Keibul Lamjao National Park
5. (b) 2 and 3 only
6. (d) Andaman and Nicobar Islands 1 (b) 1, 3 and 4 only
7. (b) Montreal Protocol 2 (d) A definite dry and wet season
8. (c) 1 and 3 only 3 (b) National Parks
4 (c) Nutrient cycling and crop pollination
5 (a) 1, 2 and 3 only
2014 6 (b) 1, 3 and 4 only
7 (c) 1, 2 and 3
1 (b) algae and fungi 8 (d) 1, 2 and 3
2 (a) 1 and 2 only 9 (b) a drug used by cattle owners for
3 (d) 1, 2, 3 and 4 treating their diseased cattle
4 (d) Neither 1 nor 2 10 (c) 1 and 3 only
5 (b) 2 and 3 only 11 (c) 1 and 3 only
6 (d) 1, 2 and 3
7 (d) 1, 2 and 3 2011
8 (c) 2 and 3 only
9 (a) Changes in ecological character 1. (b) 1, 3 and 4 only
have occurred, are occurring or are 2. (a) Bhitarkanika
likely to occur in the wetland as a 3. (d) Carbon credits are traded at a price
result of human interference fixed from time to time by the United
10 (c) 2 and 3 only Nations Environment Programme
11 (a) It serves as financial mechanism for 4. (c) 1, 3 and 5
‘Convention on Biological Diversity’ 5. (c) 1, 2, 3 and 4
and ‘United Nations Framework 6. (b) 1 and 2 only
Convention on Climate Change’ 7. (b) Indian wild ass.
12 (b) 2 only 8. (d) 1, 2, 3 and 4
9. (c) Both 1 and 2
13 (c) 1, 3 and 4 only
14 (c) Both 1 and 2
15 (c) 1, 2 and 3
16 (c) 1, 2 and 4 only

21

https://telegram.me/CivilServices_UPSC
https://t.me/UPSC_PDF

UPSC
PRELIMINARY EXAMINATION
(PAPER-I)
GENERAL STUDIES

PREVIOUS YEARS QUESTIONS

(YEAR 2010 TO 2019)

THEME  GENERAL SCIENCE

https://telegram.me/CivilServices_UPSC
https://t.me/UPSC_PDF

GENERAL SCIENCE

4. With reference to the recent developments in


2019
science, which one of the following
statements is not correct?
1. For the measurement/estimation of which of (a) Functional chromosomes can be created
the following are satellite images/remote by joining segments of DNA taken from
sensing data used? cells of different species.
1. Chlorophyll content in the vegetation (b) Pieces of artificial functional DNA can be
of a specific location created in laboratories.
2. Greenhouse gas emissions from rice (c) A piece of DNA taken out from an animal
paddies of a specific location cell can be made to replicate outside a
3. Land surface temperatures of a specific living cell in a laboratory.
location (d) Cells taken out from plants and animals
Select the correct answer using the code can be made to undergo cell division in
given below. laboratory petri dishes.
(a) 1 only
(b) 2 and 3 only 5. Consider the following statements:
(c) 3 only A digital signature is
(d) 1, 2 and 3 1. an electronic record that identifies the
certifying authority issuing it
2. In the context of which one of the following 2. used to serve as a proof of identity of an
are the terms "pyrolysis and plasma individual to access information or server
gasification' mentioned? on Internet
(a) Extraction of rare earth elements 3. an electronic method of signing an
(b) Natural gas extraction technologies electronic document and ensuring that
the original content is unchanged
(c) Hydrogen fuel-based automobiles
Which of the statements given above is/are
(d) Waste-to-energy technologies
correct?
(a) 1 only
3. In the context of digital technologies for
(b) 2 and 3 only
entertainment, consider the following
statements: (c) 3 only
1. In Augmented Reality (AR), a simulated (d) 1, 2 and 3
environment is created and the physical
world is completely shut out. 6. In the context of wearable technology, which
2. In Virtual Reality (VR), images generated of the following tasks is/are accomplished by
from a computer are projected onto real- wearable devices?
life objects or surroundings. 1. Location identification of a person
3. AR allows individuals to be present in the 2. Sleep monitoring of a person
world and improves the experience using 3. Assisting the hearing impaired person
the camera of smart-phone or PC. Select the correct answer using the code
4. VR closes the world, and transposes an given below.
individual, providing complete immersion (a) 1 only
experience.
(b) 2 and 3 only
Which of the statements given above is/are (c) 3 only
correct?
(d) 1, 2 and 3
(a) 1 and 2 only
(b) 3 and 4
7. 'RNA interference (RNAi)' technology has
(c) 1, 2 and 3 gained popularity in the last few years. Why?
(d) 4 only 1. It is used in developing gene silencing
therapies.

https://telegram.me/CivilServices_UPSC
https://t.me/UPSC_PDF

GENERAL SCIENCE

2. It can be used in developing therapies for (d) A herbicidal substance synthesized in


the treatment of cancer. genetically modified crops
3. It can be used to develop hormone
replacement therapies. 11. Which one of the following statements is not
4. It can be used to produce crop plants that correct?
are resistant to viral pathogens. (a) Hepatitis B virus is transmitted much like
Select the correct answer using the code HIV.
given below. (b) Hepatitis B, unlike Hepatitis C, does not
(a) 1, 2 and 4 have a vaccine.
(b) 2 and 3 (c) Globally, the number of people infected
(c) 1 and 3 with Hepatitis B and C viruses are
several times more than those infected
(d) 1 and 4 only
with HIV.
(d) Some of those infected with Hepatitis B
8. Recently, scientists observed the merger of
and C viruses do not show the symptoms
giant 'blackholes' billions of light-years away
for many years.
from the Earth. What is the significance of
this observation?
12. With reference to communication
(a) 'Higgs boson particles' were detected.
technologies, what is/are the
(b) 'Gravitational waves' were detected.
difference/differences between LTE (Long-
(c) Possibility of inter-galactic space travel Term Evolution) and VoLTE (Voice over
through Svormhole' was confirmed. Long-Term Evolution)?
(d) It enabled the scientists to understand 1. LTE is commonly marketed as 3G and
'singularity'. VoLTE is commonly marketed as
advanced 3G.
9. Which of the following are the reasons for the 2. LTE is data-only technology and VoLTE
occurrence of multi-drug resistance in is voice-only technology
microbial pathogens in India?
Select the correct answer using the code
1. Genetic predisposition of some given below.
people
(a) 1 only
2. Taking incorrect doses of antibiotics to
(b) 2 only
cure diseases
(c) Both 1 and 2
3. Using antibiotics in livestock
(d) Neither 1 nor 2
farming
4. Multiple chronic diseases in some people
Select the correct answer using the code 2018
given below.
(a) 1 and 2 1. "3D printing" has applications in which of the
(b) 2 and 3 only following ?
(c) 1, 3 and 4 1. Preparation of confectionery items
(d) 2, 3 and 4 2. Manufacture of bionic ears
3. Automotive industry
10. What is Cas9 protein that is often 4. Reconstructive surgeries
mentioned in news?
5. Data processing technologies
(a) A molecular scissors used in targeted
Select the correct answer using the code
gene editing
given below :
(b) A biosensor used in the accurate
(a) 1, 3 and 4 only
detection of pathogens in patients
(b) 2, 3 and 5 only
(c) A gene that makes plants pest-resistant
(c) 1 and 4 only
(d) 1, 2, 3, 4 and 5

https://telegram.me/CivilServices_UPSC
https://t.me/UPSC_PDF

GENERAL SCIENCE

using solid rocket motors; and the second


2. In which of the following areas can GPS and fourth stages using liquid rocket
technology be used ? engines.
1. Mobile phone operations Which of the statements given above is/ are
2. Banking operations correct ?
3. Controlling the power grids (a) 1 only
Select the correct answer using the code (b) 2 and 3
given below : (c) 1 and 2
(a) 1 only (d) 3 only
(b) 2 and 3 only
(c) 1 and 3 only 6. What is "Terminal High Altitude Area Defense
(d) 1, 2 and 3 (THAAD)", sometimes seen in the news ?
(a) An Israeli radar system
3. Consider the following statements : (b) India's indigenous antimissile programme
1. The Earth's magnetic field has reversed (c) An American anti-missile system
every few hundred thousand years. (d) A defence collaboration between Japan
2. When the Earth was created more than and South Korea
4000 million years ago, there was 54%
oxygen and no carbon dioxide. 7. With reference to the Indian Regional
3. When living organisms originated, they Navigation Satellite System (IRNSS),
modified the early atmosphere of the consider the following statements :
Earth. 1. IRNSS has three satellites in
Which of the statements given above is/ are geostationary and four satellites in
correct ? geosynchronous orbits.
(a) 1 only 2. IRNSS covers entire India and about
(b) 2 and 3 only 5500 sq. km beyond its borders.
(c) 1 and 3 only 3. India will have its own satellite navigation
system with full global coverage by the
(d) 1, 2 and 3
middle of 2019.
Which of the statements given above is/ are
4. The terms 'Wanna Cry, Petya and Eternal
correct ?
Blue' sometimes mentioned in the news
(a) 1 only
recently are related to
(b) 1 and 2 only
(a) Exoplanets
(c) 2 and 3 only
(b) Cryptocurrency
(d) None
(c) Cyber attacks
(d) Mini satellites
8. Consider the following phenomena :
1. Light is affected by gravity.
5. With reference to India's satellite launch
vehicles, consider the following statements : 2. The Universe is constantly expanding.
1. PSLVs launch the satellites useful for 3. Matter warps its surrounding space-
Earth resources monitoring whereas time.
GSLVs are designed mainly to launch Which of the above is/are the
communication satellites. prediction/predictions of Albert Einstein's
2. Satellites launched by PSLV appear to General Theory of Relativity, often discussed
remain permanently fixed in the same in media ?
position in the sky, as viewed from a (a) 1 and 2 only
particular location on Earth. (b) 3 only
3. GSLV Mk III is a four-staged launch (c) 1 and 3 only
vehicle with the first and third stages (d) 1, 2 and 3

https://telegram.me/CivilServices_UPSC
https://t.me/UPSC_PDF

GENERAL SCIENCE

automatically. On your way to office, your car


9. With reference to the Genetically Modified warns you about traffic congestion ahead and
mustard (GM mustard) developed in India, suggests an alternative route, and if you are
consider the following statements: late for a meeting, it sends a message to your
1. GM mustard has the genes of a soil office accordingly.
bacterium that give the plant the property In the context of emerging communication
of pest-resistance to a wide variety of technologies, which one of the following
pests. terms best applies to the above scenario?
2. GM mustard has the genes that allow the (a) Border Gateway Protocol
plant cross-pollination and hybridization. (b) Internet of Things
3. GM mustard has been developed jointly (c) Internet Protocol
by the IARI and Punjab Agricultural (d) Virtual Private Network
University.
Which of the statements given above is/ are 12. India enacted The Geographical Indications
correct ? of Goods (Registration and Protection) Act,
(a) 1 and 3 only 1999 in order to comply with the obligations
(b) 2 only to
(c) 2 and 3 only (a) ILO
(d) 1, 2 and 3 (b) IMF
(c) UNCTAD
10. Consider the following pairs (d) WTO
Terms Context /
sometimes seen Topic 13. Consider the following :
in news 1. Birds
1. Belle II experiment - Artificial 2. Dust blowing
Intelligence
3. Rain
2. Blockchain technology - Digital/
Cryptocurrency 4. Wind blowing
3. CRISPR - Cas9 - Particle Which of the above spread plant diseases?
Physics (a) 1 and 3 only
Which of the pairs given above is/are (b) 3 and 4 only
correctly matched ? (c) 1, 2 and 4 only
(a) 1 and 3 only (d) 1, 2, 3 and 4
(b) 2 only
(c) 2 and 3 only 14. Which of the following leaf modifications
(d) 1, 2 and 3 occur(s) in the desert areas to inhibit water
loss?
11. When the alarm of your smartphone rings in 1. Hard and waxy leaves
the morning, you wake up and tap it to stop 2. Tiny leaves
the alarm which causes your geyser to be 3. Thorns instead of leaves
switched on automatically. The smart mirror Select the correct answer using the code
in your bathroom shows the day's weather given below:
and also indicates the level of water in your (a) 2 and 3 only
overhead tank. After you take some groceries
(b) 2 only
from your refrigerator for making breakfast, it
(c) 3 only
recognises the shortage of stock in it and
places an order for the supply of fresh (d) 1, 2 and 3
grocery items. When you step out of your
house and lock the door, all lights, fans,
geysers and AC machines get switched off

https://telegram.me/CivilServices_UPSC
https://t.me/UPSC_PDF

GENERAL SCIENCE

(a) To detect neutrinos


2017
(b) To detect gravitational waves
(c) To detect the effectiveness of missile
1. The terms 'Event Horizon', 'Singularity', defence system
'String Theory' and 'Standard Model' are
(d) To study the effect of solar flares on our
sometimes seen in the news in the context of
communication systems
(a) Observation and understanding of the
Universe
5. It is possible to produce algae based biofuels,
(b) Study of the solar and the lunar eclipses
but what is/are the likely limitation(s) of
(c) Placing satellites in the orbit of the Earth
developing countries in promoting this
(d) Origin and evolution of living organisms industry ?
on the Earth
1. Production of algae based biofuels is
possible in seas only and not on
2. Consider the following pairs : continents.
Commonly used I Unwanted or 2. Setting up and engineering the algae
consumed controversial based biofuel production requires high
materials chemicals likely to level of expertise/ technology until the
be found in them construction is completed.
1. Lipstick - Lead 3. Economically viable production
2. Soft drinks - Brominated necessitates the setting up of large scale
vegetable oils facilities which may raise ecological and
3. Chinese fast food - Monosodium social concerns.
glutamate Select the correct answer using the code
Which of the pairs given above is/are given below :
correctly matched ? (a) 1 and 2 only
(a) 1 only (b) 2 and 3 only
(b) 2 and 3 only (c) 3 only
(c) 1 and 3 only (d) 1, 2 and 3
(d) 1, 2 and 3

6. Consider the following statements :


3. Organic Light Emitting Diodes (OLEDs) are 1. In tropical regions, Zika virus disease is
used to create digital display in many transmitted by the same mosquito that
devices. What are the advantages of OLED transmits dengue.
displays over Liquid Crystal displays ?
2. Sexual transmission of Zika virus
1. OLED displays can be fabricated on disease is possible.
flexible plastic substrates.
Which of the statements given above is/ are
2. Roll-up displays embedded in clothing correct ?
can be made using OLEDs.
(a) 1 only
3. Transparent displays are possible using
(b) 2 only
OLEDs.
(c) Both 1 and 2
Select the correct answer using the code
given below: (d) Neither 1 nor 2
(a) 1 and 3 only
(b) 2 only 7. With reference to agriculture in India, how
can the technique of 'genome sequencing',
(c) 1, 2 and 3
often seen in the news, be used in the
(d) None of the above statements is correct immediate future ?
1. Genome sequencing can be used to
4. What is the purpose of 'evolved Laser identify genetic markers for disease
Interferometer Space Antenna (eLISA)
project ?

https://telegram.me/CivilServices_UPSC
https://t.me/UPSC_PDF

GENERAL SCIENCE

resistance and -drought tolerance in (b) wireless communication technology


various crop plants. (c) solar power production technology
2. This technique helps in reducing the time (d) water conservation technology
required to develop new varieties of crop
plants.
4. Which one of the following is the best
3. It can be used to decipher the host- description of 'INS Astradharini', that was in
pathogen relationships in crops. the news recently?
Select the correct answer using the code (a) Amphibious warfare ship
given below :
(b) Nuclear-powered submarine
(a) 1 only
(c) Torpedo launch and recovery vessel
(b) 2 and 3 only
(d) Nuclear-powered aircraft carrier
(c) 1 and 3 only
(d) 1, 2 and 3
5. What is 'Greased Lightning-10 (GL-10)',
recently in the news?
8. What is the application of Somatic Cell (a) Electric plane tested by NASA
Nuclear Transfer Technology ?
(b) Solar-powered two-seater aircraft
(a) Production of biolarvicides designed by Japan
(b) Manufacture of biodegradable plastics (c) Space observatory launched by China
(c) Reproductive cloning of animals (d) Reusable rocket designed by ISRO
(d) Production of organisms free of diseases
6. Which of the following statements is/are
2016 correct? Viruses can infect
1. bacteria
2. fungi
1. In the context of the developments in
Bioinformatics, the term 'transcriptome', 3. plants
sometimes seen in the news, refers to Select the correct answer using the code
(a) a range of enzymes used in genome given below.
editing (a) 1 and 2 only
(b) the full range of mRNA molecules (b) 3 only
expressed by an organism (c) 1 and 3 only
(c) the description of the mechanism of gene (d) 1, 2 and 3
expression
(d) a mechanism of genetic mutations taking 7. With reference to 'Bitcoins', sometimes seen
place in cells in the news, which of the following
statements is/are correct?
2. "Mission Indradhanush" launched by the 1. Bitcoins are tracked by the Central Banks
Government of India pertains to of the countries.
(a) immunization of children and pregnant 2. Anyone with a Bitcoin address can send
women and receive Bitcoins from anyone else
(b) construction of smart cities across the with a Bitcoin address.
country 3. Online payments can be sent without
(c) India's own search for the Earthlike either side knowing the identity of the
planets in outer space other.
(d) New Educational Policy Select the correct answer using the code
given below.
3. 'Project Loon', sometimes seen in the news, (a) 1 and 2 only
is related to (b) 2 and 3 only
(a) waste management technology (c) 3 only

https://telegram.me/CivilServices_UPSC
https://t.me/UPSC_PDF

GENERAL SCIENCE

(d) 1, 2 and 3 2. made India the second country to have a


spacecraft orbit the Mars after USA
8. India is an important member of the 3. made India the only country to be
International Thermonuclear Experimental successful in making its spacecraft orbit
Reactor'. If this experiment succeeds, what is the Mars in its very first attempt
the immediate advantage for India? Which of the statements given above is/ are
(a) It can use thorium in place of uranium for correct?
power generation (a) 1 only
(b) It can attain a global role in satellite (b) 2 and 3 only
navigation (c) 1 and 3 only
(c) It can drastically improve the efficiency of (d) 1, 2 and 3
its fission reactors in power generation
(d) It can build fusion reactors for power 12. With reference to 'LiFi', recently in the news,
generation which of the following statements is/are
correct?
9. Regarding 'DigiLocker', sometimes seen in 1. It uses light as the medium for high-
the news, which of the following statements speed data transmission.
is/are correct? 2. It is a wireless technology and is several
1. It is a digital locker system offered by the times faster than WiFi'.
Government under Digital India Select the correct answer using the code
Programme. given below.
2. It allows you to access your e-documents (a) 1 only
irrespective of your physical location.
(b) 2 only
Select the correct answer using the code
(c) Both 1 and 2
given below.
(d) Neither 1 nor 2
(a) 1 only
(b) 2 only
13. Why does the Government of India promote
(c) Both 1 and 2
the use of 'Neem-coated Urea' in agriculture?
(d) Neither 1 nor 2
(a) Release of Neem oil in the soil increases
nitrogen fixation by the soil
10. With reference to 'Astrosat', the astronomical microorganisms
observatory launched by India, which of the (b) Neem coating slows down the rate of
following statements is/are correct? dissolution of urea in the soil
1. Other than USA and Russia, India is the (c) Nitrous oxide, which is a greenhouse
only country to have launched a similar gas, is not at all released into
observatory into space. atmosphere by crop fields
2. Astrosat is a 2000 kg satellite placed in (d) Six is a combination of a weedicide and a
an orbit at 1650 km above the surface of fertilizer for particular crops
the Earth.
Select the correct answer using the code
given below.
(a) 1 only 2015
(b) 2 only
(c) Both 1 and 2 1. H1N1 virus is sometimes mentioned in the
(d) Neither 1 nor 2 news with reference to which one of the
following diseases?
11. Consider the following statements : (a) AIDS
1. The Mangalyaan launched by ISRO is (b) Bird flu
also called the Mars Orbiter Mission (c) Dengue

https://telegram.me/CivilServices_UPSC
https://t.me/UPSC_PDF

GENERAL SCIENCE

(d) Swine flu (b) 2 only


(c) 2 and 3 only
2. With reference to bio-toilets used by the (d) 2 and 3
Indian Railways, consider the following
statements: 5. Among the following, which were frequently
(1) The decomposition of human waste in mentioned in the news for the outbreak of
the bio-toilets is initiated by a fungal Ebola virus recently?
inoculum. (a) Syria and Jordan
(2) Ammonia and water vapour are the only (b) Guinea, Sierra Leone and Liberia
end products in this decomposition which
(c) Philippines and Papua New Guinea
are released into the atmosphere.
(d) Jamaica, Haiti and Surinam
Which of the statements given above is/ are
correct?
6. With reference to 'fly ash' produced by the
(a) 1 only
power plants using coal as fuel, which of the
(b) 2 only
following statements is/are correct?-
(c) Both 1 and 2
1. Fly ash can be used in the production of
(d) Neither 1 nor 2 bricks for building construction.
2. Fly ash can be used as a replacement for
3. With reference to "fuel cells" in which some of the Portland cement contents of
hydrogen-rich fuel and oxygen are used to concrete.
generate electricity, consider the following 3. Fly ash is made up of silicon dioxide and
statements : calcium oxide only, and does not contain
(1) If pure hydrogen is used as a fuel, the any toxic elements.
fuel cell emits heat and water as by- Select the correct answer using the code
products. given below.
(2) Fuel cells can be used for powering (a) 1 and 2
buildings and not for small devices like
(b) 2 only
laptop computers.
(c) 1 and 3
(3) Fuel cells produce electricity in the form
(d) 3 only
of Alternating Current (AC).
Which of the statements given above is/ are
correct? 7. With reference to 'dugong', a mammal found
in India, which of the following statements
(a) 1 only
is/are correct?
(b) 2 and 3 only
1. It is a herbivorous marine animal.
(c) 1 and 3 only
2. It is found along the entire coast of India.
(d) 1, 2 and 3
3. It is given legal protection under
Schedule I of the Wildlife (Protection) Act,
4. What can be the impact of excessive/ 1972.
inappropriate use of nitrogenous fertilizers in
Select the correct answer using the code
agriculture?
given below.
1. Proliferation of nitrogen-fixing
(a) 1 and 2
microorganisms in soil can occur.
(b) 2 only
2. Increase in the acidity of soil can take
(c) 1 and 3
place.
(d) 3 only
3. Leaching of nitrate to the groundwater
can occur.
Select the correct answer using the code 8. With reference to the use of nano-technology
given below. in health sector, which of the following
statements is/are correct?
(a) 1 and 3 only

https://telegram.me/CivilServices_UPSC
https://t.me/UPSC_PDF

GENERAL SCIENCE

(1) Targeted drug delivery is made possible (b) The gum made from its seeds is used in
by nanotechnology. the extraction of shale gas
(2) Nanotechnology can largely contribute to (c) The leaf extract of this plant has the
gene therapy. properties of antihistamines
Select the correct answer using the code (d) It is a source of high quality biodiesel.
given below.
(a) 1 only 2. Consider the following pairs :
(b) 2 only Vitamin Deficiency
(c) Both 1 and 2 disease
(d) Neither 1 nor 2 1. Vitamin C : Scurvy
2. Vitamin D : Rickets
9. Which one of the following is the national
3. Vitamin E : Night
aquatic animal of India?
blindness
(a) Saltwater crocodile
Which of the pairs given above is/are
(b) Olive ridley turtle
correctly matched?
(c) Gangetic dolphin
(a) 1 and 2 only
(d) Gharial
(b) 3 only
(c) 1, 2 and 3
10. With reference to 'Near Field Communication
(d) None
(NFC) Technology', which of the following
statements is/are correct?
(1) It is a contactless communication 3. With reference to Neem tree, consider the
technology that uses electromagnetic following statements:
radio fields. 1. Neem oil can be used as a pesticide to
(2) NFC is designed for use by devices control the proliferation of some species
which can be at a distance of even a of insects and mites.
metre from each other. 2. Neem seeds are used in the manufacture
(3) NFC can use encryption when sending of biofuels and hospital detergents.
sensitive information. 3. Neem oil has applications in
Select the correct answer using the code pharmaceutical industry.
given below. Which of the statements given above is/ are
(a) 1 and 2 only correct?
(b) 3 only (a) 1 and 2 only
(c) 1 and 3 only (b) 3 only
(d) 1, 2 and 3 (c) 1 and 3 only
(d) 1, 2 and 3

4. Which one of the following is the process


2014 involved in photosynthesis?
(a) Potential energy is released to form free
1. In India, cluster bean (Guar) is traditionally energy
used as a vegetable or animal feed, but (b) Free energy is converted into potential
recently the cultivation of this has assumed energy and stored
significance. Which one of the following (c) Food is oxidized to release carbon
statements is correct in this context? dioxide and water
(a) The oil extracted from seeds is used in (d) Oxygen is taken, and carbon dioxide and
the manufacture of biodegradable water vapour are given out
plastics

10

https://telegram.me/CivilServices_UPSC
https://t.me/UPSC_PDF

GENERAL SCIENCE

5. In addition to fingerprint scanning, which of (a) 1 only


the following can be used in the biometric (b) 2 and 3
identification of a person? (c) 1 and 3
1. Ins scanning (d) None
2. Retinal scanning
3. Voice recognition 9. Consider the following statements
Select the correct answer using the code 1. Maize can be used for the production of
given below. starch.
(a) 1 only 2. Oil extracted from maize can be a
(b) 2 and 3 only feedstock for biodiesel.
(c) 1 and 3 only 3. Alcoholic beverages can be produced by
(d) 1, 2 and 3 using maize.
Which of the statements given above is/ are
6. Which of the following statements is/are correct?
correct regarding vegetative propagation of (a) 1 only
plants ? (b) 1 and 2 only
1. Vegetative propagation produces clonal (c) 2 and 3 only
population. (d) 1, 2 and 3
2. Vegetative propagation helps in
eliminating the virus,
10. Among the following organisms, which one
3. Vegetative propagation can be practiced does not belong to the class of other three?
most of the year.
(a) Crab
Select the correct answer using the code
(b) Mite
given below.
(c) Scorpion
(a) 1 only
(d) Spider
(b) 2 and 3 only
(c) 1 and 3 only
11. Which one of the following is the correct
(d) 1, 2 and 3
sequence of a food chain?
(a) Diatoms—Crustaceans—Herrings
7. Which of the following is/are the
(b) Crustaceans—Diatoms—Herrings
example/examples of chemical change?
(c) Diatoms—Herrings—Crustaceans
1. Crystallization of sodium chloride
(d) Crustaceans—Herrings—Diatoms
2. Melting of ice
3. Souring of milk
12. With reference to technologies for solar
Select the correct answer using the code
power production, consider the following
given below.
statements :
(a) 1 and 2 only
1. 'Photovoltaics' is a technology that
(b) 3 only generates electricity by direct conversion
(c) 1, 2 and 3 of light into electricity, whil e 'Solar
(d) None Thermal' is a technology that utilizes the
Sun's rays to generate heat which is
8. Consider the following techniques/ further used in electricity generation
phenomena process.
1. Budding and grafting in fruit plants 2. Photovoltaics generates Alternating
Current (AC), while Solar Thermal
2. Cytoplasmic male sterility
generates Direct Current (DC).
3. Gene silencing
3. India has manufacturing base for Solar
Which of the above is/are used to create
Thermal technology, but not for
transgenic crops ?
Photovoltaics.

11

https://telegram.me/CivilServices_UPSC
https://t.me/UPSC_PDF

GENERAL SCIENCE

Which of the statements given above is/ are (a) Ecotone


correct? (b) Ecological niche
(a) 1 only (c) Habitat
(b) 2 and 3 only (d) Home range
(c) 1, 2 and 3
(d) None 4. Photochemical smog is a resultant of the
reaction among
13. Consider the following diseases (a) NO2, O3 and peroxyacetyl nitrate in the
1. Diphtheria presence of sunlight
2. Chickenpox (b) CO, O2 and peroxyacetyl nitrate in the
3. Smallpox presence of sunlight
Which of the above diseases has/have been (c) CO, CO2 and NO2 at low temperature
eradicated in India? (d) high concentration of NO3, O3 and CO in
(a) 1 and 2 only the evening
(b) 3 only
(c) 1, 2 and 3 5. Consider the following minerals
(d) None 1. Calcium
2. Iron
3. Sodium
2013 Which of the minerals given above is/are
required by human body for the contraction of
1. Improper handling and storage of cereal muscles?
grains and oilseeds result in the production of (a) 1 only
toxins known as aflatoxins which are not (b) 2 and 3 only
generally destroyed by normal cooking (c) 1 and 3 only
process. Aflatoxias are produced by
(d) 1, 2 and 3
(a) bacteria
(b) protozoa
6. Recombinant DNA technology' (Genetic
(c) moulds
Engineering) allows genes to be transferred
(d) viruses 1. across different species of plants
2. from animals to plants
2. Which of the following diseases can be 3. from microorganisms to higher organisms
transmitted from one person to another
Select the correct answer using the codes
through tattooing?
given below.
1. Chikungunya
(a) 1 only
2. Hepatitis B
(b) 2 and 3 only
3. NW-AIDS
(c) 1 and 3 only
Select the correct answer using the codes
(d) 1, 2 and 3
given below.
(a) 1 only
7. Fruits stored in a cold chamber exhibit longer
(b) 2 and 3 only
storage life because
(c) 1 and 3 only
(a) exosure to sunlight is prevented
(d) 1, 2 and 3
(b) concentration of carbon dioxide in the
environment is increased
3. Which one of the following terms describes (c) rate of respiration is decreased
not only the physical space occupied by an
(d) there is an increase in humidity
organism, but also its functional role in the
community of organisms?

12

https://telegram.me/CivilServices_UPSC
https://t.me/UPSC_PDF

GENERAL SCIENCE

8. Ball bearings are used in bicycles, cars, etc., 1. Viruses lack enzymes necessary for the
because generation of energy.
(a) the actual area of contact between the 2. Viruses can be cultured in any synthetic
wheel and axle is increased medium.
(b) the effective area of contact between the 3. Viruses are transmitted from one
wheel end axle is increased organism to another by biological vectors
(c) the effective area of contact between the only.
wheel and axle is reduced Select the correct answer using the codes
(d) None of the above statements is correct given below.
(a) 1 only
9. Consider the following phenomena: (b) 2 and 3 only
1. Size of the sun at dusk (c) 1 and 3 only
2. Colour of the sun at dawn (d) 1, 2 and 3
3. Moon being visible at dawn
4. Twinkle of stars in the sky 13. Which of the following leaf modifications
5. Polestar being visible in the sky occurs/occur in desert areas to inhibit water
loss?
Which of the above are optical illusions?
1. Hard and waxy leaves
(a) 1, 2 and 3
2. Tiny leaves or no leaves
(b) 3, 4 and 5
3. Thorns instead of leaves
(c) 1, 2 and 4
Select the correct answer using the codes
(d) 2, 3 and 5
given below.
(a) 1 and 2 only
10. Rainbow is produced when sunlight falls on
(b) 2 only
drops of rain. Which of the following physical
phenomena are responsible for this? (c) 1 and 3 only
1. Dispersion (d) 1, 2 and 3
2. Refraction
3. Internal reflection 14. The known forces of nature can be divided
into four classes, viz., gravity,
Select the correct answer using the codes
electromagnetism, weak nuclear force and
given below.
strong nuclear force. With reference to them,
(a) 1 and 2 only
which one of the following statements is not
(b) 2 and 3 only correct?
(c) 1 and 3 only (a) Gravity is the strongest of the four
(d) 1, 2 and 3 (b) Electromagnetism acts only on particles
with an electric charge
11. Many transplanted seedlings do not grow (c) Week nuclear force, causes radioactivity
because (d) Strong nuclear force holds protons' and
(a) the new soil does not contain favourable neutrons inside the nucleus of an atom
minerals four
(b) most of the root hairs grip the now soil
ton hard 15. The efforts to detect the existence 'of Higgs
(c) most of the root heirs are lost during boson particle have become frequent news in
transplantation the recent past. What is/are the importance/
(d) leaves get damaged during importance's of discovering this particle?
transplantation 1. It will enable us to understand as to why
elementary particles have mass.
12. Which of the following statements is/are 2. It will enable us in the near future to
correct? 'develop the technology of transferring

13

https://telegram.me/CivilServices_UPSC
https://t.me/UPSC_PDF

GENERAL SCIENCE

matter from one point to another without 2. The increasing amount of carbon dioxide in
traversing the physical space between the air is slowly raising the temperature of the
them. atmosphere, because it absorbs:
3. It will enable us to create better fuels for (a) the water vapour of the air and retains its
nuclear fission. heat
Select the correct answer using the codes (b) the ultraviolet part of the solar radiation
given below. (c) all the solar radiations
(a) 1 only (d) the infrared part of the solar radiation
(b) 2 and 3 only
(c) 1 and 3 only 3. Which one of the following sets of elements
(d) 1, 2 and 3 was primarily responsible for the origin of life
on the Earth?
16. Mycorrhizal biotechnology has been used in (a) Hydrogen, Oxygen, Sodium
rehabilitating degraded sites because (b) Carbon, Hydrogen, Nitrogen
mycorrhiza enables the plants to (c) Oxygen, Calcium, Phosphorus
1. resist drought and increase absorptive (d) Carbon, Hydrogen, Potassium
area
2. tolerate extremes of pH 4. What are the reasons for the people's
3. resist disease infestation resistance to the introduction of Bt brinjal in
Select the correct answer using the codes India?
given below. 1. Bt brinjal has been created by inserting a
(a) 1 only gene from a soil fungus into its genome.
(b) 2 and 3 only 2. The seeds of Bt brinjal are terminator
(c) 1 and 3 only seeds and therefore, the farmers have to
(d) 1, 2 and 3 buy the seeds before every season from
the seed companies.
3. There is an apprehension that the
2012 consumption of Bt brinjal may have
1. To meet its rapidly growing energy demand, adverse impact on health.
some opine that India should pursue 4. There is some concern that the
research and development on thorium as the introduction of Bt brinjal may have
future fuel of nuclear energy. In this context, adverse effect on the biodiversity.
what advantage does thorium hold over Select the correct answer using the codes
uranium? given below :
1. Thorium is far more abundant in nature (a) 1, 2 and 3 only
than uranium. (b) 2 and 3 only
2. On the basis of per unit mass of mined (c) 3 and 4 only
mineral, thorium can generate more (d) 1, 2, 3 and 4
energy compared to natural uranium.
3. Thorium produces less harmful waste
5. Other than resistance to pests, what are the
compared to uranium.
prospects for which genetically engineered
Which of the statements above is/are plants have been created?
correct?
1. To enable them to withstand drought
(a) 1 only
2. To increase the nutritive value of the
(b) 2 and 3 only produce
(c) 1 and 3 only
3. To enable them to grow and do
(d) 1, 2 and 3 photosynthesis in spaceships and space
stations
4. To increase their shelf life

14

https://telegram.me/CivilServices_UPSC
https://t.me/UPSC_PDF

https://telegram.me/CivilServices_UPSC
https://t.me/UPSC_PDF

https://telegram.me/CivilServices_UPSC
https://t.me/UPSC_PDF

GENERAL SCIENCE

Which of the statements given above is/ are (a) 1 and 2 only
correct? (b) 2 only
(a) 1 only (c) 1 and 3 only
(b) 2, 3 and 4 only (d) 1, 2 and 3
(c) 1 and 3 only
(d) 1, 2, 3 and 4 4. Microbial fuel cells are considered a source
of sustainable energy. Why ?
1. They use living organisms as catalysts to
2011
generate electricity from certain
substrates.
1. Consider the following: 2. They use a variety of inorganic materials
1. Carbon dioxide as substrates.
2. Oxides of Nitrogen 3. They can be installed in waste water
3. Oxides of Sulphur treatment plants to cleanse water and
Which of the above is/ are the emission/ produce electricity.
emissions from coal combustion at thermal Which of the statements given above is/ are
power plants? correct ?
(a) 1 only (a) 1 only
(b) 2 and 3 only (b) 2 and 3 only
(c) 1 and 3 only (c) 1 and 3 only
(d) 1, 2 and 3 (d) 1, 2 and 3

2. Satellites used for telecommunication relay 5. The formation of ozone hole in the Antarctic
are kept in a geostationary orbit. A satellite is region has been a cause of concern. What
said to be in such an orbit when : could be the reason for the formation of this
1. The orbit is geosynchronous. hole ?
2. The orbit is circular. (a) Presence of prominent tropospheric
turbulence; and inflow of
3. The orbit lies in the plane of the Earth's
chlorofluorocarbons
equator.
4. The orbit is at an altitude of 22,236 km. (b) Presence of prominent polar front and
stratospheric clouds; and inflow of
Select the correct answer using the codes
chlorofluorocarbons
given below:
(c) Absence of polar front and stratospheric
(a) 1, 2 and 3 only
clouds; and inflow of methane and
(b) 1, 3 and 4 only chlorofluorocarbons
(c) 2 and 4 only (d) Increased temperature at polar region
(d) 1, 2, 3 and 4 due to global warming.

3. At present, scientists can determine the 6. When the bark of a tree is removed in a
arrangement or relative positions of genes or circular fashion all around near its base, it
DNA sequences on a chromosome. How gradually dries up and dies because
does this knowledge benefit us? (a) Water from soil cannot rise to aerial parts
1. It is possible to know the pedigree of (b) Roots are starved of energy
livestock.
(c) Tree is infected by soil microbes
2. It is possible to understand the causes of
(d) Roots do not receive oxygen for
all human diseases.
respiration.
3. It is possible to develop disease-resistant
animal breeds.
7. Human activities in the recent past have
Which of the statements given above is/ are
caused the increased concentration of carbon
correct?

17

https://telegram.me/CivilServices_UPSC
https://t.me/UPSC_PDF

GENERAL SCIENCE

dioxide in the atmosphere, but a lot of it does 1. The food products are not made out of
not remain in the lower atmosphere because hydrogenated oils.
of 2. The food products are not made out of
1. its escape into the outer stratosphere. animal fats/oils.
2. the photosynthesis by phytoplankton in 3. The oils used are not likely to damage
the oceans. the cardiovascular health of the
3. the trapping of air in the polar ice caps. consumers.
Which of the statements given above is/ are Which of the statements given above is/ are
correct ? correct?
(a) 1 and 2 (a) 1 only
(b) 2 only (b) 2 and 3 only
(c) 2 and 3 (c) 1 and 3 only
(d) 3 only (d) 1, 2 and 3

8. A layer in the Earth's atmosphere called 12. Consider the following:


Ionosphere facilitates radio communication. 1. Photosynthesis
Why? 2. Respiration
1. The presence of ozone causes the 3. Decay of organic matter
reflection of radio waves to Earth. 4. Volcanic action
2. Radio waves have a very long Which of the above add carbon dioxide to the
wavelength. carbon cycle on Earth ?
Which of the statements given above is/ (a) 1 and 4 only
correct?
(b) 2 and 3 only
(a) 1 only
(c) 2, 3 and 4 only
(b) 2 only
(d) 1, 2, 3 and 4
(c) Both 1 and 2
(d) Neither 1 nor 2
13. The surface of a lake is frozen in severe
winter, but the water at its bottom is still
9. A genetically engineered form of brinjal, liquid. What is the reason ?
known as the Bt-brinjal, has been developed. (a) Ice is a bad conductor of heat
The objective of this is
(b) Since the surface of the lake is at the
(a) To make it pest-resistant same temperature as the air, no heat is
(b) To improve its taste and nutritive qualities lost.
(c) To make it drought - reistant (c) The density of water is maximum at 40C.
(d) To make its shelf-life longer (d) None of the statements (a), (b) and (c)
given above is correct.
10. The function of heavy water in a nuclear
reactor is to 14. What is the difference between Bluetooth and
(a) Slow down the speed of neutrons. Wi-Fi devices ?
(b) Increase the speed of neutrons. (a) Bluetooth uses 2.4 GHz radio frequency
(c) Cool down the reactor band, whereas Wi-Fi can use 2.4 GHz or
(d) Stop the nuclear reaction. 5 GHz frequency band.
(b) Bluetooth is used for Wireless Local Area
Networks (WLAN) only, whereas Wi-Fi is
11. A company marketing food products
used for Wireless Wide Area Networks
advertises that its items do not contain trans-
(WWAN) only
fats. What does this campaign signify to the
customers? (c) When information is transmitted between
two devices using Bluetooth technology,
the devices have to be in the line of sight

18

https://telegram.me/CivilServices_UPSC
https://t.me/UPSC_PDF

GENERAL SCIENCE

of each other, but when Wi-Fi technology (d) It is the latest technology to control the
is used the devices need not be in the accidentally caused flames from oil wells
line of sight of each other.
(d) The statements (a) and (b) given above 18. A married couple adopted a male child. A few
are correct in this context. years later, twin boys were born to them. The
blood group of the couple is AB positive and
15. Aspartame is an artificial sweetener sold in O negative. The blood group of the three
the market. It consists of amino acids and sons is A positive, B positive, and O positive.
provides calories like other amino acids. Yet, The blood group of the adopted son is
it is used as a low-calorie sweetening agent (a) O positive
in food items. What is the basis of this use? (b) A positive
(a) Aspartame is as sweet as table sugar, (c) B positive
but unlike table sugar, it is not readily (d) Cannot be determined on the basis of the
oxidized in human body due to lack of given data
requisite enzymes
(b) When aspartame is used in food
19. A new optical disc format known as the Blu-
processing, the sweet taste remains, but
ray Disc (BD) is becoming popular. In what
it becomes resistant to oxidation
way is it different from the traditional DVD ?
(c) Aspartame is as sweet as sugar, but after
1. DVD supports Standard Definition video
ingestion into the body, it is converted
while BD supports High Definition video.
into metabolites that yield no calories
2. Compared to a DVD, the BD format has
(d) Aspartame is several times sweeter than
several times more storage capacity.
table sugar, hence food items made with
3. Thickness of BD is 2.4 mm while that of
small quantities of aspartame yield fewer
DVD is 1.2 mm.
calories on oxidation
Which of the statements given above is/ are
correct?
16. What is the difference between a CFL and an
(a) 1 only
LED lamp ?
(b) 1 and 2 only
1. To produce light, a CFL uses mercury
vapour and phosphor while an LED lamp (c) 2 and 3 only
uses semi-conductor material. (d) 1, 2 and 3
2. The average life span of a CFL is much
longer than that of an LED lamp. 20. Salinization occurs when the irrigation water
3. A CFL is less energy-efficient is accumulated in the soil evaporates, leaving
compared to an LED lamp. behind salts and minerals. What are the
Which of the statements given above is/ are effects of salinization on the irrigated land ?
correct? (a) It greatly increases the crop production.
(a) 1 only (b) It makes some soils impermeable
(b) 2 and 3 only (c) It raises the water table
(c) 1 and 3 only (d) It fills the air spaces in the soil with water.
(d) 1, 2 and 3
21. The "Red Data Books" published by the
17. Recently, "oilzapper" was in the news. What International Union for Conservation of
is it ? Nature and Natural Resources (IUCN)
contain lists of
(a) It is an eco-friendly technology for the
remediation of oily sludge and oil spills 1. Endemic plant and animal species
present in the biodiversity hotspots.
(b) It is the latest technology developed for
under-sea oil exploration 2. Threatened plant and animal species.
(c) It is a genetically engineered high biofuel-
yielding maize variety

19

https://telegram.me/CivilServices_UPSC
https://t.me/UPSC_PDF

GENERAL SCIENCE

3. Protected sites for conservation of nature (b) The prevent excessive oxidation of
and natural resources in various carbohydrates, fats and proteins in the
countries. body and help avoid unnecessary
Select the correct answer using the codes wastage of energy
given below : (c) They neutralize the free radicals
(a) 1 and 3 produced in the body during metabolism
(b) 2 only (d) They activate certain genes in the cells of
(c) 2 and 3 the body and help delay the ageing
process
(d) 3 only

25. The jet aircrafts fly very easily and smoothly


22. An artificial satellite orbiting around the Earth
in the lower stratosphere. What could be the
does not fall down. This is so because the
appropriate explanation?
attraction of Earth.
1. There are no clouds or water vapour in
(a) does not exist at such distance
the lower stratosphere.
(b) is neutralized by the attraction of the
2. There are no vertical winds in the lower
moon
stratosphere.
(c) provides the necessary speed for its
Which of the statements given above is/ are
steady motion
correct in this context?
(d) provides the necessary acceleration for
(a) 1 only
its motion
(b) 2 only
(c) Both 1 and 2
23. What is the difference between asteroids and
comets ? (d) Neither 1 nor 2
1. Asteroids are small rocky planetoids,
while comets are formed of frozen gases 26. What is "Virtual Private Network" ?
held together by rocky and metallic (a) It is a private computer network of an
material. organization where the remote users can
2. Asteroids are found mostly between the transmit encrypted information through
orbits of Jupiter and Mars, while comets the server of the organization
are found mostly between Venus and (b) It is a computer network across a public
Mercury. internet that provides users access to
3. Comets show a perceptible glowing tail, their organization's network while
while asteroids do not. maintaining the security of the
Which of the statements given above is/ are information transmitted
correct? (c) It is a computer network in which users
(a) 1 and 2 only can access a shared pool of computing
resources through a service provider
(b) 1 and 3 only
(d) None of the statements (a), (b) and (c)
(c) 3 only
given above is a correct description of
(d) 1, 2 and 3
Virtual Private Network

24. Regular intake of fresh fruits and vegetables


is recommended in the diet since they are a 2010
good source of antioxidants. How do
antioxidants help a person maintain health 1. If a potato is placed on a pure paper plate
and promote longevity? which is white and unprinted and put in a
(a) They activate the enzymes necessary for microwave oven, the potato heats up but the
vitamin synthesis in the body and help paper plate does not. This is because:
prevent vitamin deficiency

20

https://telegram.me/CivilServices_UPSC
https://t.me/UPSC_PDF

GENERAL SCIENCE

(a) Potato is mainly made up of starch (d) The formation of hydrogen when chlorine
whereas paper is mainly made up of is added to water
cellulose
(b) Potato transmits microwaves whereas 5. Consider the following :
paper reflects microwaves 1. Bluetooth device
(c) Potato contains water whereas paper 2. Cordless phone
does not contain water
3. Microwave oven
(d) Potato is a fresh organic material
4. Wi-Fi device
whereas paper is a dead organic material
Which of the above can operate between 2.4
and 2.5 GHz range of radio frequency band ?
2. Consider the following statements : The
(a) 1 and 2 only
satellite Oceansat-2 launched by India helps
(b) 3 and 4 only
in
(c) 1, 2 and 4 only
1. estimating the water vapour content in
the atmosphere. (d) 1, 2, 3 and 4
2. predicting the onset of monsoons.
3. monitoring the pollution of coastal waters. 6. Consider the following statements :
Which of the statements given above is/ are 1. Biodiversity hotspots are located only in
correct ? tropical regions.
(a) 1 and 2 only 2. India has four biodiversity hotspots i.e.,
Eastern Himalayas, Western Himalayas,
(b) 2 only
Western Ghats and Andaman and
(c) 1 and 3 only
Nicobar Islands.
(d) 1, 2 and 3
Which of the statements given above is/ are
correct?
3. Genetically modified "golden rice" has been (a) 1 only
engineered to meet human nutritional
(b) 2 only
requirements. Which one of the following
(c) Both 1 and 2
statements best qualifies golden rice?
(d) Neither 1 nor 2
(a) The grains have been fortified with genes
to provide three times higher grain yield
per acre than other high yielding varieties 7. Widespread resistance of malarial parasite to
(b) Its grains contain pro-vitamin A which drugs like chloroquine has prompted attempts
upon ingestion is converted to vitamin A to develop a malarial vaccine to combat
in the human body malaria. Why is it difficult to develop an
effective malaria vaccine ?
(c) Its modified genes cause the synthesis of
all the nine essential amino acids (a) Malaria is caused by several species of
Plasmodium
(d) Its modified genes cause the fortification
of its grains with vitamin D. (b) Man does not develop immunity to
malaria during natural infection
(c) Vaccines can be developed only against
4. Chlorination is a process used for water-
bacteria
purification. The disinfecting action of chlorine
is mainly due to (d) Man is only an intermediate host and not
the definitive host
(a) The formation of hydrochloric acid when
chlorine is added to, water
(b) The formation of hypochlorous acid when 8. A pesticide which is a chlorinated
chlorine is added to water hydrocarbon is sprayed on a food crop. The
food chain is: Food crop - Rat -Snake - Hawk.
(c) The formation of nascent oxygen when
In this food chain, the highest concentration
chlorine is added to water
of the pesticide would accumulate in which
one of the following?

21

https://telegram.me/CivilServices_UPSC
https://t.me/UPSC_PDF

GENERAL SCIENCE

(a) Food crop (b) Rat (b) 3 only


(c) Snake (d) Hawk (c) 2 and 3 only
(d) 1, 2 and 3
9. India-based Neutrino Observatory is included
by the Planning Commission as a mega 13. With reference to the mineral resources of
science project under the 11th Five Year India, consider the following pairs:
Plan. In this context, consider the following
Mineral 90% Natural sources
statements:
in
1. Neutrinos are chargeless elementary
particles that travel close to the speed of 1. Copper Jharkhand
light. 2. Nickel Orissa
2. Neutrinos are created in nuclear 3. Tungsten Kerala
reactions of beta decay.
Which of the pairs given above is/are
3. Neutrinos have a negligible, but nonzero
correctly matched?
mass.
(a) 1 and 2 only
4. Trillions of Neutrinos pass through
(b) 2 only
human body every second.
(c) 1 and 3 only
Which of the statements given above are
correct? (d) 1, 2 and 3
(a) 1 and 3 only
(b) 1, 2 and 3 only 14. The United Nations Framework Convention
on Climate Change (UNFCCC) is an
(c) 2, 3 and 4
international treaty drawn at
(d) 1, 2, 3 and 4
(a) United Nations Conference on the
Human Environment, Stockholm, 1972
10. Hydrogen fuel cell vehicles produce one of
(b) UN Conference on Environment and
the following as "exhaust"
Development, Rio de Janeiro, 1992
(a) NH3 (b) CH4
(c) World Summit on Sustainable
(c) H2O (d) H2 O 2 Development, Johannesburg, 2002
(d) UN Climate Change .Conference,
11. Recently, LASIK (Laser Assisted In Situ Copenhagen, 2009
Keratomileusis) procedure is being made
popular for vision correction. Which one of
15. Which bacterial strain, developed from
the following statements in this context is not
natural isolates by genetic manipulations, can
correct ?
be used for treating oil spills?
(a) LASIK procedure is used to correct
(a) Agrobacterium
refractive errors of the eye
(b) Clostridium
(b) It is a procedure that permanently
(c) Nitrosomonas
changes the shapes of the cornea
(d) Pseudomonas
(c) It reduces a person's dependence on
glasses or contact lenses
(d) It is a procedure that can be done on the 16. Other than Jatropha curcas, why is Pongamia
person of any age pinnata also considered a good option for the
production of bio-diesel in India ?
1. Pongamia pinnata grows naturally in
12. Consider the following:
most of the arid regions of India.
1. Oxides of Hydrogen
2. The seeds of Pongamia pinnata are rich
2. Oxides of Nitrogen
in lipid content of which nearly half is
3. Oxides of Sulphur oleic acid.
Which of the above causes/cause acid rain ? Which of the statements given above is/ are
(a) 1 and 2 only correct?

22

https://telegram.me/CivilServices_UPSC
https://t.me/UPSC_PDF

GENERAL SCIENCE

(a) 1 only (b) It delivers an extremely precise dose of


(b) 2 only radiation
(c) Both 1 and 2 (c) It has the capability of achieving sub-
(d) Neither 1 nor 2 millimetre accuracy
(d) It can map the spread of tumour in the
body
17. Some species of plants are insectivorous.
Why?
(a) Their growth in shady and dark places 22. King Cobra is the only snake that makes its
does not allow them to undertake own nest. Why does it make its nest?
sufficient photosynthesis and thus they (a) It is a snake-eater and the nest helps
depend on insects for nutrition. attract other snakes
(b) They are adapted to grow in nitrogen (b) It is a viviparous snake and needs a nest
deficient soils and thus depend on to give birth to its offspring
insects for sufficient nitrogenous nutrition (c) It is an oviparous snake and lays its eggs
(c) They cannot synthesize certain vitamins in the nest and guards the nest until they
themselves and depend on the insects are hatched
digested by them (d) It is a large, cold blooded animal and
(d) They have remained in that particular needs a nest to hibernate in the cold
stage of evolution as living fossils, a link season
between autotrophs and heterotrophs
23. Indiscriminate disposal of used fluorescent
18. Which one of the following reflects back more electric lamps causes mercury pollution in the
sunlight as compared to other three? environment. Why is. mercury used .in the
(a) Sand desert manufacture of these lamps ?
(b) Paddy crop land (a) A mercury coating on the inside of the
lamp makes the light bright white
(c) Land covered with fresh snow
(b) When the lamp is switched on, the
(d) Prairie land
mercury in the lamp causes the emission
of ultra-violet radiations
19. What is the principle by which a cooling
(c) When the lamp is switched on, it is the
system (Radiator) in a motor car works?
mercury which converts the ultra-violet
(a) Conduction only energy into visible light
(b) Convection (d) None of the statement given above is
(c) Radiation only correct about the use of mercury in the
(d) Both conduction and radiation manufacture of fluorescent lamps

20. Which among the following do/does not 24. Which one of the following processes in the
belong/belongs to the GSM family of wireless bodies of living organisms is a digestive
technologies ? process ?
(a) EDGE (a) Breakdown of proteins into amino acids
(b) LTE (b) Breakdown of glucose into CO2 and H2O
(c) DSL (c) Conversion of glucose into glycogen
(d) Both EDGE and LTE (d) Conversion of amino acids into proteins

21. With reference to the treatment of cancerous 25. From the point of view of evolution of living
tumours, a tool called cyberknife has been organisms, which one of the following is the
making the news. In this context, which one correct sequence of evolution ?
of the following statements is not correct ? (a) Otter -Tortoise - Shark
(a) It is a robotic image guided system (b) Shark - Tortoise - Otter

23

https://telegram.me/CivilServices_UPSC
https://t.me/UPSC_PDF

GENERAL SCIENCE

(c) Tortoise - Shark - Otter 29. With regard to the transmission of the Human
(d) Shark - Otter - Tortoise. Immunodeficiency Virus, which one of the
following statements is not correct ?
26. Consider the following statements : (a) The chances of transmission from female
to male are twice as likely as from male
1. Hepatitis B is several times more
to female
infectious than HIV/AIDS
(b) The chances of transmission are more if
2. Hepatitis B can cause liver cancer
a person suffers from other sexually
Which of the statements given above is /are
transmitted infections
correct ?
(c) An infected mother can transmit the
(a) 1 only
infection to her baby during pregnancy, at
(b) 2 only childbirth and by breast feeding
(c) Both 1 and 2 (d) The risk of contracting infection from
(d) Neither 1 nor 2 transfusion of infected blood is much
higher than an exposure to contaminated
27. Excessive release of the pollutant carbon needle
monoxide (CO) into the air may produce a
condition in which oxygen supply in the 30. What are the possible limitations of India in
human body decreases. What causes this mitigating the global warming at present and
condition? in the immediate future ?
(a) When inhaled into the human body, CO 1. Appropriate alternate technologies are
is converted into CO2 not sufficiently available.
(b) The inhaled CO has much higher affinity 2. India cannot invest huge funds in
for haemoglobin as compared to oxygen research and development.
(c) The inhaled CO destroys the chemical 3. Many developed countries have already
structure of haemoglobin set up their polluting industries in India.
(d) The inhaled CO adversely affects the Which of the statements given above is/ are
respiratory centre in the brain correct?
(a) 1 and 2 only
28. Consider the following statements : (b) 2 only
1. Every individual in the population is (c) 1 and 3 only
equally susceptible host for Swine Flu. (d) 1, 2 and 3
2. Antibiotics have no role in the primary
treatment of Swine Flu
31. Consider the following which can be found in
3. To prevent the future spread of Swine Flu the ambient atmosphere:
in the epidemic area, the swine (pigs)
1. Soot
must all be culled.
2. Sulphur hexafluoride
Which of the statements given above is/ are
3. Water vapour
correct?
Which of the above contribute to the warming
(a) 1 and 2 only
up of the atmosphere?
(b) 2 only
(a) 1 and 2 only
(c) 2 and 3 only
(b) 3 only
(d) 1, 2 and 3
(c) 2 and 3 only
(d) 1, 2 and 3

24

https://telegram.me/CivilServices_UPSC
https://t.me/UPSC_PDF

ANSWERS - GENERAL SCIENCE

ANSWERS OF GENERAL SCIENCE

2019 7. (d) 1, 2 and 3


8. (c) Reproductive cloning of animals
1. (d) 1, 2 and 3
2. (d) Waste-to-energy technologies 2016
3. (b) 3 and 4
4. * 1. (b) the full range of mRNA molecules
5. (b) 2 and 3 only expressed by an organism
6. (d) 1, 2 and 3 2. (a) immunization of children and
7. (a) 1, 2 and 4 pregnant women
8. (b) 'Gravitational waves' were detected. 3. (b) wireless communication technology
9. (d) 2, 3 and 4 4. (c) Torpedo launch and recovery vessel
10. (a) A molecular scissors used in 5. (a) Electric plane tested by NASA
targeted gene editing 6. (d) 1, 2 and 3
11. (b) Hepatitis B, unlike Hepatitis C, does 7. (b) 2 and 3 only
not have a vaccine. 8. (d) It can build fusion reactors for power
12. (d) Neither 1 nor 2 generation
9. (c) Both 1 and 2
10. (d) Neither 1 nor 2
2018 11. (c) 1 and 3 only
12. (c) Both 1 and 2
1. (d) 1, 2, 3, 4 and 5 13. (b) Neem coating slows down the rate of
dissolution of urea in the soil
2. (d) 1, 2 and 3
3. (c) 1 and 3 only
4. (c) Cyber attacks 2015
5. (a) 1 only
6. (c) An American anti-missile system 1 (d) Swine flu
7. (a) 1 only 2. (d) Neither 1 nor 2
8. (d) 1, 2 and 3 3. (a) 1 only
9. (b) 2 only 4. (c) 2 and 3 only
10. (b) 2 only 5. (b) Guinea, Sierra Leone and Liberia
11. (b) Internet of Things 6. (a) 1 and 2
12. (d) WTO 7. (c) 1 and 3
13. (d) 1, 2, 3 and 4 8. (c) Both 1 and 2
14. (d) 1, 2 and 3 9. (c) Gangetic dolphin
10. (c) 1 and 3 only
2017
2014
1. (a) Observation and understanding of
the Universe 1 (b) The gum made from its seeds is
2. (d) 1, 2 and 3 used in the extraction of shale gas
3. (c) 1, 2 and 3 2 (a) 1 and 2 only
4. (b) To detect gravitational waves 3 (d) 1, 2 and 3
5. (b) 2 and 3 only 4 (b) Free energy is converted into
6. (c) Both 1 and 2 potential energy and stored

25

https://telegram.me/CivilServices_UPSC
https://t.me/UPSC_PDF

ANSWERS - GENERAL SCIENCE

5 (d) 1, 2 and 3 12 (c) 1, 2 and 4 only


6 (c) 1 and 3 only 13 (b) 1 and 3 only
7 (b) 3 only 14 (b) 2 and 3 only
8 (b) 2 and 3 15 (c) 1, 3 and 4 only
9 (d) 1,2 and 3 16 (b) 2 and 3 only
10 (a) Crab 17 (a) The Earth’s magnetic field diverts
11 (a) Diatoms—Crustaceans—Herrings them towards its poles
12 (a) 1 only 18 (c) 1 and 3 only
13 (b) 3 only
2011
2013
1. (d) 1, 2 and 3
1 (c) moulds 2. (a) 1, 2 and 3 only
2 (b) 2 and 3 only 3. (c) 1 and 3 only
3 (b) Ecological niche 4. (d) 1, 2 and 3
4 (a) NO2, O3 and peroxyacetyl nitrate in 5. (b) Presence of prominent polar front
the presence of sunlight and stratospheric clouds; and inflow
5 (c) 1 and 3 only of chlorofluorocarbons.
6 (c) 1 and 3 only 6. (b) Roots are starved of energy
7 (b) concentration of carbon dioxide in 7. (b) 2 only
the environment is increased 8. (b) 2 only
8 (c) the effective area of contact between 9. (a) To make it pest-resistant.
the wheel and axle is reduced 10. (a) Slow down the speed of neutrons.
9 (c) 1, 2 and 4 11. (d) 1, 2 and 3
10 (d) 1, 2 and 3 12. (c) 2, 3 and 4 only
11 (c) most of the root heirs are lost during 13. (c) The density of water is maximum at
transplantation 40C.
12 (a) 1 only 14. (a) Bluetooth uses 2.4 GHz radio
13 (d) 1, 2 and 3 frequency band, whereas Wi-Fi can
14 (a) Gravity is the strongest of the four use 2.4 GHz or 5 GHz frequency
15 (a) 1 only band.
16 (d) 1, 2 and 3 15. (d) Aspartame is several times sweeter
than table sugar, hence food items
made with small quantities of
2012
aspartame yield fewer calories on
oxidation.
1 (d) 1, 2 and 3 16. (c) 1 and 3 only
2 (d) the infrared part of the solar radiation 17. (a) It is an eco-friendly technology for
3 (b) Carbon, Hydrogen, Nitrogen the remediation of oily sludge and oil
4 (c) 3 and 4 only spills.
18. (a) O positive
5 (c) 1, 2 and 4 only 19. (b) 1 and 2 only
6 (d) 1, 2 and 3 20. (d) It fills the air spaces in the soil with
7 (a) Great Indian Bustard, Musk Deer, Red water.
Panda and Asiatic Wild Ass 21. (b) 2 only
8 (b) 1, 3 and 4 only 22. (d) provides the necessary acceleration
9 (a) Oryx is adapted to live in hot and arid for its motion
areas whereas Chiru is adapted to live 23. (b) 1 and 3 only
in steppes and semi-desert areas of 24. (c) They neutralize the free radicals
cold high mountains. produced in the body during
metabolism
10 (d) 1, 2 and 3
25. (c) Both 1 and 2
11 (a) 1 only

26

https://telegram.me/CivilServices_UPSC
https://t.me/UPSC_PDF

ANSWERS - GENERAL SCIENCE

26. (b) It is a computer network across a 24. (a) Breakdown of proteins into amino
public internet that provides users acids
access to their organization’s
network while maintaining the 25. (b) Shark - Tortoise - Otter
security of the information 26. (c) Both 1 and 2
transmitted 27. (b) The inhaled CO has much higher
affinity for haemoglobin as compared
2010 to oxygen
28. (c) 2 and 3 only
1. (c) Potato contains water whereas paper 29. (a) The chances of transmission from
does not contain water female to male are twice as likely as
2. (d) 1, 2 and 3 from male to female
3. (b) Its grains contain pro-vitamin A which 30. (a) 1 and 2 only
upon ingestion is converted to 31. (d) 1, 2 and 3
vitamin A in the human body
4. (b) The formation of hypochlorous acid
when chlorine is added to water
5. (d) 1, 2, 3 and 4
6. (d) Neither 1 nor 2
7. (a) Malaria is caused by several species
of Plasmodium
8. (d) Hawk
9. (b) 1, 2 and 3 only
10. (c) H2O
11. (d) It is a procedure that can be done on
the person of any age
12. (c) 2 and 3 only
13. (b) 2 only
14. (b) UN Conference on Environment and
Development, Rio de Janeiro, 1992
15. (d) Pseudomonas
16. (c) Both 1 and 2
17. (b) They are adapted to grow in nitrogen
deficient soils and thus depend on
insects for sufficient nitrogenous
nutrition
18. (c) Land covered with fresh snow
19. (b) Convection
20. (c) DSL
21. (d) It can map the spread of tumour in the
body
22. (c) It is an oviparous snake and lays its
eggs in the nest and guards the nest
until they are hatched
23. (b) When the lamp is switched on, the
mercury in the lamp causes the
emission of ultra-violet radiations

27

https://telegram.me/CivilServices_UPSC
https://t.me/UPSC_PDF

UPSC
PRELIMINARY EXAMINATION
(PAPER-I)
GENERAL STUDIES

PREVIOUS YEARS QUESTIONS

(YEAR 2010 TO 2019)


THEME  CURRENT EVENTS OF NATIONAL
AND INTERNATIONAL IMPORTANCE

https://telegram.me/CivilServices_UPSC
https://t.me/UPSC_PDF

CURRENT EVENTS OF NATIONAL AND INTERNATIONAL IMPORTANCE

2019 2018

1. Which of the following adopted a law on 1. What is/are the consequence/ consequences
data protection and privacy for its citizens of a country becoming the member of the
known as 'General Data Protection 'Nuclear Suppliers Group'?
Regulation' in April 2016 and started 1. It will have access to the latest and most
implementation of it from 25th May, 2018? efficient nuclear technologies.
(a) Australia 2. It automatically becomes a member of
(b) Canada "The Treaty on the Non-Proliferation of
(c) The European Union Nuclear Weapons (NPT)".
(d) The United States of America Which of the statements given above is/ are
correct ?
2. Recently, India signed a deal known as (a) 1 only
'Action Plan for Prioritization and (b) 2 only
Implementation of Cooperation Areas in the (c) Both 1 and 2
Nuclear Field' with which of the following (d) Neither 1 nor 2
countries?
(a) Japan 2. In the Indian context, what is the implication
(b) Russia of ratifying the 'Additional Protocol' with the
(c) The United Kingdom international Atomic Energy Agency (IAEA)'?
(d) The United States of America (a) The civilian nuclear reactors come under
IAEA safeguards.
3. Consider the following statements : (b) The military nuclear installations come
1. The United Nations Convention under the inspection of IAEA.
against Corruption (UNCAC) has a (c) The country will have the privilege to buy
'Protocol against the Smuggling of uranium from the Nuclear Suppliers
Migrants by Land, Sea and Air'. Group (NSG).
2. The UNCAC is the ever-first legally (d) The country automatically becomes a
binding global anti-corruption member of the NSG.
instrument.
3. A highlight of the United Nations 3. Consider the following countries
Convention against Transnational 1. Australia
Organized Crime (UNTOC) is the 2. Canada
inclusion of a specific chapter aimed
at returning assets to their rightful 3. China
owners from whom they had been 4. India
taken illicitly. 5. Japan
4. The United Nations Office on Drugs
6. USA
and Crime (UNODC) is mandated by
Which of the above are among the 'free-trade
its member States to assist in the
partners' of ASEAN?
implementation of both UNCAC and
UNTOC. (a) 1, 2, 4 and 5
Which of the statements given above are (b) 3, 4, 5 and 6
correct? (c) 1, 3, 4 and 5
(a) 1 and 3 only (d) 2, 3, 4 and 6
(b) 2, 3 and 4 only
(c) 2 and 4 only 4. Regarding Wood's Dispatch, which of the
(d) 1, 2, 3 and 4 following statements are true ?
1. Grants-in-Aid system was introduced.

https://telegram.me/CivilServices_UPSC
https://t.me/UPSC_PDF

CURRENT EVENTS OF NATIONAL AND INTERNATIONAL IMPORTANCE

2. Establishment of universities was 1. It is a centrally sponsored scheme for


recommended. developing every city of our country into
3. English as a medium of instruction at all Smart Cities in a decade.
levels of education was recommended. 2. It is an initiative to identify new digital
Select the correct answer using the code technology innovations for solving the
given below : many problems faced by our country.
(a) 1 and 2 only 3. It is a programme aimed at making all the
(b) 2 and 3 only financial transactions in our country
completely digital in a decade.
(c) 1 and 3 only
Select the correct answer using the code
(d) 1, 2 and 3
given below :
(a) 1 and 3 only
5. The term "two-state solution" is sometimes
(b) 2 only
mentioned in the news in the context of the
affairs of (c) 3 only
(a) China (d) 2 and 3 only
(b) Israel
(c) Iraq 4. The term 'Digital Single Market Strategy'
seen in the news refers to
(d) Yemen
(a) ASEAN
(b) BRICS
2017 (c) EU
(d) G20
1. 'Recognition of Prior Learning Scheme' is
sometimes mentioned in the news with 5. With reference to the 'Prohibition of Benami
reference to Property Transactions Act, 1988 (PBPT Act)',
(a) Certifying the skills acquired by consider the following statements :
construction workers through traditional 1. A property transaction is not treated as a
channels. benami transaction if the owner of the
(b) Enrolling the persons in Universities for property is not aware of the transaction.
distance learning programmes. 2. Properties held benami are liable for
(c) Reserving some skilled jobs to rural and confiscation by the Government.
urban poor in some public sector 3. The Act provides for three authorities for
undertakings. investigations but does not provide for
(d) Certifying the skills acquired by trainees any appellate mechanism.
under the National Skill Development Which of the statements given above is/ are
Programme. correct ?
(a) 1 only
2. Which of the following gives 'Global Gender (b) 2 only
Gap Index' ranking to the countries of the (c) 1 and 3 only
world ?
(d) 2 and 3 only
(a) World Economic Forum
(b) UN Human Rights Council
6. Which of the following are the objectives of
(c) UN Women 'National Nutrition Mission' ?
(d) World Health Organization
1. To create awareness relating to
malnutrition among pregnant women and
3. Which of the following statements is/are lactating mothers.
correct regarding Smart India Hackathon 2. To reduce the incidence of anaemia
2017 ? among young children, adolescent girls
and women.

https://telegram.me/CivilServices_UPSC
https://t.me/UPSC_PDF

CURRENT EVENTS OF NATIONAL AND INTERNATIONAL IMPORTANCE

3. To promote the consumption of millets, (d) Neither 1 nor 2


coarse cereals and unpolished rice.
4. To promote the consumption of poultry 10. With reference to 'Asia Pacific Ministerial
eggs. Conference on Housing and Urban
Select the correct answer using the code Development (APMCHUD)', consider the
given below : following statements :
(a) 1 and 2 only 1. The first APMCHUD was held in India in
(b) 1, 2 and 3 only 2006 on the theme 'Emerging Urban
(c) 1, 2 and 4 only Forms — Policy Responses and
Governance Structure'.
(d) 3 and 4 only
2. India hosts all the Annual Ministerial
Conferences in partnership with ADB,
7. The term 'Domestic Content Requirement' is
APEC and ASEAN.
sometimes seen in the news with reference
Which of the statements given above is/ are
to
correct ?
(a) Developing solar power production in our
(a) 1 only
country
(b) 2 only
(b) Granting licences to foreign T.V.
channels in our country (c) Both 1 and 2
(c) Exporting our food products to other (d) Neither 1 nor 2
countries
(d) Permitting foreign educational institutions 11. With reference to the role of U N-Habitat in
to set up their campuses in our country the United Nations programme working
towards a better urban future, which of the
statements is/are correct ?
8. Consider the following statements :
1. UN-Habitat has been mandated by the
1. The Nuclear Security Summits are
United Nations General Assembly to
periodically held under the aegis of the
promote socially and environmentally
United Nations
sustainable towns and cities to provide
2. The International Panel on Fissile
adequate shelter for all.
Materials is an organ of International
2. Its partners are either governments or
Atomic Energy Agency.
local urban authorities only.
Which of the statements given above is/ are
3. UN-Habitat contributes to the overall
correct ?
objective of the United Nations system to
(a) 1 only
reduce poverty and to promote access to
(b) 2 only safe drinking water and basic sanitation.
(c) Both 1 and 2 Select the correct answer using the code
(d) Neither 1 nor 2 given below :
(a) 1, 2 and 3
9. With reference to 'Quality Council of India (b) 1 and 3 only
(QCI)', consider the following statements: (c) 2 and 3 only
1. QCI was set up jointly by the Government (d) 1 only
of India and the Indian Industry.
2. Chairman of QCI is appointed by the
12. Consider the following in respect of 'National
Prime Minister on the recommendations
Career Service' :
of the industry to the Government.
1. National Career Service is an initiative of
Which of the above statements is/are correct
the Department of Personnel and
?
Training, Government of India.
(a) 1 only
2. National Career Service has been
(b) 2 only launched in a Mission Mode to improve
(c) Both 1 and 2

https://telegram.me/CivilServices_UPSC
https://t.me/UPSC_PDF

CURRENT EVENTS OF NATIONAL AND INTERNATIONAL IMPORTANCE

the employment opportunities to Which of the above statements is/are correct


uneducated youth of the country. ?
Which of the above statements is/are correct (a) 1 only
? (b) 2 only
(a) 1 only (c) Both 1 and 2
(b) 2 only (d) Neither 1 nor 2
(c) Both 1 and 2
(d) Neither 1 nor 2 16. The term 'M-STrIPES' is sometimes seen in
the news in the context of
13. Which of the following statements best (a) Captive breeding of Wild Fauna
describes the term 'Scheme for Sustainable (b) Maintenance of Tiger Reserves
Structuring of Stressed Assets (S4A)', (c) Indigenous Satellite Navigation System
recently seen in the news?
(d) Security of National Highways
(a) It is a procedure for considering
ecological costs of developmental
17. 'Broad-based Trade and Investment
schemes formulated by the Government.
Agreement (BTIA)' is sometimes seen in the
(b) It is a scheme of RBI for reworking the
news in the context of negotiations held
financial structure of big corporate
between India and
entities facing genuine difficulties.
(a) European Union
(c) It is a disinvestment plan of the
(b) Gulf Cooperation Council
Government regarding Central Public
Sector Undertakings. (c) Organization for Economic
Cooperation and Development
(d) It is an important provision in 'The
Insolvency and Bankruptcy Code' (d) Shanghai Cooperation Organization
recently implemented by the
Government. 18. Consider the following statements :
1. India has ratified the Trade Facilitation
14. Consider the following statements : Agreement (TFA) of WTO.
1. Climate and Clean Air Coalition (CCAC) 2. TFA is a part of WTO's Bali Ministerial
to Reduce Short Lived Climate Pollutants Package of 2013.
is a unique initiative of G20. 3. TFA came into force in January 2016.
2. The CCAC focuses on methane, black Which of the statements given above is/ are
carbon and hydrofluorocarbons. correct ?
Which of the statements given above is/ are (a) 1 and 2 only
correct ? (b) 1 and 3 only
(a) 1 only (c) 2 and 3 only
(b) 2 only (d) 1, 2 and 3
(c) Both 1 and 2
(d) Neither 1 nor 2 19. What is the importance of developing
Chabahar Port by India ?
15. Consider the following in respect of Indian (a) India's trade with African countries will
Ocean Naval Symposium (IONS): enormously increase.
1. Inaugural IONS was held in India in 2015 (b) India's relations with oil-producing Arab
under the chairmanship of the Indian countries will be strengthened.
Navy. (c) India will not depend on Pakistan for
2. IONS is a voluntary initiative that seeks to access to Afghanistan and Central Asia.
increase maritime cooperation among (d) Pakistan will facilitate and protect the
navies of the littoral states of the Indian installation of a gas pipeline between Iraq
Ocean Region. and India.

https://telegram.me/CivilServices_UPSC
https://t.me/UPSC_PDF

CURRENT EVENTS OF NATIONAL AND INTERNATIONAL IMPORTANCE

education of rural and urban poor, and


20. In India, it is legally mandatory for which of organizing skill development programmes
the following to report on cyber security and vocational training for them.
incidents ?
1. Service providers
2016
2. Data centres
3. Body corporate
1. With reference to the Trans-Pacific
Select the correct answer using the code
Partnership', consider the following
given below :
statements :
(a) 1 only
1. It is an agreement among all the Pacific
(b) 1 and 2 only Rim countries except China and Russia.
(c) 3 only 2. It is a strategic alliance for the purpose of
(d) 1, 2 and 3 maritime security only.
Which of the statements given above is/ are
21. What is the purpose of 'Vidyanjali Yojana'? correct?
1. To enable the famous foreign educational (a) 1 only
institutions to open their campuses in (b) 2 only
India. (c) Both 1 and 2
2. To increase the quality of education (d) Neither 1 nor 2
provided in government schools by taking
help from the private sector and the
community. 2. Consider the following statements : The
India-Africa Summit
3. To encourage voluntary monetary
1. held in 2015 was the third such Summit
contributions from private individuals and
organizations so as to improve the 2. was actually initiated by Jawaharlal
infrastructure facilities for primary and Nehru in 1951
secondary schools. Which of the statements given above is/ are
Select the correct answer using the code correct?
given below : (a) 1 only
(a) 2 only (b) 2 only
(b) 3 only (c) Both 1 and 2
(c) 1 and 2 only (d) Neither 1 nor 2
(d) 2 and 3 only
3. 'Doctors Without Borders (Medecins Sans
22. What is the aim of the programme 'Unnat Frontieres)', often in the news, is
Bharat Abhiyan' ? (a) a division of World Health Organization
(a) Achieving 100% literacy by promoting (b) a non-governmental international
collaboration between voluntary organization
organizations and government's (c) an inter-governmental agency sponsored
education system and local communities. by European Union
(b) Connecting institutions of higher (d) a specialized agency of the United
education with local communities to Nations
address development challenges through
appropriate technologies.
4. Consider the following statements :
(c) Strengthening India's scientific research
1. New Development Bank has been set up
institutions in order to make India a by APEC.
scientific and technological power.
2. The headquarters of N ew Development
(d) Developing human capital by allocating Bank is in Shanghai.
special funds for health care and

https://telegram.me/CivilServices_UPSC
https://t.me/UPSC_PDF

CURRENT EVENTS OF NATIONAL AND INTERNATIONAL IMPORTANCE

Which of the statements given above is/ are 2. Madhesi : Nepal


correct?
3. Rohingya : Myanmar
(a) 1 only
Which of the pairs given above is/are
(b) 2 only
correctly matched?
(c) Both 1 and 2
(a) 1 and 2
(d) Neither 1 nor 2
(b) 2 only
(c) 2 and 3
5. Which of the following is not a member of
(d) 3 only
'Gulf Cooperation Council'?
(a) Iran
10. With reference to 'Organization for the
(b) Saudi Arabia
Prohibition of Chemical Weapons (OPCW)',
(c) Oman
consider the following statements :
(d) Kuwait
1. It is an organization of European Union in
working relation with NATO and WHO.
6. 'Belt and Road Initiative' is sometimes 2. It monitors chemical industry to prevent
mentioned in the news in the context of the new weapons from emerging.
affairs of
3. It provides assistance and protection to
(a) African Union States (Parties) against chemical
(b) Brazil weapons threats.
(c) European Union Which of the statements given above is/ are
(d) China correct?
(a) 1 only
7. 'European Stability Mechanism', sometimes (b) 2 and 3 only
seen in the news, is an (c) 1 and 3 only
(a) agency created by EU to deal with the (d) 2 and 3
impact of millions of refugees arriving
from Middle East
(b) agency of EU that provides financial 2015
assistance to eurozone countries
(c) agency of EU to deal with all the bilateral 1. 'Beijing Declaration and Platform for 'Action',
and multilateral agreements on trade often seen in the news, is
(d) agency of EU to deal with the conflicts (a) a strategy to tackle the regional terrorism,
arising among the member countries an outcome of a meeting of the Shanghai
Cooperation Organization
8. A recent movie titled The Man Who Knew (b) a plan of action for sustainable economic
Infinity is based on the biography of growth in the Asia-Pacific Region, an
(a) S. Ramanujan outcome of the deliberations of the Asia-
Pacific Economic Forum
(b) S. Chandrasekhar
(c) an agenda for women's empowerment,
(c) S. N. Bose
an outcome of a World Conference
(d) C. V. Raman
convened by the United Nations
(d) a strategy to combat wildlife trafficking, a
9. Consider the following pairs : declaration of the East Asia Summit.
Community In the affairs of
sometimes
2. Consider the following countries :
mentioned in the
1. China
news
2. France.
1. Kurd : Bangladesh
3. India

https://telegram.me/CivilServices_UPSC
https://t.me/UPSC_PDF

CURRENT EVENTS OF NATIONAL AND INTERNATIONAL IMPORTANCE

4. Israel
5. Pakistan 6. In the context of modern scientific research,
Which among the above are Nuclear consider the following statements about
Weapons States as recognized by the Treaty 'IceCube', .a particle detector located at
on the Non-Proliferation of Nuclear Weapons, South Pale, which was recently in the news:
commonly known as Nuclear Non- (1) It is the world's largest neutrino detector,
proliferation Treaty (NPT)? encompassing a cubic kilometre of ice.
(a) 1 and 2 only (2) It is a powerful telescope to search for
(b) 1, 3, 4 and 5 only dark matter.
(c) 2, 4 and 5 only (3) It is buried deep in the ice.
(d) 1, 2, 3, 4 and 5 Which of the statements given above is/ are
correct?
3. Which of the following has/have been (a) 1 only
accorded 'Geographical Indication' status? (b) 2 and 3 only
1. Banaras Brocadeand Sarees (c) 1 and 3 only
2. Rajasthani Daal-Bati-Churma (d) 1, 2 and 3
3. Tirupathi Laddu
Select the correct answer using the code 7. What is Rio+20 Conference, often mentioned
given below. in the news?
(a) 1 only (a) It is the United Nations Conference on
(b) 2 and 3 only Sustainable Development
(c) 1 and 3 only (b) It is a Ministerial Meeting of the World
Trade Organization
(d) 1, 2 and 3
(c) 'It is a, Conference of, the
Intergovernmental Panel on Climate
4. The Fortaleza Declaration', recently in the
Change
news related to the affairs of
(d) It is a Conference of the Member
(a) ASEAN
Countries of the Convention on Biological
(b) BRICS Diversity
(c) OECD
(d) WTO 8. The term 'Goldilocks Zone' is often seen in
the news in the context of
5. Which of the following statements is/are (a) the limits of habitable zone above the
correct regarding National Innovation surface of the Earth
Foundation-India (NW)? (b) regions inside the Earth where shale gas
1. NIF is an autonomous body of the is available
Department of Science and Technology (c) search for the Earth-like planets in outer
under the Central Government space
2. NIF is an initiative to strengthen the (d) search for meteorites containing precious
highly advanced scientific research in metals
India's premier scientific institutions in
collaboration with highly advanced
9. Which of the following statements regarding
foreign scientific institutions.
Green Climate Fund' is/are correct?
Select the correct answer using the code
1. It is intended to assist the developing
given below.
countries in adaptation and mitigation
(a) 1 only
practices to counter climate change.
(b) 2 only
2. It is founded under the aegis of UNEP,
(c) Both 1 and 2 OECD, Asian Development Bank and
(d) Neither 1 nor 2 World Bank.

https://telegram.me/CivilServices_UPSC
https://t.me/UPSC_PDF

CURRENT EVENTS OF NATIONAL AND INTERNATIONAL IMPORTANCE

Select the correct answer using the code (b) Geographical Indications of Goods
given below. (Registration and Protection) Act, 1999
(a) 1 only (c) Environment (Protection) Act, 1986
(b) 2 only (d) Wildlife (Protection) Act, 1972
(c) Both 1 and 2
(d) Neither 1 nor 2 15. In the Mekong-Ganga Cooperation, an
initiative of six countries, which of the
10. Indira Gandhi Prize for Peace, Disarmament following is/are not a participant/ participants?
and Development for 2014 was given to 1. Bangladesh
which one of the following? 2. Cambodia
(a) Bhabha Atomic Research Centre 3. China
(b) Indian Institute of Science 4. Myanmar
(c) Indian Space Research Organization 5. Thailand
(d) Tata Institute of Fundamental Research Select the correct answer using the code
given below.
11. Amnesty International is (a) 1 only
(a) an agency of the United Nations to help (b) 3 and 4
refugees of civil wars (c) 1 and 3
(b) a global Human Rights Movement (d) 1, 2 and 5
(c) a nongovernmental voluntary
organization to help very poor people 16. 'Basel-III Accord' or simply 'Basel-III', often
(d) an inter-governmental agency to cater to seen in the hews, seeks to
medical emergencies in war-ravaged (a) develop national strategies for the
regions conservation and sustainable use of
biological diversity
12. 'BioCarbon Fund Initiative for Sustainable (b) improve banking sector's ability to deal
Forest Landscapes' is managed by the with financial and economic stress and
(a) Asian Development Bank improve risk management
(b) International Monetary Fund (c) reduce the greenhouse gas emissions
(c) United Nations Environment Programme but places a heavier burden on
developed countries
(d) World Bank
(d) transfer technology froth developed
countries to poor countries to enable
13. India is a member of which among the
them to replace the use of
following?
chlorofluorocarbons in refrigeration with
1 Asia-Pacific Economic Cooperation harmless chemicals.
2. Association of South-East Asian Nations
3. East Asia Summit 17. With reference to 'Indian Ocean Rim
Select the correct answer using the code Association for Regional Cooperation (IOR-
given below. ARC)', consider the following statements :
(a) 1 and 2 Only (1) It was established very recently in
(b) 3 only response to incidents of piracy and
(c) 1, 2 and 3 accidents of oil spills.
(d) India is a member of none of them (2) It is an affiance meant for, maritime
security only.

14. The Genetic Engineering Appraisal Which of the statements given above is/ are
Committee is constituted under the correct?

(a) Food Safety and Standards Act, (a) 1 Only


(b) 2 only

https://telegram.me/CivilServices_UPSC
https://t.me/UPSC_PDF

CURRENT EVENTS OF NATIONAL AND INTERNATIONAL IMPORTANCE

(c) Both 1 and 2 21. Which one of the followings given classical
(d) Neither 1 nor 2 language status recently?
(a) Odia
18. In a particular region in India, the local people (b) Konkani
train the roots of living trees into robust (c) Bhojpuri
bridges across the streams. As the time (d) Assamese
passes, these bridges become stronger.
These unique 'living root bridges' are found in
(a) Meghalaya 2014
(b) Himachal Pradesh
(c) Jharkhand 1. Recently, a series of uprisings of people
(d) Tamil Nadu referred to as 'Arab Spring' originally started
from
(a) Egypt
19. The term 'IndARC', sometimes seen in the
news, is the name of (b) Lebanon
(a) an indigenously developed radar system (c) Syria
inducted into Indian Defence (d) Tunisia
(b) India's 'satellite to provide services to the
countries of Indian Ocean Rim 2. Consider the following countries
(c) a scientific establishment set up by India 1. Denmark
in Antarctic region 2. Japan
(d) India's underwater observatory to 3. Russian Federation
scientifically study the Arctic region
4. United Kingdom
5. United States of America
20. With reference to 'Forest Carbon Partnership
Which of the above are the members of the
Facility', which of the following statements 'Arctic Council'?
is/are correct?
(a) 1, 2 and 3
1. It is a global partnership of governments,
(b) 2, 3 and 4
businesses, civil society and indigenous
peoples. (c) 1, 4 and 5
2. It provides financial aid to universities, (d) 1, 3 and 5
individual scientists and institutions
involved in scientific forestry research to 3. Consider the following pairs :
develop eco-friendly and climate Region often in Country
adaptation technologies for sustainable News
forest management.
3. It assists the countries in their 'REDD+ 1. Chechnya : Russian
(Reducing Emissions from Deforestation Federation
and Forest Degradation+)' efforts by
2. Darfur : Mali
providing them with financial and
technical assistance.
3. Swat Valley : Iraq
Select the correct answer using the code
given below. Which of the above pairs is/are correctly
(a) 1 only matched?
(b) 2 and 3 only (a) 1 only
(c) 1 and 3 only (b) 2 and 3 only
(d) 1, 2 and 3 (c) 1 and 3 only
(d) 1, 2 and 3

10

https://telegram.me/CivilServices_UPSC
https://t.me/UPSC_PDF

CURRENT EVENTS OF NATIONAL AND INTERNATIONAL IMPORTANCE

7. With reference to Agni—IV Missile, which of 1. The First Summit of BRICS was held in
the following statements is/are correct? Rio de Janeiro in 2009.
1. It is a surface-to-surface missile. 2. South Africa was the last to join the
2. It is fuelled by liquid propellant only. BRICS grouping.
3. It can deliver one-tonne nuclear Which of the statements given above is/ are
warheads about 7500 km away. correct ?
Select the correct answer using the code (a) 1 only
given below. (b) 2 only
(a) 1 only (c) Both 1 and 2
(b) 2 and 3 only (d) Neither 1 nor 2
(c) 1 and 3 only
(d) 1, 2 and 3
2012
1. Which of the following can be said to be
8. Which of the following pairs is/are correctly
essentially the parts of 'Inclusive Governance
matched?
?
Spacecraft Purpose
1. Permitting the Non-Banking Financial
1. Cassini- : Orbiting the Venus Companies to do banking.
Huygens and Transmitting 2. Establishing effective District Planning
data to the Earth Committees in all the districts
3. Increasing the government spending on
2. Messenger : Mapping and public health
investigating the 4. Strengthening the Mid-day Meal Scheme
Mercury
Select the correct answer using the codes
3. Voyager 1 and 2 : Exploring the outer given below:
solar system (a) 1 and 2 only
(b) 3 and 4 only
Select the correct answer using the code (c) 2, 3 and 4 only
given below. (d) 1, 2, 3 and 4
(a) 1 only
(b) 2 and 3 only 2. With reference to National Rural Health
(c) 1 and 3 only Mission, which of the following are the jobs of
(d) 1, 2 and 3 'ASHA', a trained community health worker?
1. Accompanying women to the health
9. Consider the following languages : facility for antenatal care checkup
1. Gujarati 2. Using pregnancy test kits for early
2. Kannada detection of pregnancy
3. Telugu 3. Providing information on nutrition and
immunization
Which of the above has/have been declared
as 'Classical Language/ Languages' by the 4. Conducting the delivery of baby
Government? Select the correct answer using the codes
(a) 1 and 2 only given below :
(b) 3 only (a) 1, 2 and 3 only
(c) 2 and 3 only (b) 2 and 4 only
(d) 1, 2 and 3 (c) 1 and 3 only
(d) 1, 2, 3 and 4
10. With reference to a grouping of countries
known as BRICS consider the following
statements

11

https://telegram.me/CivilServices_UPSC
https://t.me/UPSC_PDF

CURRENT EVENTS OF NATIONAL AND INTERNATIONAL IMPORTANCE

3. How does the National Rural Livelihood


Mission seek to improve livelihood options of 3. With reference to "Look East Policy" of India,
rural poor? consider the following statements:
1. By setting up a large number of new 1. India wants to establish itself as an
manufacturing industries and important regional player in the East
agribusiness centres in rural areas Asian affairs.
2. By strengthening 'self-help groups' and 2. India wants to plug the vacuum created
providing skill development by the termination of Cold War.
3. By supplying seeds, fertilizers, diesel 3. India wants to restore the historical and
pump-sets and micro- irrigation cultural ties with its neighbours in
equipment free of cost to farmers Southeast and East Asia.
Select the correct answer using the codes Which of the statements given above is/ are
given below : correct?
(a) 1 and 2 only (a) 1 only
(b) 2 only (b) 1 and 3 only
(c) 1 and 3 only (c) 3 only
(d) 1, 2 and 3 (d) 1, 2 and 3

2011 4. Consider the following :


1. Right to education
2. Right to equal access to public service.
1. The "New START" treaty was in the news.
What is this treaty ? 3. Right to food.
(a) It is a bilateral strategic nuclear arms Which of the above is/are Human Right/
reduction treaty between the USA and Human Rights under " Universal Declaration
the Russian Federation of Human Rights"?
(b) It is a multilateral energy security (a) 1 only
cooperation treaty among the members (b) 1 and 2 only
of the East Asia Summit (c) 3 only
(c) It is a treaty between the Russian (d) 1, 2 and 3
Federation and the European Union for
the energy security cooperation 5. Recently, the USA decided to support India's
(d) It is a multilateral cooperation treaty membership in multi-lateral export control
among the BRICS countries for the regimes called the "Australia Group" and the
promotion of trade "Wassenaar Arrangement". What is the
difference between them ?
2. Southeast Asia has captivated the attention 1. The Australia Group is an informal
of global community over space and time as arrangement which aims to allow
a geostrategically significant region. Which exporting countries to minimize the risk of
among the following is the most convincing assisting chemical and biological
explanation for this global perspective ? weapons proliferation, whereas the
(a) It was the hot theatre during the Second Wassenaar Arrangement is a formal
World War. group under the OECD holding identical
(b) Its location between the Asian powers of objectives.
China and India. 2. The Australia Group comprises
(c) It was the arena of superpower predominantly of Asian, African and
confrontation during the Cold War period. North American countries, whereas the
member countries of Wassenaar
(d) Its location between the Pacific and
Arrangement are predominantly from the
Indian oceans and its pre-eminent
European Union and American
maritime character.
continents.

12

https://telegram.me/CivilServices_UPSC
https://t.me/UPSC_PDF

CURRENT EVENTS OF NATIONAL AND INTERNATIONAL IMPORTANCE

Which of the statements given above is/ are 4. As regards the use of international food
correct? safety standards as reference point for the
(a) 1 only dispute settlements, which one of the
(b) 2 only following does WTO collaborate with ?
(c) Both 1 and 2 (a) Codex Alimentarius Commission
(d) Neither 1 nor 2 (b) International Federation of Standards
Users
(c) International Organization for
2010 Standardization
(d) World Standards Cooperation
1. Mon 863 is a variety of maize. It was in the
news for the following reason 5. India is a party to the Ramsar Convention
(a) It is a genetically modified dwarf variety and has declared many areas as Ramsar
which is resistant to drought Sites. Which of the following statements best
(b) It is a genetically modified variety which describes as to how we should maintain
is pest resistant these sites in the context of this Convention ?
(c) It is a genetically modified variety with ten (a) Keep all the sites completely inaccessible
times higher protein content than regular to man so that they will not be exploited
maize crop (b) Conserve all the sites through ecosystem
(d) It is a genetically modified variety used approach and permit tourism and
exclusively for bio-fuel production recreation only
(c) Conserve all the sites through ecosystem
2. Consider the following countries : approach for a period without any
exploitation, with specific criteria and
1. Brazil
specific period for each site, and then
2. Mexico allow sustainable use of them by future
3. South Africa generations
According to UNCTAD, which of the above (d) Conserve all the sites through ecosystem
is/are categorized as "Emerging approach and allow their simultaneous
Economies"? sustainable use
(a) 1 only
(b) 1 and 3 only 6. In the context of governance, consider the
(c) 2 and 3 only following:
(d) 1, 2 and 3 1. Encouraging Foreign Direct Investment
inflows
3. Stiglitz Commission established by the 2. Privatization of higher educational
President of the United Nations General Institutions
Assembly was in the international news. The 3. Down-sizing of bureaucracy
commission was supposed to deal with 4. Selling/offloading the shares of Public
(a) The challenges posed by the impending Sector Undertakings
global climate change and prepare a road Which of the above can be used as
map measures to control the fiscal deficit in India?
(b) The workings of the global financial (a) 1, 2 and 3
systems and to explore ways and means (b) 2, 3 and 4
to secure a more sustainable global order
(c) 1, 2 and 4
(c) Global terrorism and prepare a global
(d) 3 and 4 only
action plan for the mitigation of terrorism
(d) Expansion of the United Nations Security
Council in the present global scenario 7. As per the UN-Habitat's Global Report on
Human Settlements 2009, which one among
the following regions has shown the fastest

13

https://telegram.me/CivilServices_UPSC
https://t.me/UPSC_PDF

CURRENT EVENTS OF NATIONAL AND INTERNATIONAL IMPORTANCE

growth rate of urbanization in the last three 10. With reference to the Consumer Disputes
decades ? Redressal at district level in India, which one
(a) Asia of the following statements is not correct ?
(b) Europe (a) A State Government can establish more
(c) Latin America and Caribbean than one District Forum in a district if it
deems fit
(d) North America
(b) One of the members of the District Forum
shall be a woman
8. With reference to the N ational Rehabilitation
(c) The District Forum entertains the
and Resettlement Policy, 2007, consider the
complaints where the value of goods or
following statements:
services does not exceed rupees fifty
1. This policy is applicable only to the
lakhs
persons affected by the acquisition of
(d) A complaint in relation to any goods sold
land for projects and not to the
or any service provided may be filed with
involuntary displacement due to any
a District Forum by the State Government
other reason.
as a representative of the interests of the
2. This policy has been formulated by the
consumers in general
Ministry of Social Justice and
Empowerment.
11. In the context of bilateral trade negotiations
Which of the statements given above is/ are
between I ndia and European Union, what is
correct ?
the difference between European
(a) 1 only
Commission and European Council ?
(b) 2 only
1. European Commission represents the EU
(c) Both 1 and 2 in trade negotiations whereas European
(d) Neither 1 nor 2 Council participates in the legislation of
matters pertaining to economic policies of
9. Two of the schemes launched by the the European Union.'
Government of India for Women's 2. European Commission comprises the
development are Swadhar and Swayam Heads of State or government of member
Siddha. As regards the difference between countries whereas the European Council
them, consider the following statements: comprises of the persons nominated by
1. Swayam Siddha is meant for those in European Parliament.
difficult circumstances such as women Which of the statements given above is/ are
survivors of natural disasters or terrorism, correct ?
women prisoners released from jails, (a) 1 only
mentally challenged women etc. whereas (b) 2 only
Swadhar is meant for holistic
(c) Both 1 and 2
empowerment of women through Self
(d) Neither 1 nor 2
Help Groups.
2. Swayam Siddha is implemented through
Local Self Government bodies or reputed 12. With reference to the National Investment
Voluntary Organizations whereas Fund to which the disinvestment proceeds
Swadhar is implemented through the are routed, consider the following statements:
ICDS. units set up in the states. 1. The assets in the National Investment
Which of the statements given above is/ are Fund are managed by the Union Ministry
correct? of Finance.
(a) 1 only 2. The National Investment Fund is to be
maintained within the Consolidated Fund
(b) 2 only
of India.
(c) Both 1 and 2
3. Certain Asset Management Companies
(d) Neither 1 nor 2
are appointed as the fund managers.

14

https://telegram.me/CivilServices_UPSC
https://t.me/UPSC_PDF

CURRENT EVENTS OF NATIONAL AND INTERNATIONAL IMPORTANCE

4. A certain proportion of annual income is (c) 3 only


used for financing select social sectors. (d) 1, 2 and 3
Which of the statements given above is/ are correct
? 15. Consider the following statements :
(a) 1 and 2 1. The Union Government fixes the
(b) 2 only Statutory Minimum Price of sugarcane for
(c) 3 and 4 each sugar season.
(d) 3 only 2. Sugar and sugarcane are essential
commodities under the Essential
13. In India, which of the following is regulated by Commodities Act.
the Forward Markets Commission ? Which of the statements given above is/ are
(a) Currency Futures Trading correct ?
(b) Commodities Futures Trading (a) 1 only
(c) Equity Futures Trading (b) 2 only
(d) Both Commodities Futures and Financial (c) Both 1 and 2
Futures Trading (d) Neither 1 nor 2

14. With reference to India, consider the 16. The International Development Association, a
following: lending agency, is administered by the
1. Nationalization of Banks (a) International Bank for Reconstruction and
2. Formation of Regional Rural Banks Development
3. Adoption of villages by Bank Branches (b) International Fund for Agricultural
Development
Which of the above can be considered as
steps taken to achieve the "financial (c) United Nations Development Programme
inclusion" in India? (d) United Nations Industrial Development
(a) 1 and 2 only Organization
(b) 2 and 3 only

15

https://telegram.me/CivilServices_UPSC
https://t.me/UPSC_PDF

ANSWERS - CURRENT EVENTS OF NATIONAL AND INTERNATIONAL IMPORTANCE

ANSWERS OF CURRENT EVENTS OF NATIONAL AND


INTERNATIONAL IMPORTANCE

2019 19. (c) India will not depend on Pakistan for


access to Afghanistan and Central
Asia.
1. (c) The European Union 20. (d) 1, 2 and 3
2. (b) Russia
21. (a) 2 only
3. (b) 2, 3 and 4 only
22. (b) Connecting institutions of higher
education with local communities to
2018 address development challenges
through appropriate technologies.
1. (a) 1 only
2. (a) The civilian nuclear reactors come 2016
under IAEA safeguards.
3. (c) 1, 3, 4 and 5 1. (d) Neither 1 nor 2
4. (a) 1 and 2 only 2. (a) 1 only
5. (b) Israel 3. (b) a non-governmental international
organization
2017 4. (b) 2 only
5. (a) Iran
1. (a) Certifying the skills acquired by 6. (d) China
construction workers through 7. (b) agency of EU that provides financial
traditional channels. assistance to eurozone countries
2. (a) World Economic Forum 8. (a) S. Ramanujan
3. (b) 2 only 9. (c) 2 and 3
4. (c) EU 10. (b) 2 and 3 only
5. (b) 2 only
6. (a) 1 and 2 only 2015
7. (a) Developing solar power production in
our country
1 (c) an agenda for women’s
8. (d) Neither 1 nor 2
empowerment, an outcome of a
9. (c) Both 1 and 2 World Conference convened by the
10. (d) Neither 1 nor 2 United Nations
11. (b) 1 and 3 only 2. (a) 1 and 2 only
12. (d) Neither 1 nor 2 3. (c) 1 and 3 only
13. (b) It is a scheme of RBI for reworking 4. (b) BRICS
the financial structure of big 5. (a) 1 only
corporate entities facing genuine
6. (d) 1, 2 and 3
difficulties.
7. (a) It is the United Nations Conference
14. (b) 2 only
on Sustainable Development
15. (b) 2 only
8. (c) search for the Earth-like planets in
16. (b) Maintenance of Tiger Reserves outer space
17. (a) European Union 9. (a) 1 only
18. (a) 1 and 2 only 10. (c) Indian Space Research Organization
11. (b) a global Human Rights Movement
12. (d) World Bank
16

https://telegram.me/CivilServices_UPSC
https://t.me/UPSC_PDF

ANSWERS - CURRENT EVENTS OF NATIONAL AND INTERNATIONAL IMPORTANCE

13. (b) 3 only means to secure a more sustainable


14. (c) Environment (Protection) Act, 1986 global order
15. (b) 3 and 4 4. (a) Codex Alimentarius Commission
16. (b) improve banking sector’s ability to 5. (c) Conserve all the sites through
deal with financial and economic ecosystem approach for a period
stress and improve risk management without any exploitation, with specific
17. (d) Neither 1 nor 2 criteria and specific period for each
18. (a) Meghalaya site, and then allow sustainable use
19. (d) India’s underwater observatory to of them by future generations
scientifically study the Arctic region 6. (b) 2, 3 and 4
20. (c) 1 and 3 only 7. (a) Asia
21. (a) Odia
8. (a) 1 only
9. (d) Neither 1 nor 2
2014 10. (b) One of the members of the District
Forum shall be a woman
1 (d) Tunisia 11. (a) 1 only
2 (d) 1, 3 and 5 12. (c) 3 and 4
3 (a) 1 only 13. (b) Commodities Futures Trading
4 (a) 1 only
14. (d) 1, 2 and 3
5 (b) 2 and 3 only
15. (c) Both 1 and 2
6 (c) 2 and 3 only
16. (a) International Bank for Reconstruction
7 (b) 2 only
and Development

2012

1 (c) 2, 3 and 4 only


2 (a) 1, 2 and 3 only
3 (b) 2 only

2011

1. (a) It is a bilateral strategic nuclear arms


reduction treaty between the USA
and the Russian Federation
2. (d) Its location between the Pacific and
Indian oceans and its pre-eminent
maritime character.
3. (b) 1 and 3 only
4. (d) 1, 2 and 3
5. (a) 1 only

2010

1. (b) It is a genetically modified variety


which is pest resistant
2. (d) 1, 2 and 3
3. (b) The workings of the global financial
systems and to explore ways and

17

https://telegram.me/CivilServices_UPSC

You might also like